Combo with "NBDE Part II - 2006 Exam C" and 11 others

Pataasin ang iyong marka sa homework at exams ngayon gamit ang Quizwiz!

The speech sounds that bring the mandible closest to the maxilla are the A. "s" sounds. B. "f" and "v" sounds. C. "t" and "th" sounds. D. vowel sounds.

A. "s" sounds.

What is the mode of the following data set? 1, 1, 1, 1, 2, 2, 3, 3, 7, 9 (n=10; sum=30) A. 1 B. 2 C. 3

A. 1

Primary herpetic gingivostomatitis is MOST likely to occur in which of the following age groups? A. 1-5 years B. 6-10 years C. 11-15 years

A. 1-5 years

Primary herpetic gingivostomatitis is most likely to occur in which of the following age groups? A. 1-5 years B. 6-10 years C. 11-15 years D. 16-20 years E. 21-25 years

A. 1-5 years

Which of the following is the preferred pharmalogic treatment for a xerostomia? A. 5 mg Pilocarpine B. 1 mg Epinephrine C. 5 mg Levonordefrin D. 1 mg Malathion

A. 5 mg Pilocarpine

Excisional biopsy is most appropriate for which of the following conditions? A. A 1-cm exophytic mass on the buccal mucosa B. A 2-cm painless ulcer of ten weeks' duration C. A 3-cm submucosal indurated lump D. A leukoplakic area covering most of the buccal mucosa

A. A 1-cm exophytic mass on the buccal mucosa

Each of the following is likely to be a sign or symptom of a metastatic jaw lesion EXCEPT one. Which one is this EXCEPTION? A. A non-erupted tooth B. Paresthesia of the lower lip C. A symptomless radiolucency D. An unexplained mobility of a tooth

A. A non-erupted tooth

Which of the following most accurately describes the effect of individual plaque control instructions that are conducted in the classroom? A. A short-term decrease in gingivitis B. A long-term decrease in gingivitis C. A short-term decrease in caries D. A long-term decrease in caries

A. A short-term decrease in gingivitis

How do the surface characteristics of a restoration affect its perceived form? A. A surface smoother than normal will give the impression of a larger size. B. Increasing the value of the restoration makes the tooth appear smaller. C. Horizontal highlights give an illusion of increased length. D. Vertical highlights give an illusion of increased width.

A. A surface smoother than normal will give the impression of a larger size.

After a threshold stimulus, the cell-membrane permeability becomes altered. The liberation of which of the following transmitter substances causes this alteration? A. Acetylcholine B. Cholinesterase C. Hydroxycholine D. Acetylsalicylic acid

A. Acetylcholine

Although no causal relationships between particular bacteria and periodontal diseases have been determined, there does appear to be an association between localized juvenile periodontitis and the presence of A. Actinobacillus actinomycetemcomitans. B. Porphyromonas gingivalis. C. Actinomyces viscosus. D. Treponema denticola.

A. Actinobacillus actinomycetemcomitans.

Which of the following microorganisms is the MOST commonly associated with root surface caries? A. Actinomyces viscosus B. Streptococcus mutans C. Streptococcus salivarius D. Lactobacillus acidophilus

A. Actinomyces viscosus

Over a period of years, which of the following is the most likely reason for the breakage of a maxillary denture along the midline? A. Alveolar resorption B. Porosity of the denture base material C. Over-relief of the incisive papilla D. Inadequate extension of the posterior palatal seal

A. Alveolar resorption

Which of the following compounds is used as an antiviral agent? A. Amantadine (Symmetrel) B. Novobiocin C. Miconazole (Monistat) D. Amphotericin B

A. Amantadine (Symmetrel)

Which of the following compounds is used as an antiviral agent? A. Amantadine (Symmetrel®) B. Novobiocin C. Miconazole (Monistat®) D. Amphotericin B

A. Amantadine (Symmetrel®)

The dentist, who planned to sedate a patient intravenously, first injects a test dose into the patient's arm. Immediately, upon receiving the injection, the patient experiences a severe, burning pain at the site of insertion; it radiates distally. Simultaneously, her arm becomes blotchy with several blanched areas. Although her pulse in that arm is regular, it is weaker than her unaffected side. Which of the following might account for these findings? A. An injection into an artery B. An injection into the radial nerve C. An injection into the muscle compartment D. An impending anaphylactic reaction

A. An injection into an artery

In which of the following do adenomatoid odontogenic tumors MOST often occur? A. Anterior maxilla B. Posterior maxilla C. Anterior mandible D. Posterior mandible

A. Anterior maxilla

Hypertrophy of the mandibular condyle in an adult can result in each of the following EXCEPT one. Which one is this EXCEPTION? A. Anterior open bite B. Anterior crossbite C. Ipsilateral posterior open bite D. Unilateral Class III malocclusion E. Asymmetric facial appearance

A. Anterior open bite

Contraction of the lateral pterygoid muscle moves the articular disk in which of the following directions? A. Anteriorly and medially B. Posteriorly and medially C. Anteriorly and laterally D. Posteriorly and laterally E. Anteriorly, laterally, and inferiorly

A. Anteriorly and medially

A dentist inadvertently sealed a small carious lesion in the occlusal surface of a maxillary first molar. This would most likely result in A. Arrested caries. B. Extension of caries. C. Discoloration of the tooth. D. Increased microleakage.

A. Arrested caries.

Bradycardia is MOST commonly treated with which of the following drugs? A. Atropine B. Epinephrine C. A diuretic D. A potent vasodilator

A. Atropine

Which of the following injuries to the teeth most often results in pulpal necrosis? A. Avulsion B. Concussion C. Intrusive luxation D. Extrusive luxation E. Mid-root horizontal root fracture

A. Avulsion

Which type of glass filler is the most radiopaque? A. Barium B. Quartz C. Silicate D. Fumed silica

A. Barium

Which of the following drugs is excreted primarily by renal tubular secretion? A. Benzylpenicillin B. Streptomycin C. Tetracycline D. Bacitracin E. Polymyxin

A. Benzylpenicillin

Child abuse/neglect most commonly involves children in which age group? A. Birth to age three years B. Four to six years C. Seven to nine years D. All age groups are equally affected.

A. Birth to age three years

Damage to which of the following predisposes a patient to the development of osteoradionecrosis? A. Blood vessels B. Nerves C. Salivary glands D. Muscle

A. Blood vessels

Duration of treatment regimens affect patient compliance. The severity of disease perceived by the patient affects patient compliance. A. Both statements are TRUE. B. Both statements are FALSE. C. The first statement is TRUE, the second is FALSE. D. The first statement is FALSE, the second is TRUE.

A. Both statements are TRUE.

One objective of root-canal obturation is to develop a fluid tight seal. Another objective is to create a favorable biologic environment for the process of tissue healing. A. Both statements are TRUE. B. Both statements are FALSE. C. The first statement is TRUE, the second is FALSE. D. The first statement is FALSE, the second is TRUE.

A. Both statements are TRUE.

The Occupational Safety and Health Administration (OSHA) is concerned with regulated waste within the office. The Environmental Protection Agency (EPA) regulates the transportation of waste from the office. A. Both statements are TRUE. B. Both statements are FALSE. C. The first statement is TRUE, the second is FALSE. D. The first statement is FALSE, the second is TRUE.

A. Both statements are TRUE.

Before undergoing any periodontal surgery, a patient should be controlling the accumulation of plaque; BECAUSE, in the plaque-infected dentition after surgery, the incidence of disease recurrence will be greater. A. Both the statement and the reason are correct and related. B. Both the statement and the reason are correct but NOT related. C. The statement is correct, but the reason is NOT. D. The statement is NOT correct, but the reason is accurate. E. NEITHER the statement NOR the reason is correct.

A. Both the statement and the reason are correct and related.

In determining the caries rate for older adults, an epidemiologist usually uses the DIVIFT index, BECAUSE it can be difficult to ascertain why teeth are missing. A. Both the statement and the reason are correct and related. B. Both the statement and the reason are correct but NOT related. C. The statement is correct, but the reason is NOT. D. The statement is NOT correct, but the reason is accurate. E. NEITHER the statement NOR the reason is correct.

A. Both the statement and the reason are correct and related.

In treatment planning, the dentist should consider a patient's ethnic identity and heritage BECAUSE the patient's cultural background influences the formation of his health-related beliefs and attitudes. A. Both the statement and the reason are correct and related. B. Both the statement and the reason are correct but NOT related. C. The statement is correct, but the reason is NOT. D. The statement is NOT correct, but the reason is accurate. E. NEITHER the statement NOR the reason is correct.

A. Both the statement and the reason are correct and related.

The light source affects the perception of color, BECAUSE the light source must contain the wavelength of the color to be matched in order to see that color. A. Both the statement and the reason are correct and related. B. Both the statement and the reason are correct but NOT related. C. The statement is correct, but the reason is NOT. D. The statement is NOT correct, but the reason is accurate. E. NEITHER the statement NOR the reason is correct.

A. Both the statement and the reason are correct and related.

The long-term prognosis of a single-abutment cantilever fixed partial denture may be compromised, because forces are best tolerated by the periodontal supporting structures when directed in the long axes of the teeth. A. Both the statement and the reason are correct and related. B. Both the statement and the reason are correct but NOT related. C. The statement is correct, but the reason is NOT. D. The statement is NOT correct, but the reason is correct. E. NEITHER the statement NOR the reason is correct.

A. Both the statement and the reason are correct and related.

How should the margins of a dental amalgam restoration be trimmed? A. By carving along the margins with a sharp instrument that rests on the tooth surface B. By carving from the restoration to the tooth with a sharp instrument C. By carving from the tooth to the restoration with a sharp instrument D. By burnishing from the tooth to the restoration until the amalgam is trimmed to the margin

A. By carving along the margins with a sharp instrument that rests on the tooth surface

Which of the following systems is thought to malfunction in the hereditary form of angioneurotic edema? A. C-1 esterase B. C-1q inhibitor C. CH50 consumption D. Serine phosphatase E. Complement synthetase

A. C-1 esterase

A patient presents with numerous carious lesions. To restore these lesions, the dentist will place composite resins and use a visible-light curing unit. The patient's history of which of the following will necessitate that the dentist take extra precautions when using this unit? A. Cataract removal B. Myopia C. Glaucoma

A. Cataract removal

Which condition is characterized by lesions in the central nervous system that manifests itself as various types of neuromuscular dysfunction? A. Cerebral palsy B. Down syndrome C. A learning disability D. Hyperkinesis (attention deficit disorder)

A. Cerebral palsy

Which of the following conditions can cause delayed eruption of permanent teeth? A. Cherubism B. Hyperthyroidism C. Hyperparathyroidism D. Paget's disease of bone

A. Cherubism

On a prepayment basis, dental patients receive care at specified facilities from a limited number of dentists. This practice plan is classified as which of the following? A. Closed panel B. Open panel C. Group practice D. Solo practice

A. Closed panel

Of the following local anesthetics, which has intrinsic vasoconstrictive actions? A. Cocaine B. Procaine C. Xylocaine D. Bupivacaine

A. Cocaine

Which of the following diagnoses represents an injury that causes the tooth to loosen but is not displaced? A. Concussion B. Luxation C. Subluxation D. Extrusion E. Intrusion

A. Concussion

Where is the principal site of vertical growth of the mandible? A. Condylar head B. Sigmoid notch C. Coronoid process D. Alveolar process

A. Condylar head

Each of the following is an indication for a posterior maxillary osteotomy EXCEPT one. Which one is this EXCEPTION? A. Correction of mandibular prognathism B. Intrusion of the tuberosity, preparatory to denture construction C. Closure of the anterior open bite in some cases of apertognathia D. Skeletal arch width discrepancies that require lateral positioning of E. posterior segments

A. Correction of mandibular prognathism

Aspirin is CONTRAINDICATED with which of the following drugs? A. Coumarin (Coumadin®) B. Triazolam (Halcion®) C. Barbiturates (Phenobarbital®) D. Pentobarbital (Nembutal®) E. Methylprednisolone (Medrol®)

A. Coumarin (Coumadin®)

Which of the following types of chemical bonding is the least likely to be involved in a drug-receptor interaction? A. Covalent bonding B. Hydrogen bonding C. Dipole-dipole bonding D. Electrostatic bonding E. van der Waal's forces

A. Covalent bonding

At birth, which of the following structures is nearest the size it will eventually attain in adulthood? A. Cranium B. Mandible C. Middle face D. Nasal capsule

A. Cranium

Which of the following drugs is an "irreversible" cholinesterase inhibitor? A. DFP (Isoofluorophate®) B. Pilocarpine (pilocar) C. Neostigmine (Prostigmine®) D. Physostigmine (Antilirum®)

A. DFP (Isoofluorophate®)

Which of the following is the most appropriate initial treatment for a patient with HIV-associated necrotizing ulcerative gingivo-periodontitis? A. Debridement and antimicrobial rinses B. Definitive root planing and curettage C. Administration of antibiotics D. Gingivectomy and gingivoplasty

A. Debridement and antimicrobial rinses

Which of the following is most often associated with sensitivity of the skin to sun's rays? A. Demeclocycline B. Penicillin V C. Erythromycin D. Lincomycin E. Nystatin

A. Demeclocycline

Which of the following is seen most frequently among chronic temporomandibular-joint disorder patients? A. Depression B. Psychosis C. Sociopathy D. Schizotypical behavior E. Passive-aggressive behavior

A. Depression

Which of the following is the most common functional psychiatric disorder in the elderly? A. Depression B. Anxiety C. Senility D. Sociopathic behavior E. Mania

A. Depression

Which of the following is the current drug-of-choice for status epilepticus? A. Diazepam (Valium®) B. Phenytoin (Dilantin®) C. Chlorpromazine (Thorazine®) D. Carbamazepine (Tegretol®) E. Chlordiazepoxide (Librium®)

A. Diazepam (Valium®)

Which of the following can be a clinical feature of acute herpetic gingivostomatitis? A. Discrete, spherical vesicles B. Temperature-sensitive teeth C. Solitary, localized, edematous gingival lesions D. Necrotic destruction of interdental gingival tissues

A. Discrete, spherical vesicles

Which of the following tetracyclines is most slowly excreted? A. Doxycycline B. Tetracycline C. Oxytetracycline D. Chlortetracycline

A. Doxycycline

Which of the following tetracyclines with a long biological half- life allows once-per-day dosage? A. Doxycycline B. Oxytetracycline C. Chlortetracycline D. Demethylchlortetracycline

A. Doxycycline

Which of the following erythromycins is both enteric coated and long acting? A. ERYC B. E.E.S. C. Erythrocin D. Ilosone

A. ERYC

A child has a history of generalized growth failure ("failure to thrive") during the first six months of his life. Which of the following dental sequela is likely in this child? A. Enamel hypoplasia B. Retrusive midface C. Retrusive mandible D. Small permanent teeth E. Dentinogenesis imperfecta

A. Enamel hypoplasia

Blade and root form implants are examples of which of the following types of implants? A. Endosseous B. Transosteal C. Subperiosteal D. Ceramic

A. Endosseous

Which dietary supplement has been used by the public for weight loss and is considered very dangerous A. Ephedra B. Ginseng C. Garlic D. Gingko E. Echinacea

A. Ephedra

Gastric regurgitation causes which of the following? A. Erosion B. Abrasion C. Attrition D. Hypercementosis

A. Erosion

What happens to the permanent molar occlusion in the presence of a flush (straight) terminal plane and mandibular primate spaces? A. Erupts end-to-end; early mesial shift into Class I occlusion B. Erupts end-to-end; late mesial shift into Class I occlusion C. Erupts with Class II tendency D. Erupts with Class III tendency

A. Erupts end-to-end; early mesial shift into Class I occlusion

On a new patient's initial appointment, the dentist should do which of the following first? A. Establish rapport B. Inform C. Gather information D. Evaluate

A. Establish rapport

The Health Belief Model attempts to explain patient recommendations by A. Examining patients' perceptions of disease-seriousness and of the treatment effectiveness. B. Focusing on the doctor-patient relationship. C. Emphasizing the barriers that are external to behavior D. Change, for example, payment mechanisms. E. Focusing on patients' skills at carrying out recommendations.

A. Examining patients' perceptions of disease-seriousness and of the treatment effectiveness.

A patient presents with facial lacerations that have irregular or devitalized wound edges. In excising the wound elliptically, a practitioner takes into account each of the following EXCEPT one. Which is the EXCEPTION? A. Excision is perpendicular to the natural skin tension lines. B. Length to width ratio of the excision is at least 3 to 1 C. Excision is done as conservatively as possible. D. The wound is undermined prior to closure.

A. Excision is perpendicular to the natural skin tension lines.

Which of the following is an outstanding advantage of phenytoin over phenobarbital in the treatment of grand mal epilepsy? A. Exerts less sedation for a given degree of motor cortex depression B. Is effective in a greater percentage of cases C. Needs to be given much less frequently D. Does not cause gastric distress E. Has a longer duration of action

A. Exerts less sedation for a given degree of motor cortex depression

To determine the exact form of a bony defect, it will be necessary for the dentist to do which of the following? A. Expose the area surgically B. Examine in detail the gingival architecture C. Take two radiographs using different angulations D. Measure pocket depths from the cementoenamel junction E. Relate the septal bone height to the cementoenamel junction

A. Expose the area surgically

A dentist is preparing Tooth #30 for an occlusal amalgam restoration. Once the ideal outline form and depth have been established, the dentist notes that caries remains on the facial, pulpal, and lingual walls of the preparation. The next step in treatment is to A. Extend the outline form. B. Remove the caries with a spoon excavator. C. Remove the caries with a larger round bur.

A. Extend the outline form.

A 6-year-old patient has a dark brown discoloration of his maxillary central incisor. The discoloration started following trauma accompanied with a facial sinus tract. What is the treatment-of- choice? A. Extraction B. Pulpectomy C. Pulpotomy D. Direct pulp treatment

A. Extraction

During the delivery of dental treatment, which of the following is (are) the MOST likely to reveal patient discomfort? A. Eyes/eyebrow area B. Legs C. Voice D. Fingers/hands

A. Eyes/eyebrow area

In the preauricular approach to the TMJ, which of the following anatomic structures, if damaged, presents the potential for greatest morbidity? A. Facial nerve B. Parotid gland C. Auriculotemporal nerve D. Superficial temporal vein E. Facial artery

A. Facial nerve

Based on the Jones criteria, each of the following is a major manifestation of rheumatic fever EXCEPT one. Which one is this EXCEPTION? A. Fever B. Chorea C. Carditis D. Polyarthritis E. Erythema marginatum

A. Fever

Which of the following is the most consistent finding in systemic infections? A. Fever B. Tachypnea C. Lymphadenopathy D. Abscess formation E. Cellulitis formation

A. Fever

A dentist will reline an extension-base removable partial denture. Which of the following should this dentist use to establish proper placement at the impression stage? A. Finger pressure to the rests and indirect retainer B. Finger pressure to the base area only C. Simulated masticatory movements D. Gentle biting force

A. Finger pressure to the rests and indirect retainer

Which of the following drugs is administered orally to treat vaginal candidiasis? A. Fluconazole (Diflucan®) B. Griseofulvin (Grifulvin®) C. Clotrimazole (Mycelex® Troche) D. Miconazole (Monistat®) E. Nystatin (Mycostatin®)

A. Fluconazole (Diflucan®)

Streptococcus mutans is considered to be a principal etiologic agent of caries because it produces organic acids and it A. Forms a gelatinous matrix. B. Metabolizes substrate from saliva. C. Derives energy from enamel constituents. E. Lives symbiotically with Lactobacillus acidophilus.

A. Forms a gelatinous matrix.

Addition cured silicones (polyvinyl siloxanes) are often the material of choice for impressions for fixed restorations. Each of the following is true about these materials EXCEPT one. Which one is this EXCEPTION? A. Give off ethyl alcohol during their setting reaction B. Can be poured more than once and still remain accurate C. Can be poured after 24 hours and still remain accurate D. Rebound from undercuts without permanent deformation E. May release hydrogen gas during setting

A. Give off ethyl alcohol during their setting reaction

Which areas should be avoided for an IM injection before a child can walk? A. Gluteal muscles B. Ventrogluteal C. Vastus lateralis D. Deltoid area

A. Gluteal muscles

Papillary hyperplasia is MOST frequently found in which of the following sites? A. Hard palate B. Soft palate C. Anterior gingival tissue D. Posterior gingival tissue

A. Hard palate

Which of the following best explains how high-copper amalgam restorations differ from conventional amalgam restorations? The high-copper restorations A. Have little or no tin-mercury phase. B. Require less mercury, so there is more matrix formed. C. Corrode at an accelerated rate due to increased copper content. D. Are unaffected by moisture contamination in the presence of zinc.

A. Have little or no tin-mercury phase.

As an x-ray tube operates, electrons carry energy from the cathode to the anode. Into which of the following is most of this energy converted in the target? A. Heat B. X rays C. Magnetism D. Electricity E. Visible light

A. Heat

Which of the following tissues is most sensitive to radiation? A. Hemopoietic B. Endothelial C. Muscular D. Neural E. Cartilagenous

A. Hemopoietic

Fluoride therapy and occlusal sealants modify which of the following four factors the most? A. Host B. Time C. Substrate D. Microflora

A. Host

Which of the following is related to an enzyme deficiency and involves periodontal destruction around primary teeth? A. Hypophosphatasia B. Cyclic neutropenia C. Juvenile periodontitis D. Papillon-Lefevre syndrome

A. Hypophosphatasia

Cephalosporins are definitely CONTRAINDICATED for penicillin-allergic patients who exhibit A. Immediate-type reactions. B. Nausea and vomiting with erythromycin. C. Any type of reaction to the penicillins.

A. Immediate-type reactions.

A 5-year-old child has a posterior unilateral crossbite that is accompanied by a functional shift of the mandible. When should this crossbite be corrected? A. Immediately, without waiting for the eruption of permanent first molars B. When all the primary teeth have exfoliated C. After the permanent first molars have fully erupted D. When the child is approximately 9 years of age

A. Immediately, without waiting for the eruption of permanent first molars

A dentist primarily splints adjacent abutment teeth in a fixed partial denture in order to A. Improve the distribution of the occlusal load. B. Improve embrasure contours. C. Stabilize the abutment teeth. D. Improve mesiodistal spacing.

A. Improve the distribution of the occlusal load.

The rate at which new disease occurs is classified as which of the following? A. Incidence B. Prevalence C. Extent D. Attributable risk

A. Incidence

Which of the following landmarks will best guide the most esthetic location of the maxillary anterior artificial teeth on a complete denture? A. Incisive papilla B. Midline of the nose C. Median palatal suture D. Anterior labial frenum E. Anterior edentulous ridge

A. Incisive papilla

When trying-in a porcelain fused-to-metal crown, the dentist observes that the gingival-margin finish-line integrity is excellent, but that the occlusal surface is 1 mm too high. Which of the following is the most probable cause? A. Incorrectly related casts B. Proximal contacts being too tight C. A distortion of the metal during firing D. An expansion of the porcelain during firing

A. Incorrectly related casts

Each of the following side effects can occur as a result of systemic absorption of lidocaine EXCEPT one. Which one is this EXCEPTION? A. Increased gastric motility B. Tonic-clonic convulsions C. Decreased cardiac output D. Respiratory depression

A. Increased gastric motility

Cholinomimetic drugs can produce which of the following effects? A. Increased salivation B. Peripheral vasoconstriction C. Decreased gastric secretions D. Decreased gastrointestinal peristalsis E. Increased force of myocardial contraction

A. Increased salivation

Which of the following most affects curing of a light-activated composite resin? A. Increment thickness B. Shade of composite C. Length of exposure

A. Increment thickness

A conservative Class II preparation for dental amalgam should have which of the following characteristics? A. Independent retention and resistance form for both the proximal and occlusal portions B. Proximal retention and resistance form that depends upon a well-defined occlusal dovetail C. As wide on the occlusal as one-third the intercuspal distance D. Preparation depth twice the width of the isthmus

A. Independent retention and resistance form for both the proximal and occlusal portions

Which of the following is a nonsteroidal anti-inflammatory agent with a tendency to produce blood dyscrasias? A. Indomethacin (Indocin®) B. Ibuprofen (Motrin®) C. Ketorolac (Toradol®) D. Acetaminophen E. Aspirin

A. Indomethacin (Indocin®)

The dentist seats a full gold crown on a patient's mandibular right second molar. As the patient closes and as the teeth come into initial contact, the patient's jaw deflects to the right. Before treatment, the patient's occlusal relationship had been stable. To regain stability, the dentist will adjust the crown. On which incline of which cusp should the adjustments be made? A. Inner (lingual) incline, facial cusp B. Outer (facial) incline, facial cusp C. Inner (facial), incline, lingual cusp D. Outer (lingual) incline, lingual cusp

A. Inner (lingual) incline, facial cusp

Following cleansing of a tooth, new plaque growth accumulates principally on which of the following surfaces? A. Interproximal B. Occlusal C. Lingual D. Facial

A. Interproximal

Carcinoma in situ exhibits each of the following EXCEPT one. Which one is this EXCEPTION? A. Invasion B. Pleomorphism C. Hyperchromatism D. Abnormal mitoses E. Loss of polarity

A. Invasion

Which of the following describes the titration of diazepam to Verrill's sign for IV conscious sedation? A. It is recommended as an end-point. B. It is recommended only when supplemental 02 is used. C. It is usually not attainable with diazepam alone. D. It is not recommended since it can indicate a too-deeply sedated patient. E. It is not recommended since few patients are adequately sedated at that level.

A. It is recommended as an end-point.

Which of the following describes the laterotrusive movement (Bennett angle)? A. It is the angle that is formed by the non-working condyle and the sagittal plane during lateral movements. B. It is the angle that is formed by the condyle and the horizontal plane during protrusive movements. C. It is the difference in condylar inclination between protrusive and lateral movements. D. It is the difference between the condylar and incisal inclinations.

A. It is the angle that is formed by the non-working condyle and the sagittal plane during lateral movements.

A patient who has Parkinson's disease is being treated with levodopa. Which of the following characterizes this drug's central mechanism of action? A. It replenishes a deficiency of dopamine. B. It increases concentrations of norepinephrine. C. It stimulates specific L-dopa receptors. D. It acts through a direct serotonergic action.

A. It replenishes a deficiency of dopamine.

The penetrating quality of x-ray beams is influenced by which of the following? A. Kilovoltage B. Milliamperage C. Exposure time D. Focal-film distance E. Filament temperature

A. Kilovoltage

During an interview, a dentist asks a patient, "You are not afraid of dentistry, are you?" This is an example of which type of question? A. Leading B. Closed C. Open D. Funneling

A. Leading

What percentage of liquid is in an oxygen "E" cylinder when it is half full? A. Less than 1% B. .25 C. .5 E. .95

A. Less than 1%

A patient presents with a bilateral, grayish-white lesion of the buccal mucosa. This lesion disappears when the mucosa is stretched. Which of the following is the MOST likely condition? A. Leukoedema B. Leukoplakia C. Lichen planus D. White sponge nevus E. Pachyonychia congenita

A. Leukoedema

Which of the following mucocutaneous disorders is characterized by (1) degeneration of the basal cell layer, (2) "saw-tooth" rete pegs, and (3) a dense mononuclear inflammatory-cell infiltrate in the subepithelial connective tissue? A. Lichen planus B. Erythema multiforme C. Keratosis follicularis D. Dermatitis herpetiformis E. Benign mucous membrane pemphigoid

A. Lichen planus

Which pair of anesthetics is most likely to show cross-allergy? A. Lidocaine - mepivacaine B. Prilocaine - tetracaine C. Procaine - mepivacaine D. Procaine - lidocaine E. Lidocaine - benzocaine

A. Lidocaine - mepivacaine

Class I carious lesions are the LEAST likely to occur on which of the following? A. Lingual surfaces of mandibular incisors B. Lingual surfaces of maxillary incisors C. Lingual surfaces of maxillary molars D. Facial surfaces of mandibular molars

A. Lingual surfaces of mandibular incisors

The periosteal elevator is used to do each of the following EXCEPT one. Which one is the EXCEPTION? A. Luxate the teeth B. Retract the soft tissue flap C. Elevate the interdental papillae D. Provide protection to the soft tissue flap

A. Luxate the teeth

A dentist determined that it would be necessary to adjust a patient's denture teeth to correct the centric occlusion at the wax try-in appointment. Which of the following should this dentist do? A. Make a new centric relation record and remount B. Make a new face bow and centric record C. Make slight occlusal adjustments in the mouth D. Make adjustments only for Class II jaw relation patients

A. Make a new centric relation record and remount

The TMJ is innervated by the branches of each of the following nerves EXCEPT one. Which one is this EXCEPTION? A. Marginal mandibular B. Auriculotemporal C. Masseteric D. Mandibular division of the trigeminal E. Deep temporal

A. Marginal mandibular

Myositis usually involves which of the following muscles? A. Masseter B. Digastric C. Mylohyoid D. Lateral pterygoid

A. Masseter

The distofacial periphery of the mandibular impression should receive special attention. Which of the following anatomical structures might cause soreness if the denture is overextended? A. Masseter B. Buccinator C. Pterygomandibular raphe D. Internal pterygoid E. Lateral tendon of the temporal

A. Masseter

In which of the following teeth does the pulpal anatomy dictate a triangular-access cavity preparation? A. Maxillary central incisor B. Maxillary first premolar C. Mandibular second premolar D. Mandibular canine

A. Maxillary central incisor

Which of the following teeth are usually affected by nursing-bottle caries? A. Maxillary incisors B. Mandibular molars C. Mandibular incisors D. Maxillary molars

A. Maxillary incisors

Which of the following teeth are the least likely to have more than one canal? A. Maxillary lateral incisors B. Mandibular lateral incisors C. Mandibular first premolars D. Maxillary second premolars E. Maxillary second molars

A. Maxillary lateral incisors

A patient has a chronically tender, knife-edge mandibular residual ridge. In fabricating a complete denture for this patient, a dentist should consider which of the following? A. Maximal extension of the denture--to distribute forces of occlusion over a greater area B. Minimal extension of the denture--to limit tenderness to a smaller area C. Decreased occlusal vertical dimension--to decrease biting forces D. A broad occlusal table--to provide a firmer contact in eccentric jaw relations

A. Maximal extension of the denture--to distribute forces of occlusion over a greater area

Retention of fissure sealants is chiefly the result of which of the following? A. Mechanical microretention B. Adhesion of the sealant C. Chemical bonding of the sealant with the enamel D. Physical penetration of the sealant into the pits and fissures

A. Mechanical microretention

Which of the following represents an effective treatment for mottled enamel (chalky white color mixed with brown spots)? A. Microabrasion with HCl/pumice B. Vital tooth bleaching at home C. Walking bleach after root canal treatment D. In-office bleaching with 35% hydrogen peroxide

A. Microabrasion with HCl/pumice

The reduction of which of the following represents the MOST significant advantage of the acid-etch technique? A. Microleakage B. Pulpal irritation C. Setting shrinkage of the matrix D. Coefficient of thermal expansion

A. Microleakage

Although a dentist wishes to use the long-cone paralleling technique (40 cm source-object distance), the x-ray machine is equipped with only a short cone (20 cm PID). As a result, the dentist moves the machine away from the patient's face so the source-object distance is 40 cm and adjusts the exposure time appropriately. Compared to the degree of tissue covered by an actual long cone, this "extended" short cone exposes what amount of tissue? A. More B. The same C. Less

A. More

The supraspinal analgesic activity of morphine is mediated primarily through its influence upon which opioid receptor subtype? A. Mu B. Kappa C. Delta D. Sigma E. Epsilon

A. Mu

Each of the following developmental cysts appears on radiographs EXCEPT one. Which one is this EXCEPTION? A. Nasolabial B. Median palatal C. Globulomaxillary D. Nasopalatine

A. Nasolabial

Which of the following cysts is the LEAST likely to be visible radiographically? A. Nasolabial B. Nasopalatine C. Lateral periodontal D. Traumatic bone

A. Nasolabial

A permanent tooth with a mature apex has become intruded deeply into the bone. As a result of this trauma, which of the following pulpal changes is the most likely to occur? A. Necrosis B. Internal resorption C. Calcific metamorphosis D. Transient inflammation

A. Necrosis

The combination of a Schedule II narcotic with an antipsychotic drug produces which of the following? A. Neuroleptic analgesia B. Conscious sedation C. Dissociative anesthesia D. Psychotomimetic analgesia

A. Neuroleptic analgesia

Having had a complete radiographic survey made 12 months ago, an asymptomatic adult patient presents for a recall appointment. There is no clinical evidence of caries or periodontal disease. The United States Food and Drug Administration would recommend which of the following for this adult? A. No radiographs B. Bite-wing radiographs only C. Panoramic radiograph only D. Panoramic and bite-wing radiographs E. Full-mouth survey including bite-wing radiographs

A. No radiographs

Which drug group is the least likely to cause xerostomia? A. Opioids B. Antidepressants C. Antihistaminics D. Benzodiazepines E. Anticholinergics

A. Opioids

Which of the following is an example of an enteral route of administration? A. Oral B. Submucosal C. Inhalation D. Subcutaneous E. Intramuscular

A. Oral

Which of the following routes of administration. Exhibits the slowest rate of absorption? A. Oral B. Sublingual C. Subcutaneous D. Intramuscular

A. Oral

A 20-year-old patient presents with an upper maxillary central incisor previously treated for endodontic fractures at the level of the alveolar crest. Which of the following represents the first necessary step to restore and maintain the tooth's esthetics? A. Orthodontically extrude remaining tooth structure B. Extract remaining tooth structure C. Fabricate FPD over the remaining root D. Fabricate RPD over the remaining root

A. Orthodontically extrude remaining tooth structure

Spontaneous osteogenic sarcoma is associated with which of the following disease processes of bone? A. Paget's disease B. Histiocytosis-X C. Hyperparathyroidism D. Letterer-Siwe disease E. Osteogenesis imperfecta

A. Paget's disease

The axial wall of an occlusolingual amalgam preparation on a maxillary molar should be in dentin and A. Parallel to the dentinoenamel junction. B. Parallel to the long axis of the tooth. C. At an acute angle with the pulpal floor.

A. Parallel to the dentinoenamel junction.

A patient has painful lesions on her buccal mucosa. A biopsy reveals acantholysis and a suprabasilar vesicle. Which of the following represents the MOST likely diagnosis? A. Pemphigus B. Psoriasis C. Erythema multiforme D. Bullous lichen planus E. Systemic lupus erythematosus

A. Pemphigus

Which of the following drugs is most likely to cause convulsions when a physically dependent person is withdrawn? A. Pentobarbital (Nembutal®) B. Diazepam (Valium®) C. Hydroxyzine (Vistaril®) D. Chloral hydrate (Noctec®) E. Chlorzoxazone (Parafon Forte)

A. Pentobarbital (Nembutal®)

Which of the following periodontal diseases causes the most rapid destruction of alveolar bone? A. Periodontal abscess B. Chronic periodontitis C. Phenytoin-induced gingival enlargement D. Necrotizing ulcerative gingivitis

A. Periodontal abscess

A 35-year-old patient presents with a swelling that is facial to a mandibular premolar. He reports that, although he has no severe pain, he is experiencing a mild discomfort. His radiographs reveal that there is a round radiolucency in the middle third of the root. The involved teeth are vital; there are no periodontal pockets. Which of the following is the most likely diagnosis? A. Periodontal cyst B. Gingival abscess C. Osteogenic sarcoma D. Periodontal abscess

A. Periodontal cyst

Which of the following reactive lesions of the gingival tissue reveals bone formation microscopically? A. Peripheral ossifying fibroma B. Traumatic neuroma C. Pyogenic granuloma D. Irritation fibroma E. Osteoma

A. Peripheral ossifying fibroma

Grand mal seizure is best treated with which of the following drugs? A. Phenytoin (Dilantin®) B. Thioridazine (Mellaril®) C. Ethosuximide (Zarontin®) D. Trimethadione (Tridione®) E. Chlorpromazine (Thorazine®)

A. Phenytoin (Dilantin®)

During an emergency dental visit, a moderately retarded 4-year-old child resists physically and cries excessively. In this instance, the dentist should use which of the following? A. Physical restraints B. General anesthesia C. The hand-over-mouth exercise D. Voice control

A. Physical restraints

Which of the following is the primary etiology of gingivitis during puberty and pregnancy? A. Plaque B. Estradiol C. Calculus D. Estrogen E. Progesterone

A. Plaque

Which of the following is the MOST effective means of controlling postsurgical root sensitivity after a periodontal surgery? A. Plaque control B. Applications of formalin C. Applications of calcium hydroxide D. Use of a desensitizing dentifrice

A. Plaque control

Which of the following syringe materials is the most rigid and, therefore, the most difficult to remove from the mouth? A. Polyether B. Polysulfide C. Addition silicone D. Condensation silicone E. Reversible hydrocolloid

A. Polyether

Which of the following oral bacteria have been implicated as periodontal pathogens? A. Porphyromonas gingivalis, Eubacterium sp., and Campylobacter recta B. Streptococcus gordonii, Streptococcus mutans, and Gemella sp. C. Lactobacillus casei and Veillonella sp. D.. Streptococcus mitior, Streptococcus gordonii, and Bacteroides gracilis

A. Porphyromonas gingivalis, Eubacterium sp., and Campylobacter recta

Which of the following oral bacteria have been implicated as periodontal pathogens? A. Porphyromonas gingivalis, Eubacterium sp., and Campylobacter recta B. Streptococcus gordonii, Streptococcus mutans, and Giamella sp. C. Lactobacillus casei and Veillonella sp. D. Streptococcus mitior, Streptococcus gordonii, and Bacteroides gracilis

A. Porphyromonas gingivalis, Eubacterium sp., and Campylobacter recta

Which of the following properties increases when the intermediate chain of a local anesthetic drug is lengthened? A. Potency B. Allergenicity C. Solubility in water D. Rate of biotransformation

A. Potency

Barbiturates taken in combination with very high doses of ethanol exhibit an interaction best described as which of the following? A. Potentiation B. Protein bumping C. Negative synergism D. Competitive inhibition E. Enterohepatic circulation

A. Potentiation

A 5-year-old child has acute primary herpetic gingivostomatitis. To treat the condition, a dentist should consider each of the following EXCEPT one. Which one is this EXCEPTION? A. Prescribe erythromycin. B. Place the child on a bland diet. C. Encourage the child to drink plenty of fluids. D. Order bed rest and minimal contact with other children. E. Prescribe a topical anesthetic rinse for use before meals.

A. Prescribe erythromycin.

What would be the immediate concern of a dentist after removing the primary mandibular left second molar of a 3-year-old child? A. Preventing lip biting B. Preventing a dry socket C. Controlling postoperative pain D. Providing space maintenance E. Preventing postoperative swelling

A. Preventing lip biting

Oral cytology smears are MOST appropriately used for the diagnosis of which of the following? A. Pseudomembraneous candidiasis B. Lichen planus C. Squamous cell carcinoma D. Benign mucous membrane pemphigoid

A. Pseudomembraneous candidiasis

A needle tract infection following an inferior alveolar block injection would initially involve which space? A. Pterygomandibular B. Sublingual C. Submandibular D. Lateral pharyngeal

A. Pterygomandibular

Which of the following is characteristic of Verrill's sign? A. Ptosis of eyelid B. Nystagmus in lateral gaze C. Paresis due to CNS depression D. Ophthalmoplegia upon exposure to light

A. Ptosis of eyelid

Prolonged, unstimulated night pain suggests which of the following conditions of the pulp? A. Pulp necrosis B. Mild hyperemia C. Reversible pulpitis D. No specific condition

A. Pulp necrosis

A 4-year-old child has a history of frequent spontaneous pain in a primary mandibular second molar. This molar has a necrotic pulp. Which of the following represents the treatment of choice? A. Pulpectomy B. Pulpotomy C. Extraction D. Pulp cap

A. Pulpectomy

Which of the following characteristics most readily distinguishes amelogenesis imperfecta from dentinogenesis imperfecta? A. Radiographic appearance B. Hereditary background C. Presence of blue sclera D. Color of the teeth E. Associated hair loss

A. Radiographic appearance

To evaluate the effectiveness of a dental health program, researchers will use a pretest-posttest design. Which of the following will represent the preferred method for placing subjects into the experimental and control groups? A. Random assignment B. Interviewing C. Matching

A. Random assignment

Which of the following factors is of greatest importance in selecting a restorative material for a Class V cavity on the facial surface of a mandibular premolar? A. Reaction of gingival tissue B. Resistance to stress C. Ease of manipulation D. Esthetics

A. Reaction of gingival tissue

An examination of a complete denture patient reveals that the retromolar pad contacts the maxillary tuberosity at the occlusal vertical dimension. To remedy this situation, which of the following should be performed? A. Reduce the maxillary tuberosity by surgery. B. Cover the tuberosity with a metal base. C. Increase the occlusal vertical dimension. D. Reduce the retromolar pad by surgery. E. Omit coverage of the retromolar pad by the mandibular denture.

A. Reduce the maxillary tuberosity by surgery.

The pulpal floor is perforated during access preparation. Which of the following is the best course of action? A. Repair the perforation immediately; continue the root canal treatment B. Continue the root canal treatment; repair the perforation at a subsequent appointment only if associated pathosis develops C. Continue the root canal treatment; repair the perforation at a subsequent appointment D. Perform no further treatment at this time; repair the perforation and continue the root canal treatment at a subsequent appointment

A. Repair the perforation immediately; continue the root canal treatment

Two adjacent cavities involving proximal contact can be prepared and restored with composite resin at one appointment for each of the following reasons EXCEPT one. Which one is this EXCEPTION? A. Restoration of contact is enhanced. B. Access to adjacent cavities is simplified. C. Color matching is easier.

A. Restoration of contact is enhanced.

The relative position of the maxilla to the cranial base can be determined cephalometrically by the angle A. SNA. B. SNB. C. Sn-GoGn. D. SN-Pog. E ANB.

A. SNA.

To prevent physical injury most effectively when using pickling solutions, dentists should wear which of the following? A. Safety goggles B. Utility gloves C. A face mask

A. Safety goggles

A patient undergoes an extraction of a permanent mandibular first molar and elects NOT to have the tooth replaced. Over a period of time, the teeth adjacent to the extraction site are likely to move in which of the following directions? A. Second molar, mesially; second premolar, distally B. Second molar, no movement; second premolar, distally C. Second molar mesially; second premolar, no movement

A. Second molar, mesially; second premolar, distally

Opiates are CONTRAINDICATED for patients who have which of the following? A. Severe head injury B. Bronchial asthma C. Renal dysfunction D. Acute myocardial infarction

A. Severe head injury

Which of the following is the best method for evaluating centric occlusion on a newly placed onlay restoration? A. Shim stock B. Articulating paper C. Patient feedback D. Occlusal indicator wa

A. Shim stock

The long axis of a mandibular second molar, Tooth #18, is mesially tipped 30 degrees to the plane of occlusion. A 3-unit fixed partial denture from Tooth #18 to Tooth #20 is treatment planned for this 70-year-old patient. Which of the following complications during preparation of Tooth #18 would most likely affect the restoration's long-term prognosis? A. Short distal axial wall B. Irreversible pulpitis C. Latent sensitivity D. Pulp exposure

A. Short distal axial wall

The concentration of which of the following ions determines the binding affinity of agonists and antagonists to the opioid receptor? A. Sodium B. Calcium C. Chloride D. Potassium E. Phosphate

A. Sodium

Which of the following is measured by the modulus of elasticity? A. Stiffness or rigidity B. Ultimate strength C. Yield strength D. Ductility or malleability

A. Stiffness or rigidity

Which of the following is an early colonizer of supragingival plaque? A. Streptococcus sanguis B. Streptococcus salivarius C. Porphyromonas gingivalis D. Capnocytophaga gingivalis

A. Streptococcus sanguis

Which of the following should be included to ensure the best prognosis in the management of localized juvenile periodontitis? A. Systemic antibiotic therapy B. Chlorhexidine mouthrinses C. High doses of vitamin C D. Free gingival grafts E. Peroxide rinses

A. Systemic antibiotic therapy

To enhance the patient's comfort in the dental setting, the dentist should do which of the following? A. Tell the patient what to expect during the appointment B. Limit eye contact to avoid being unduly intimate C. Provide an immediate evaluation of the patient's oral health D. Demonstrate control through a directive interviewing style

A. Tell the patient what to expect during the appointment

Drug interaction resulting in serious adverse cardiovascular events, including death, might occur between erythromycin and which of the following antihistamine drugs? A. Terfenadine (Seldane®) B. Promethazine (Phenergan®) C. Hydroxyzine (Atarax®) D. Diphenhydramine (Benadryl®) E. Chlorpheniramine (Chlor-Trimeton®)

A. Terfenadine (Seldane®)

Drug interaction resulting in serious adverse cardiovascular events, including death, might occur between erythromycin and which of the following antihistamine drugs? A. Terfenadine (Seldane®) B. Promethazine (Phenergan®) C. Hydroxyzine (Atarax®) D. Diphenhydramine (Benadryl®) E. Chlorpheniramine (Chlortrimenton®)

A. Terfenadine (Seldane®)

The four mandibular incisors of a patient were traumatized in an automobile accident three years ago. Current radiographs show bone rarefaction in the region of the apices of the incisors. Which of the following courses of treatment are indicated? A. Test the pulp vitality of the incisors and perform root canal therapy only in those teeth that do not respond to pulp testing. B. Perform root canal therapy on the incisors and curet the entire area of bone involvement. C. Extract the four incisors and construct a fixed partial denture. D. Postpone all treatment and recheck the status of the condition periodically.

A. Test the pulp vitality of the incisors and perform root canal therapy only in those teeth that do not respond to pulp testing.

Which of the following adverse conditions may arise if the occlusal vertical dimension is increased? A. The closing muscles may become strained. B. The opening muscles may become strained. C. The closing muscles may become too relaxed. D. Soreness may occur at the corners of the mouth.

A. The closing muscles may become strained.

A practitioner has burnished a margin. As a result of this procedure, the dental casting gold has become stronger, but less ductile. Which of the following explains why? A. The gold has work-hardened. B. The grains have elongated. C. The stiffness has increased. D. The surface flaws have been removed.

A. The gold has work-hardened.

The success of a free gingival graft procedure depends upon which of the following? A. The graft being immobilized at the recipient site B. The donor tissue being as thick as possible and containing periosteum C. A thick blood clot remaining between the recipient and donor tissue D. A periosteal fenestration being present

A. The graft being immobilized at the recipient site

Each of the following parameters has an effect on the distribution of a drug EXCEPT one. Which one is this EXCEPTION? A. The intrinsic activity of the drug B. The binding of a drug to plasma proteins C. The blood flow to various organs and tissues D. The relative solubility of the drug in the tissues

A. The intrinsic activity of the drug

The day after receiving an inferior alveolar nerve block, a patient experiences limited ability to open his mouth. Which of the following structures was most probably injured? A. The medial pterygoid muscle B. The stylomandibular ligament C. The deep fibers of the masseter muscle D. The posterior belly of the digastric muscle E. The inferior head of the lateral pterygoid muscle

A. The medial pterygoid muscle

The crescent-like radiolucencies at the crevices of the maxillary central incisors can be seen in the radiograph. These radiolucencies are MOST likely to be which of the following? A. The nares B. Fracture lines C. The incisive fossa D. Finger-nail crimp marks

A. The nares

The proximal portion of a Class II cavity preparation in a primary molar extends rather deep gingivally. A satisfactory gingival seat may NOT be obtained because of which of the following? A. The primary teeth have a marked cervical constriction. B. The proximal contact of primary molars is broad and flat. C. The facial and lingual surfaces of primary molars converge occlusally. D. The enamel rods in the gingival third of the primary first molars extend occlusally.

A. The primary teeth have a marked cervical constriction.

Which of the following penicillins would be used to treat a Pseudomonas infection? A. Ticarcillin (Ticar ) B. Ampicillin (Omnipen ) C. Cloxacillin (Tegopen ) D. Phenoxymethyl penicillin (Pen-Vee K)

A. Ticarcillin (Ticar )

Each of the following is a reason for beveling a preparation for restoration with composite resins EXCEPT one. Which one is this EXCEPTION? A. To expose more inorganic tooth structure B. To increase the surface area of enamel for etching C. To expose the ends rather than the sides of enamel rods D. To enhance the enamo-resin marginal seal

A. To expose more inorganic tooth structure

Each of the following is a reason for restoring an endodontically treated posterior tooth with a dowel or post EXCEPT one. Which one is this EXCEPTION? A. To strengthen the root B. To enhance retention of the core C. To enhance the lateral force resistance

A. To strengthen the root

The inferior concha is composed of all or part of which of the following bones? A. Maxilla B. Ethmoid C. Sphenoid D. Palatine E. Turbinate

A. Turbinate

A 5-year-old boy presents for his first dental appointment. His primary dentition is complete; all teeth are in proximal contact. There is no clinical evidence of caries or other pathology. For this child, the dentist should recommend which of the following radiographs? A. Two bite-wing B. Two bite-wing and one panoramic C. Two bite-wing, four periapical, and two occlusal

A. Two bite-wing

Which of the following is a principle of alveoloplasty? A. Undercuts that interfere with the seating of the denture should be removed. B. The remaining ridge should be as broad as possible, even if some undercuts exist. C. Sharp bony projections need not be smooth as they will round off in a few days during the healing process.

A. Undercuts that interfere with the seating of the denture should be removed.

An employee wishes to clean a dental-office operatory. To receive the greatest protection during these procedures, this individual should wear which of the following types of gloves? A. Utility B. Vinyl C. Fiberglass D. Latex

A. Utility

Which radiographic view best demonstrates the lesions of the maxillary sinus? A. Waters' B. Towne's C. Submental vertex D. Panore

A. Waters'

When is the appropriate time to take the first bite-wing radiographs of a clinically caries-free child? A. When spaces between the posterior teeth have closed B. When the entire primary dentition is completely erupted C. When the permanent first molars have erupted D. At the first dental examination

A. When spaces between the posterior teeth have closed

When is the appropriate time to correct a crossbite of a permanent maxillary central incisor? A. When the tooth is erupting B. After the tooth is fully erupted C. After all anterior teeth are erupted D. After the opposite central incisor has erupted

A. When the tooth is erupting

Which of the following statements about maxillofacial bone grafts is INCORRECT? A. Xenografts are frequently used in mandibular reconstruction. B. The most common source of autologous bone is the iliac crest. C. Allogenic banked bone is often used as a carrier, supplemented with an autogenous graft. D. An example of isogeneic bone grafting is transplanting bone marrow in the treatment of such diseases as leukemia.

A. Xenografts are frequently used in mandibular reconstruction.

Which of the following represents the basic constituent of most root canal sealers? A. Zinc oxide B. Zinc stearate C. Polyvinyl resin D. Polycarboxylate E. Zinc oxyphosphate

A. Zinc oxide

Waiting to allow intruded permanent central incisors to reerupt in an 8-year-old child will likely lead to A. a healing response and reeruption. B. calcific metamorphosis. C. internal resorption. D. tooth discoloration. E. ankylosis.

A. a healing response and reeruption.

Treatment with sulfonamides is less likely to be accompanied by crystalluria if A. a mixture of sulfonamides is used. B. cortisone is administered concurrently. C. probenecid is administered concurrently. D. the urine is acidified with ammonium chloride. E. para-aminosalicylic acid is administered concurrently.

A. a mixture of sulfonamides is used.

The success of a pulpotomy for a primary molar depends primarily upon A. a vital root pulp. B. the patient's age. C. the amount of root resorption. D. the absence of internal root resorption.

A. a vital root pulp.

If a patient suddenly becomes unconscious during treatment, the practitioner should immediately assess the patient's A. airway and pulse. B. airway and blood pressure. C. pulse and blood pressure. D. aolor and pupils.

A. airway and pulse.

A freeze-dried decalcified bone graft taken from a human donor and placed in a periodontal defect in another patient is termed an A. allograft. B. autograft. C. isograft. D. alloplastic graft.

A. allograft.

Anterior open bites can be classified as a form of A. apertognathism. B. micrognathism. C. retrognathism. D. prognathism. E. microgenia.

A. apertognathism.

Gingivectomy is NOT likely to be the treatment-of -choice for the elimination of pockets when the base of the pocket is located at the mucogingival junction or A. apical to the alveolar crest. B. below the free gingival groove. C. coronal to the cementoenamel junction. D. apical to the cervical convexity of the tooth crown.

A. apical to the alveolar crest.

In addition to the treatment of epilepsy, phenytoin may be indicated for the treatment of A. arrhythmia. B. narcolepsy. C. hypertension. D. schizophrenia. E. panic attacks.

A. arrhythmia.

In a clinical pharmacologic comparison of atropine and its quatemary ammonium derivative methylatropine nitrate, one might expect to observe that A. atropine is more potent than methylatropine nitrate after oral administration. B. methylatropine nitrate is more potent and effective than atropine in the treatment of Parkinson's disease. C. the ratio of the effective oral dose to the effective parenteral dose is higher for atropine than for methylatropine nitrate. D. in doses equieffective in affecting the salivary glands, drowsiness and clouding of consciousness is a more prominent effect with methylatropine nitrate than with atropine.

A. atropine is more potent than methylatropine nitrate after oral administration.

The ability of a patient to make choices among treatment options represents the concept of A. autonomy. B. competence. C. beneficence. D. paternalism.

A. autonomy.

The complete separation of a tooth from its alveolus is A. avulsion. B. ankylosis. C. subluxation. D. extrusive luxation.

A. avulsion.

Metastatic carcinoma to the jaws is LEAST likely to arise from the A. brain. B. breast. C. kidney. D. prostate. E. lung.

A. brain.

Orange stain is used to A. change the hue of porcelain color. B. increase the interproximal translucency. C. increase the value of porcelain color. D. decrease the chroma of porcelain color.

A. change the hue of porcelain color.

The only local anesthetic that increases the pressor activity of both epinephrine and norepinephrine is A. cocaine. B. procaine. C. dibucaine. D. lidocaine. E. mepivacaine.

A. cocaine.

The ability to "maintain a patient airway independently" is a requirement of A. conscious sedation B. deep sedation C. general anesthesia

A. conscious sedation

Allowing the ultrasonic-scaler tip to remain on the tooth surface too long will A. damage the tooth surface. B. dull the tip of the instrument. C. burnish the calculus onto the D. tooth surface. E. cause irreversible pulpal damage.

A. damage the tooth surface.

The size of the x-ray tube focal spot influences radiographic A. definition. B. contrast. C. density. D. distortion.

A. definition.

Currently, bonding of composite restorative materials to dentin depends on A. difunctional coupling agents. B. dehydration of the dentin. C. etching with dilute phosphoric acid. D. covalent bonding.

A. difunctional coupling agents.

For a patient with multiple sclerosis A. epinephrine is contraindicated in local anesthetic. B. the amount of anesthetic needed for a given procedure is less than for a normal patient. C. the amount of anesthetic needed for a given procedure is more than for a normal patient. D. a single cartridge of anesthetic will most likely not last as long as it would for a normal patient.

A. epinephrine is contraindicated in local anesthetic.

An x-radiation dose of 4 Gy administered locally to a patient's arm is most likely to cause A. erythema. B. bone marrow death. C. carcinoma of the skin. D. acute radiation syndrome.

A. erythema.

A diagnostic test failed to identify five cases of true disease. This type of failure is known as a A. false negative. B. false positive. C. positive predictive value. D. negative predictive value.

A. false negative.

In therapeutic doses, digitalis acts primarily on the cardiac muscle. It does so by increasing the A. force of contraction. B. refractory period of the atrial muscle. C. refractory period of the ventricular muscle. D. rate of conduction of impulses to the muscle.

A. force of contraction.

The major effect of a drug is produced by the amount of the drug that is A. free in plasma. B. excreted by the kidney. C. detoxified in the liver. D. bound to plasma protein.

A. free in plasma.

Crevicular epithelium is one example of non-keratinized gingival tissue. Another example is (the) A. gingival col. B. marginal gingiva. C. attached gingiva. D. outer gingival epithelium.

A. gingival col.

A dentist will remove a mandibular lingual torus from a patient's premolar region. This dentist should use an envelope flap design that A. has no vertical components. B. has an anterior vertical component only. C. has anterior and posterior vertical components. D. is incised at the junction of attached gingival tissue and free mucosa.

A. has no vertical components.

Sedation of a patient with nitrous oxide should always end with administration of 100% oxygen to prevent A. hypoxia. B. hypercarbia. C. tachycardia. D. hypotension.

A. hypoxia.

A practitioner takes a radiograph of a patient's mandibular molar at 0 degrees vertical angulation. It shows the superior border of the mandibular canal in apparent contact with the apices. This practitioner takes a second radiograph at -2 degrees vertical angulation; it shows the mandibular canal separated from the apices by several millimeters. These radiographs confirm that the anatomic location of the mandibular canal, in relation to the root apices of the molar, is A. inferior and lingual. B. inferior and facial. C. superior and lingual. D. superior and facial. E. in juxtaposition.

A. inferior and lingual.

The most potent antitubercular drug is A. isoniazid. B. vancomycin. C. trimethoprim. D. sulfamethoxazole.

A. isoniazid.

A dentist should place the terminus of a retentive clasp arm at (in) the A. junction of the gingival and middle one third of the clinical crown. B. junction of the middle and occlusal one third of the clinical crown. C. level of the free gingival margin. D. occlusal one third of the clinical crown.

A. junction of the gingival and middle one third of the clinical crown.

Which of the following best explains why drugs that are highly ionized tend to be more rapidly excreted than those that are less ionized? The highly ionized are A. less lipid soluble. B. less water soluble. C. more rapidly metabolized. D. more extensively bound to tissue.

A. less lipid soluble.

A broad understanding of the development of human behavior requires knowledge of the basic concepts of A. maturation and learning. B. dependence and independence. C. gender and race. D. generalization and facilitation

A. maturation and learning.

Which of these opioid analgesics is associated with a serious life threatening drug interaction when administered with an MAO inhibitor? A. meperidine B. morphine C. fentanyl D. propoxyphene E. codeine

A. meperidine

The prognosis for internal bleaching is favorable when the discoloration is caused by A. necrotic pulp tissue. B. amalgam restorations. C. precipitation of metallic salts. D. silver-containing root canal sealers.

A. necrotic pulp tissue.

Growth at the mandibular condyle during puberty usually results in increases in A. posterior facial height. B. maxillary arch length. C. mandibular intermolar width. D. upper anterior facial height.

A. posterior facial height.

In vestibuloplasty, a skin graft functions to A. prevent reattachment of muscle to periosteum. B. promote osteogenesis to establish a bony barrier to loss of sulcus depth. C. inhibit fibroblasts in the underlying tissue. D. provide an adequate amount of nonkeratinized tissue.

A. prevent reattachment of muscle to periosteum.

In preparing a Class I cavity for dental amalgam, the dentist will diverge the mesial and distal walls toward the occlusal surface. This divergence serves to A. prevent undermining of the marginal ridges. B. provide convenience form. C. resist the forces of mastication. D. extend the preparation into areas more readily cleansed.

A. prevent undermining of the marginal ridges.

In the treatment of localized alveolar osteitis, the primary purpose of the dressing is to A. provide a vehicle for obtudent medication. B. promote the growth of the adjacent epithelium. C. prevent saliva from contaminating the alveolus. D. stimulate the formation of a blood clot.

A. provide a vehicle for obtudent medication.

A 26-year-old female patient states that for several years, small, red to white oral lesions have occurred irregularly. They are painful but heal in 7 to 10 days. Clinical examination reveals these lesions are located on alveolar nonkeratinized mucosa. The most likely diagnosis is A. recurrent aphthous stomatitis. B. acute necrotizing ulcerative gingivitis. C. recurrent herpes labialis. D. acute herpetic gingivostomatitis.

A. recurrent aphthous stomatitis.

A patient's early recovery from an ultrashort-acting barbiturate is related primarily to A. redistribution. B. breakdown in the liver. C. excretion in the urine. D. breakdown in the blood. E. binding to plasma proteins.

A. redistribution.

A patient who has a moderate bony undercut on the facial from canine-to-canine needs an immediate maxillary denture. There is also a tuberosity that is severely undercut. This patient is best treated by A. reducing surgically the tuberosity only. B. reducing surgically the facial bony undercut only. C. reducing surgically both tuberosity and facial bony undercut. D. leaving the bony undercuts and relieving the denture base.

A. reducing surgically the tuberosity only.

In a 12-year-old, an ANB angle of 2 degrees usually signifies that the A. relationship of the maxilla to the mandible is favorable. B. relationship of the mandible to the cranium is favorable. C. cranial growth is poor. D. incisors are upright.

A. relationship of the maxilla to the mandible is favorable.

Visible light curing units are MOST hazardous to the A. retina. B. lens. C. cornea. D. iris.

A. retina.

The ratio of LD50 to ED50 provides information as to the drug's A. safety. B. efficacy. C. potency. D. allergenicity. E. therapeutic application.

A. safety.

The therapeutic index of a given drug indicates the relative A. safety. B. potency. C. efficacy. D. duration. E. solubility.

A. safety.

Postural hypotension is a common complaint of patients who take antihypertensive agents because many of these agents interfere with the A. sympathetic control of vascular reflexes. B. release of acetylcholine in the ganglia. C. epinephrine release from the adrenal medulla. D. parasympathetic control of vascular resistance. E. neuromuscular transmission in skeletal muscles.

A. sympathetic control of vascular reflexes.

The portion of an artificial tooth that is found only in porcelain anterior teeth is A. the pin. B. the collar. C. the finish line. D. the diatoric. E. None of above

A. the pin.

As radiographs are exposed, the heat of the filament determines the quantity of electrons produced. The number of electrons, in turn, determines A. the quantity of X rays produced. B. the quality of X rays produced. C. both quality and quantity of X rays produced.

A. the quantity of X rays produced.

During surgical correction of mandibular prognathism, the position of the mandibular condyle should be A. unaltered. B. retruded. C. advanced. D. intruded.

A. unaltered.

Excessive depth of the posterior palatal seal usually results in A. unseating of the denture. B. a tingling sensation. C. greater retention. D. increased gagging.

A. unseating of the denture.

Delayed eruption, blunted root apices, hypoplastic enamel, and other dental manifestations of hypoparathyroidism, can be prevented by early treatment with A. vitamin D. B. iodine. C. ADH. D. insulin.

A. vitamin D.

One can accelerate the setting time of zinc oxide-eugenol impression pastes by adding a small amount of A. water. B. glycerin. C. petrolatum. D. plaster of paris.

A. water.

Each of the following is a symptom of poisoning by an organophosphate insecticide EXCEPT one. Which one is the EXCEPTION? A. Diarrhea B. Hot, dry skin C. Excessive salivation D. Increased lacrimation E. Skeletal muscle fasciculation

B Hot, dry skin

A patient was administered a flow rate of 4 liters of oxygen and 2 liters of nitrous oxide. What percentage of nitrous oxide did the patient inhale? A. 0.25 B. 0.33 C. 0.5 D. 0.66

B. 0.33

Which of the following is the OPTIMAL reduction for the lingual cusp on Tooth #3 to receive an MODL onlay? A. 1.0 mm B. 1.5 mm C. 2.0 mm D. 2.5 mm

B. 1.5 mm

Guided tissue regeneration is most commonly used to treat which of the following Classes of furcations? A. 1 B. 2 C. 3 D. 4

B. 2

Which of the following represents the median of the data set below? 1, 1, 1, 1, 2, 2, 3, 3, 7, 9 (n=10; sum=30) A. 1 B. 2 C. 3 D. 7 E. 9

B. 2

A patient has a history of significant cardiovascular impairment. The maximum safe dose of epinephrine that can be administered to this patient is A. 1 cc, 1:50,000. B. 2 cc, 1:50,000. C. 1 cc, 1:100,000. D. 2 cc, 1 : 100, 000.

B. 2 cc, 1:50,000.

A dentist uses an unsaturated, chemical sterilizer for sterilizing dental instruments. To effectively kill all microorganisms and spores, this dentist must maintain a temperature of 132 degrees Celsius at 20 - 40 psi for at LEAST how many minutes? A. 10 B. 20 C. 30 D. 40

B. 20

In mg/kg body weight, the lethal dose of fluoride falls in the range of A. 0.5-1.0. B. 20-50. C. 100-200. D. 300-500.

B. 20-50.

Postextraction alveolar osteitis most commonly develops in how many days? A. 1 B. 3 C. 5 D. 7 E. 10

B. 3

The materials used in the walking bleach technique are sodium perborate and A. 3% aqueous hydrogen peroxide. B. 30% aqueous hydrogen peroxide. C. 25% ethereal hydrogen peroxide. D. 5% phosphoric acid.

B. 30% aqueous hydrogen peroxide.

The wave length of light emitted by a light-curing unit should be checked periodically with a dental light meter. This meter detects light only in the range of A. 300-399 nm. B. 400-499 nm. C. 500-599 nm. D. 600-699 nm.

B. 400-499 nm.

The nitrous oxide/oxygen sedation machine is set at 3 liters/min oxygen and 4 liters/min nitrous oxide. What is the concentration of oxygen administered? A. 33 percent B. 43 percent C. 53 percent D. 63 percent E. 73 percent

B. 43 percent

What is the pH threshold level at which enamel demineralization occurs? A. 6.5 B. 5.5 C. 4.5

B. 5.5

Which of the following represents a dental program in which eligible patients receive services at specified facilities from a limited number of dentists? A. An open-panel B. A closed-panel C. A capitation group D. A prepaid group

B. A closed-panel

Which of the following best defines the term "Temporomandibular Disorders"? A. A clinical condition related to dysfunction of the temporomandibular joint and disk B. A collective term for a heterogeneous group of musculoskeletal disorders of varying etiologies, that present with similar signs and symptoms C. A clinical condition caused by disharmony between the occlusal proprioceptors and masticatory muscles D. A group of clinical conditions that arise from excessive loads being placed on the TM joints by muscle spasms, resulting in disk displacement on one or both sides

B. A collective term for a heterogeneous group of musculoskeletal disorders of varying etiologies, that present with similar signs and symptoms

Each of the following osseous defects would be classified as infrabony EXCEPT one. Which one is the EXCEPTION? A. A trough B. A dehiscence C. A hemiseptum D. An interdental crater

B. A dehiscence

Each of the following osseous defects would be classified as infrabony EXCEPT one. Which one is this EXCEPTION? A. A trough B. A dehiscence C. A hemiseptum D. An interdental crater

B. A dehiscence

Which of the following drugs produces sufficient central nervous system depression to cause a state of sleep from which one may be aroused? A. A sedative B. A hypnotic C. An opiate D. An anesthetic

B. A hypnotic

Which of the following patients should preferably be hospitalized for the oral surgery procedure indicated? A. A patient with a prosthetic cardiac valve, who requires a single tooth removed, and an alveoloplasty procedure. B. A severely retarded patient, who requires full mouth gingivectomy for treatment of phenytoin-induced hyperplasia. C. A patient on anticoagulant therapy whose present prothrombin time is 1.25 times the control time, who requires removal of a single tooth. D. A juvenile diabetic whose disease is controlled by daily insulin injections, who requires removal of a chronically infected tooth.

B. A severely retarded patient, who requires full mouth gingivectomy for treatment of phenytoin-induced hyperplasia.

In an attempted venipuncture, each of the following indicates accidental intra-arterial injection EXCEPT one. Which one is this EXCEPTION? A. The needle moves with pulsation action. B. A vessel tends to collapse and obstruct. C. Aspirated blood is bright red in color. D. Injection of a small test dose of the agent is acutely painful. E. Penetration of a vessel meets resistance and is painful.

B. A vessel tends to collapse and obstruct.

Which of the following conditions has a very favorable tooth- whitening prognosis? A. Blue-gray teeth B. Aged-yellow teeth C. Teeth exhibiting amelogenesis imperfecta D. Old composite-resin restorations

B. Aged-yellow teeth

To remain stable, a rubber-dam clamp must contact the anchor tooth gingival to the height of contour. Which other criterion must the clamp satisfy? A. All four points must be sharp. B. All four points must contact the tooth. C. The bow must be directed to the distal side of the tooth.

B. All four points must contact the tooth.

Which of the following contributes principally to the increase in height of maxillary bones? A. Sutural growth B. Alveolar growth C. Apposition on the tuberosity D. Apposition on the anterior surface

B. Alveolar growth

Which of the following filling materials is least desirable for use as Preventive Resin Restoration? A. Microfilled resin B. Amalgam C. Hybrid resin D. Glass ionomer

B. Amalgam

Additional space for successive eruption of permanent maxillary molars is provided by A. Interstitial bone growth. B. Appositional growth at the maxillary tuberosity. C. Continuous expansion of the dental arch due to sutural growth. D. An increase in palatal vault height due to alveolar growth.

B. Appositional growth at the maxillary tuberosity.

Which of the following represents the MOST effective way of encouraging a child to floss regularly? A. Presenting a message that concerns the potential loss of teeth, should the patient not floss. B. Arranging for the child to earn privileges for flossing. C. Repeating periodically the message on the value of flossing. D. Providing written material that explains the mechanism of flossing.

B. Arranging for the child to earn privileges for flossing.

A dentist has offered plaque-control instructions to a patient. Which of the following should the dentist do to make certain that the patient understands these instructions? A. Repeat the message a second time using simpler words B. Ask the patient to verbalize her understanding C. Ask the patient if she understands D. Schedule a review appointment in two weeks

B. Ask the patient to verbalize her understanding

A 16-year-old patient presents with a firm, non-painful swelling in the mandibular premolar area. Adjacent teeth are vital. Radiographic evaluation reveals a discrete 4 cm radiolucency in the premolar area. Which of the following diagnostic procedures would be performed next? A. Excisional biopsy B. Aspiration C. CT scan D. Alkaline phosphatase determination E. Complete blood cell count with a differential

B. Aspiration

Which of the following principles requires health professionals to inform their patients about treatment and to protect their confidentiality? A. Justice B. Autonomy C. Beneficence D. Nonmaleficence

B. Autonomy

Which of the following drug groups increases intraocular pressure and is, therefore, contraindicated in patients with glaucoma? A. Catecholamines B. Belladonna alkaloids C. Anticholinesterases D. Organophosphates

B. Belladonna alkaloids

Ectodermal dysplasia is characterized by each of the following EXCEPT one. Which one is this EXCEPTION? A. Hypodontia (oligodontia) B. Blue sclera C. Atrophic skin D. Defective hair E Hypoplastic sweat glands

B. Blue sclera

Maxillary first premolars with mesiodistal furcation involvements are often managed successfully by surgery. Therefore, they have a good prognosis and can be included as key abutments in a fixed prosthesis. A. Both statements are TRUE. B. Both statements are FALSE. C. The first statement is TRUE, the second is FALSE. D. The first statement is FALSE, the second is TRUE.

B. Both statements are FALSE.

Oral hygiene improvement in the presence of systemic or nutritional deficiencies affects the incidence of chronic inflammatory periodontal disease. It also affects the severity of chronic inflammatory periodontal disease. A. Both statements are TRUE. B. Both statements are FALSE. C. The first statement is TRUE, the second is FALSE. D. The first statement is FALSE, the second is TRUE.

B. Both statements are TRUE.

A traumatic injury can cause the pulp space to calcify, because the accident can trigger odontoclasts into accelerated activity. A. Both the statement and the reason are correct and related. B. Both the statement and the reason are correct but NOT related. C. The statement is correct but the reason is NOT. D. The statement is NOT correct but the reason is correct. E. NEITHER the statement NOR the reason is correct.

B. Both the statement and the reason are correct but NOT related.

Which of the following describes a material with high compressive strength but low tensile strength? A. Resilient B. Brittle C. Ductile D. Malleable

B. Brittle

Which of the following describes a material with high compressive strength but low tensile strength? A. Tough B. Brittle C. Ductile D. Malleable E. Resilient

B. Brittle

The pterygomandibular raphe is a tendon between which muscles? A. Stapedius and stylohyoid B. Buccinator and superior constrictor C. Medial pterygoid and lateral pterygoid D. Levator veli palatini and tensor veli palatini E. Anterior belly of digastric and sternocleidomastoid

B. Buccinator and superior constrictor

Which of the following drugs causes the LEAST CNS depression and impairment of psychomotor skills? A. Diazepam (Valium®) B. Buspirone (Buspar®) C. Alprazolam (Xanax®) D. Chloral hydrate

B. Buspirone (Buspar®)

During which stage of tooth development is the cariostatic effect of fluoride manifested? A. Apposition B. Calcification C. Proliferation D. Histodifferentiation

B. Calcification

Children and adolescents are the LEAST likely to develop which of the following? A. Ameloblastic fibroma B. Calcifying epithelial odontogenic tumor C. Adenomatoid odontogenic tumor D. Compound odontoma

B. Calcifying epithelial odontogenic tumor

A patient is receiving radiotherapy for cancer. Which of the following represents the MOST common oral infection that necessitates drug treatment in this patient? A. Herpes simplex B. Candida albicans C. Histoplasma capsulatum D. Staphylococcus aureus E. Streptococcus viridans

B. Candida albicans

The mandibular intercanine width continues to increase until which permanent tooth is fully erupted? A. Lateral incisor B. Canine C. First premolar D. Second premolar

B. Canine

Which of the following represent(s) the serious effects of a repeated exposure to low doses of X radiation? A. Purpura B. Carcinogenesis C. Diarrhea and dehydration D. Alteration of the oral microflora

B. Carcinogenesis

Which of the following eruption sequences is most commonly found in the primary dentition? A. Central incisor, lateral incisor, canine, first molar, and second molar B. Central incisor, lateral incisor, first molar, canine, and second molar C. First molar, central incisor, lateral incisor, canine, and second molar D. First molar, central incisor, lateral incisor, second molar, and canine

B. Central incisor, lateral incisor, first molar, canine, and second molar

Which of the following represents a constitutional symptom? A. Tinnitus B. Chills C. Xerostomia D. Dysphagia

B. Chills

A child who has a distal step in the primary dentition generally develops which of the following molar relationships in the permanent dentition? A. Class I B. Class II C. Class III

B. Class II

Each of the following statements describes intracanal instruments and their action EXCEPT one. Which one is this EXCEPTION? A. A K-type file can be used in a filing and reaming action, while a reamer can be used in only a reaming action. B. Clockwise-turning of a root canal instrument will force debris in an apical direction, while counterclockwise-turning will cause debris to move in a coronal direction out of the canal. C. A reaming action will enlarge a root canal and produce a cross-sectional preparation that is approximately round in shape. D. An intracanal instrument is more susceptible to breakage if locked in dentin and then rotated.

B. Clockwise-turning of a root canal instrument will force debris in an apical direction, while counterclockwise-turning will cause debris to move in a coronal direction out of the canal.

After receiving one cartridge of a local anesthetic, a healthy adult patient became unconscious in the dental chair. The occurrence of a brief convulsion is A. Pathognomonic of grand mal epilepsy. B. Consistent with a diagnosis of syncope. C. Usually caused by the epinephrine in the local anesthetic. D. Pathognomonic of intravascular injection of a local anesthetic.

B. Consistent with a diagnosis of syncope.

Which of the following most strongly suggest cause-and-effect relationships? A. Correlational studies B. Controlled clinical trials C. Case reports D. Epidemiologic surveys

B. Controlled clinical trials

Each of the following characterizes an impending insulin shock EXCEPT one. Which one is the EXCEPTION? A. Weakness B. Convulsions C. Mental confusion D. Cold perspiration

B. Convulsions

A practitioner who is restoring a tooth with composites wishes to be sure that the matrix has adhered to the filler. Which of the following agents adheres the resin matrix to the filler? A. Wetting B. Coupling C. Catalyzing D. Activating

B. Coupling

A dentist extracts a patient's mandibular molar, fracturing the mesial root. The distal root and superior portion of the mesial root are removed with the crown. The remaining portion of the mesial root should be removed with a A. Crane pick. B. Cryer elevator. C. straight elevator. D. lower universal forceps.

B. Cryer elevator.

Principles for managing a localized alveolar osteitis after extraction of a mandibular first molar include each of the following EXCEPT one. Which one is the EXCEPTION? A. Flushing out debris with normal saline solution B. Curetting the boney wall to promote bleeding C. Placing a sedative dressing in the socket to protect exposed bone D. Administering mild analgesic drugs as an adjunct to local treatment

B. Curetting the boney wall to promote bleeding

Which condition describes a combination of steatorrhea, chronic respiratory infections, and functional disturbances in secretory mechanisms of various glands? A. Lead poisoning B. Cystic fibrosis C. Juvenile onset diabetes D. Acquired immune deficiency syndrome

B. Cystic fibrosis

A patient will receive a free gingival graft. The graft epithelium will undergo which of the following alterations? A. Dysplasia B. Degeneration C. Proliferation D. Orthokeratinization E. Formation of keratohyalin granules

B. Degeneration

Some teeth appear to be clinically normal, but exhibit (1 ) globular dentin, (2) very early pulpal obliteration, (3) defective root formation, (4) periapical granulomas and cysts, and (5) premature exfoliation. The condition is known as which of the following? A. Shell teeth B. Dentin dysplasia C. Regional odontodysplasia D. Amelogenesis imperfecta E. Dentinogenesis imperfecta

B. Dentin dysplasia

Which of the following is the most important factor affecting pulpal response? A. Heat B. Depth to which dentinal tubules are cut C. Desiccation D. Invasion of bacteria

B. Depth to which dentinal tubules are cut

Which of the following systemic conditions lowers resistance, impairs healing, and indicates early use of antibiotics for infections? A. Vitamin C deficiency B. Diabetes mellitus C. Polycythemia vera D. Systemic lupus erythematosus

B. Diabetes mellitus

Which of the following drugs, when administered intravenously, is LEAST likely to produce respiratory depression? A. Fentanyl B. Diazepam C. Thiopental D. Meperidine E. Pentobarbital

B. Diazepam

Which of the following dentifrice components is most likely to inactivate the fluoride ion? A. Anionic detergent B. Dicalcium phosphate C. Polyacrylic spheres D. Monofluorophosphate

B. Dicalcium phosphate

A dentist administers very high concentrations of nitrous oxide to the patient. The patient inhales room air immediately after the nitrous oxide is terminated. The patient is likely to experience which of the following complications? A. Vasovagal syncope B. Diffusion hypoxia C. Carbon dioxide retention D. Hyperventilation syndrome E. Respiratory depression of apnea

B. Diffusion hypoxia

Which of the following drugs is commonly used in the treatment of congestive heart failure? A. Phenytoin B. Digitalis C. Quinidine D. Procainamide E. Nitroglycerin

B. Digitalis

A patient's permanent tooth crown fractures, creating a small (1 mm) pulp exposure for about 30 minutes. Which of the following pulp therapies is the most appropriate for this patient? A. Pulpectomy and apexification, if necessary B. Direct pulp capping with calcium hydroxide C. Pulpotomy with formocresol D. Pulpotomy with calcium hydroxide

B. Direct pulp capping with calcium hydroxide

A chronic infection of the mandible of a middle-aged female patient has been treated with oral penicillin for three weeks. The primary problem appears to be resolved, but the patient now has many white patches on the dorsum of the tongue and buccal mucosa. Which of the following is the appropriate care for this patient? A. Discontinue all antibiotics. B. Discontinue penicillin and prescribe nystatin troches. C. Discontinue penicillin and prescribe a mouthwash containing metronidazole. D. Discontinue penicillin treatment and substitute a broad spectrum antibiotic.

B. Discontinue penicillin and prescribe nystatin troches.

In a protrusive condylar movement, interferences can occur between which posterior cusp inclines? A. Mesial inclines of maxillary, distal inclines of mandibular B. Distal inclines of maxillary, mesial inclines of mandibular C. Mesial inclines of maxillary, mesial inclines of mandibular D. Distal inclines of maxillary, distal inclines of mandibular

B. Distal inclines of maxillary, mesial inclines of mandibular

Which of the following describes ciprofloxacin (Cipro®)? A. Inhibits cell wall synthesis. B. Effective against Pseudomonas aeruginosa. C. Effective only against anaerobic bacteria. D. An antibiotic-of-choice for treating otitis media in young children. E. Effective against oral anaerobes.

B. Effective against Pseudomonas aeruginosa.

X-rays are produced when A. Protons strike the anode. B. Electrons strike the anode. C. The anode is heated above 3,000 degrees Centigrade. D. The filament becomes positively charged.

B. Electrons strike the anode.

During root planing and scaling, each of the following can take place EXCEPT one. Which one is this EXCEPTION? A. Dentin removal B. Enamel removal C. Calculus removal D. Cementum removal E. Inadvertent curettage

B. Enamel removal

Calcium hydroxide is generally the material-of-choice in vital pulp capping because it A. Is less irritating to the pulp. B. Encourages dentin bridge formation. C. Seals the cavity better than most other materials.

B. Encourages dentin bridge formation.

Which of the following antibiotics is the substitute of choice for penicillin in the penicillin-sensitive Patient? A. Cephalexin B. Erythromycin C. Tetracycline D. Clindamycin

B. Erythromycin

Which of the following should be performed first for a collapsed or unconscious victim of illness or accident? A. Open the airway B. Establish unresponsiveness C. Establish pulselessness D. Examine the victim for bleeding E. and fractures

B. Establish unresponsiveness

Which of the following is the best reason for pouring a condensation impression material as soon as possible? A. The degree of polymerization of condensation polymers is initially high and then decreases with time. B. Evaporation of a volatile by-product causes shrinkage of the set material. C. If left unpoured, condensation impression materials expand as they absorb water from air. D. Condensation polymers will start to react with the polymer of the impression tray and cause distortion.

B. Evaporation of a volatile by-product causes shrinkage of the set material.

Which of the following is the most common reason for a cast crown not to seat on a patient's tooth? A. Overextended margins B. Excessive proximal contact C. Undercut areas on the preparation D. Casting too wide bucco-lingually E. Porosity within the inner surface of the crown

B. Excessive proximal contact

When a removable partial denture is terminally seated, the retentive clasp tips should A. Be invisible. B. Exert no force. C. Apply retentive force into the body of the teeth. D. Resist torque through the long axis of the teeth.

B. Exert no force.

Each of the following determines the outline form for a Class III composite restoration EXCEPT one. Which one is this EXCEPTION? A. Convenience for access B. Extension for prevention C. Size, shape, and location of caries

B. Extension for prevention

When root canal therapy is indicated for a vital tooth, which of the following is the MOST biologically acceptable? A. Filing to the radiographic apex and filling to the radiographic apex B. Filing 0.5 - 1.0 mm short of the radiographic apex and filling to the same length C. Filing to the radiographic apex and filling 0.5 - 1 .0 short of the radiographic apex D. Filing 0.5 - 1.0 mm beyond the radiographic apex and filling to the radiographic apex

B. Filing 0.5 - 1.0 mm short of the radiographic apex and filling to the same length

Squamous cell carcinoma appears in a variety of locations. In which of the following locations is its prognosis the LEAST favorable? A. Lower lip B. Floor of the mouth C. Hard palate D. Buccal mucosa

B. Floor of the mouth

Which of the following drugs best reverses the effects of benzodiazepines? A. Naloxone B. Flumazenil C. Midazolam D. Aminophylline E. Physostigmine

B. Flumazenil

The children in an elementary school exhibit a high interproximal caries rate. For these children, which of the following school-based programs will be the MOST effective? A. Sealant B. Fluoride mouthrinse C. Brushing and flossing

B. Fluoride mouthrinse

The routine examination of a healthy 20-year-old man discloses a round, brown lesion 3 mm in diameter, on his lower lip. The lesion is solitary, asymptomatic, and flat. Its duration is unknown. Which of the following is the MOST likely diagnosis? A. Blue nevus B. Focal melanosis C. Compound nevus D. Malignant melanoma E. Peutz-Jeghers syndrome

B. Focal melanosis

What type of mucoperiosteal flap is used for removal of a mesiodens? A. Envelope B. Full palatal C. Facial flap with relaxing incisions D. Facial flap without relaxing incisions E. None of these

B. Full palatal

Which of the following legally protects health professionals who provide emergency treatment at the scene of an accident? A. Health Professional Protection Act B. Good Samaritan Act C. States' Human Rights Act D. Americans with Disabilities Act

B. Good Samaritan Act

Which of the following describes the use of hydroxylapatite for alveolar ridge augmentation? A. Hydroxylapatite is considered to be osteoinductive (bone forming). B. Granule migration and excessive augmentation are common problems with hydroxylapatite materials. C. Compaction or compression of hydroxylapatite after augmentation generally does not occur. D. Patients with hydroxylapatite augmentation to the anterior maxilla should be without their maxillary prosthesis for a minimum of one month.

B. Granule migration and excessive augmentation are common problems with hydroxylapatite materials.

An "emergency" occlusal separator should be constructed for patients who has which of the following conditions? A. Myositis B. Hemarthrosis C. Capsular fibrosis D. Degenerative arthritis E. Unilateral condylar hyperplasia

B. Hemarthrosis

Which of the following bone donor grafts has the greatest osteogenic potential? A. Freeze-dried bone B. Hemopoietic marrow C. Heterogenous marrow D. Corticocancellous block

B. Hemopoietic marrow

Inadvertent crush injuries to the lip are caused by which of the following parts of the forcep? A. Beak B. Hinge C. Shank D. Blade E. Handle

B. Hinge

A patient works at a nuclear power plant. He is frequently exposed to a small amount of radiation, although the dose he receives is below the occupational limits. Which of the following best explains why this patient need not wear his employee film badge while he is having dental radiographs made? A. His badge records only gamma-radiation, not x rays. B. His film badge must be used only to measure occupational doses. C. It is unlikely that the current dose will rise above his occupational limit. D. The dose from the dental radiographs is too low to be measured accurately by the badge.

B. His film badge must be used only to measure occupational doses.

Each of the following is a symptom of poisoning by an organophosphate insecticide EXCEPT one. Which one is this EXCEPTION? A. Diarrhea B. Hot, dry skin C. Excessive salivation D. Increased lacrimation E. Skeletal muscle fasciculation

B. Hot, dry skin

Which of the following sedatives is most likely to cause a dry mouth? A. Buspirone (BuSpar®) B. Hydroxyzine (Vistaril®) C. Chloral hydrate (Noctec®) D. Phenobarbital (Luminal®)

B. Hydroxyzine (Vistaril®)

Which alteration of which type of papillae results in black hairy tongue? A. Atrophy of Filiform B. Hypertrophy of Filiform C. Atrophy of Fungiform D. Hypertrophy of Fungiform

B. Hypertrophy of Filiform

Which of the following responses made to a child is MOST likely to reinforce positively the child's appropriate behavior in the dental setting? A. You're doing a great job for me today. B. I like it when you sit quietly in the chair. C. This visit is going really well, don't you think? D. Coming to the dentist isn't so bad after all, is it?

B. I like it when you sit quietly in the chair.

Primary crown mineralization (calcification) generally begins A. In the third to sixth week in utero. B. In the second trimester. C. In the third trimester. D. At birth.

B. In the second trimester.

Selective beta1-adrenergic agonists produce which of the following effects? A. Glycogenolysis B. Increased cardiac output C. Decreased peripheral resistance D. Relaxation of bronchial smooth muscle

B. Increased cardiac output

Biologic changes in the aging patient affect both pharmacokinetics and pharmacodynamics of drugs, which in turn, are manifested as each of the following EXCEPT one. Which one is this EXCEPTION? A. Decreased renal excretion B. Increased plasma protein binding C. Increased half lives of some drugs D. Decreased rate of biotransformation E. Altered volume of distribution and sequestration of drug in body fat

B. Increased plasma protein binding

The strength of a soldered connector of a fixed partial denture is best enhanced by A. Using a higher carat solder. B. Increasing its height. C. Increasing its width. D. Increasing the gap.

B. Increasing its height.

Injection of a local anesthetic into an inflamed area usually has a less than optimal result. Which of the following best explains why? A. The prostaglandins stabilize the nerve membrane. B. Inflammation reduces the availability of the free base. C. The drug will be absorbed more rapidly because of the increased blood supply. D. The chemical mediators of inflammation will present a chemical antagonism to the anesthetic.

B. Inflammation reduces the availability of the free base.

Occlusal morphology is determined by the height of the cusp and the depth of the fossa. Each of the following is a vertical component of this morphology EXCEPT one. Which one is this EXCEPTION? A. Side shift B. Intercondylar distance C. Curve of Spee D. Horizontal overlap of anterior teeth

B. Intercondylar distance

When the mandible is in its physiologic rest position, the distance between the occluding surfaces of the maxillary and mandibular teeth or occlusion rims is A. Vertical dimension. B. Interocclusal distance. C. Vertical dimension of rest. D. Centric occlusion (maximum intercuspation).

B. Interocclusal distance.

Which of the following traumatic injuries is most likely to result in pulp necrosis of a permanent tooth? A. Luxation B. Intrusion C. Extrusion D. Concussion E. Subluxation

B. Intrusion

Which of the following combinations is appropriate for treatment of uncomplicated respiratory tuberculosis? A. Streptomycin and chloramphenicol, because both are lipid- soluble. B. Isoniazid (INH) and rifampin, because they have different modes of action. C. Ethambutol and isoniazid, because both drugs inhibit bacterial folic acid synthesis. D. Griseofulvin and streptomycin, because both drugs accumulate on bacterial 50S ribosomes. E. None of these

B. Isoniazid (INH) and rifampin, because they have different modes of action.

Why is a matrix for a Class II dental amalgam restoration extended occlusally to the cavity preparation? A. It serves as a guide to determine the completed restoration. B. It allows for overfilling the amalgam. C. It prevents escape of the amalgam during condensation.

B. It allows for overfilling the amalgam.

A patient says to the dentist, "I know it would be good if I'd brush more regularly, but it's difficult to get into that habit." Which of the following statements the dentist might make illustrates a reflection-of-feeling response? A. Really, you should just make a point of doing so. B. It can be frustrating trying to begin a new habit. C. I know you'll be able to do better in the future. D. Some things you just have to make yourself do.

B. It can be frustrating trying to begin a new habit.

A dentist who is planning to restore a tooth with gold alloy desires a certain amount of expansion in the casting investment mold. Which of the following best explains why this expansion is desirable? A. It allows for the shrinkage of the molten metal and the wax pattern. B. It compensates for the shrinkage of the molten metal as it cools in the mold. C. It makes the metal casting larger than the wax pattern so that the casting will fit the tooth tightly.

B. It compensates for the shrinkage of the molten metal as it cools in the mold.

The dentist will use the acid-etch technique to make a minimal cavity preparation for composite resin. Each of the following is an advantage of this technique EXCEPT one. Which one is this EXCEPTION? A. It conserves tooth structure. B. It provides greater access for finishing procedures. C. It improves the esthetics of the restoration.

B. It provides greater access for finishing procedures.

Which of the following conclusions would be correct if, after six weeks, a pulp-capped tooth were asymptomatic? A. Pulp capping was a success. B. Lack of adverse symptoms might be temporary. C. Reparative dentin formation at the exposure site was complete. D. Adjacent odontoblasts had proliferated to cover the site of exposure.

B. Lack of adverse symptoms might be temporary.

A 35-year-old patient presents with swelling facial to a mandibular premolar. There is no severe pain, only mild discomfort. The involved teeth are vital and there are no periodontal pockets. Radiograph reveals a round radiolucency in the middle third of the root. Which of the following represents the most likely diagnosis? A. Gingival abscess B. Lateral periodontal cyst C. Osteogenic sarcoma D. Periodontal abscess E. Root resorption

B. Lateral periodontal cyst

Which of the following is the most commonly used surgical procedure to correct maxillary retrognathia? A. C-osteotomy B. Le Fort I osteotomy C. Inverted L-osteotomy D. Anterior maxillary osteotomy

B. Le Fort I osteotomy

Which of the following is the most commonly used surgical procedure to correct maxillary retrognathia? A. C-osteotomy B. LeFort I osteotomy C. Inverted L osteotomy D. Anterior maxillary osteotomy

B. LeFort I osteotomy

The late mesial shift of a permanent first molar is primarily the result of closure of which of the following spaces? A. Canine B. Leeway C. Primate D. Extraction

B. Leeway

The dentist plans to place a crown on a patient's maxillary canine. Altering the existing canine-guided occlusion in a right-lateral excursive movement to that of a group-function will result in which of the following? A. Greater clearance on the left side-less potential for a non- working contact B. Less clearance on the left side--greater potential for a non-working contact C. No effect on the non-working side clearance

B. Less clearance on the left side--greater potential for a non-working contact

Which of the following explains why a properly designed rest on the lingual surface of a canine is preferred to a properly designed rest on the incisal surface? A. The enamel is thicker on the lingual surface. B. Less leverage is exerted against the tooth by the lingual rest. C. The visibility of, as well as access to, the lingual surface is better. D. The cingulum of the canine provides a natural surface for the recess.

B. Less leverage is exerted against the tooth by the lingual rest.

Which of the following systemic disorders can modify the inflammatory reaction? A. Epilepsy B. Leukemia C. Hemophilia D. Hyperparathyroidism

B. Leukemia

Which of the following drugs is often administered intravenously to treat life-threatening ventricular arrhythmias? A. Quinidine B. Lidocaine C. Verapamil D. Propranolol

B. Lidocaine

Which of the following local anesthetics is most effective as a topical agent? A. Procaine B. Lidocaine C. Prilocaine D. Bupivacaine

B. Lidocaine

Periapical radiographs of the mandibular premolar region reveals a completely embedded, supernumerary tooth. The image of this tooth is superimposed over the first premolar and canine roots. These radiographs suggest that the supernumerary tooth lies A. Facial to the first premolar. B. Lingual to the first premolar. C. Facial to the canine. D. Lingual to the canine. E. In the same plane as the erupted teeth.

B. Lingual to the first premolar.

Where is the orifice to the fourth canal in a maxillary first molar usually found? A. Under the distofacial cusp B. Lingual to the orifice of the mesiofacial canal C. On a line running from the palatal orifice to the distofacial orifice D. On a line running from the distofacial orifice to the mesiofacial orifice

B. Lingual to the orifice of the mesiofacial canal

Major connectors most frequently encounter interferences from which of the following? A. Lingually inclined maxillary molars B. Lingually inclined mandibular premolars C. Facially inclined maxillary molars and premolars D. Bony areas on the facial aspect of edentulous spaces

B. Lingually inclined mandibular premolars

Which of the following represents the most common form of gingival periodontal disease in school-aged children? A. Juvenile periodontitis B. Localized acute gingivitis C. Primary herpetic gingivostomatitis D. Necrotizing ulcerative gingivitis

B. Localized acute gingivitis

Which of the following is classified as an antianxiety drug? A. Methohexital B. Lorazepam C. Haloperidol D. Pentazocine E. Phenylpropanolamine

B. Lorazepam

Which of the following is usually NOT used for IV conscious sedation in the dental office because of its long duration of clinically significant sedative effect? A. Diazepam B. Lorazepam C. Midazolam D. Hydroxyzine E, Propofol

B. Lorazepam

In a state of gingival health, the narrowest width of keratinized gingival tissue is usually found on the facial aspect of which of the following teeth? A. Mandibular central incisors B. Mandibular first premolars C. Maxillary second premolars

B. Mandibular first premolars

A dentist will perform a Moyers' mixed dentition analysis. Which of the following teeth will be measured to predict the size of the unerupted canines and premolars? A. Maxillary incisors B. Mandibular incisors C. Primary molars and canines D. Maxillary incisors for the maxillary arch; mandibular incisors for the mandibular arch

B. Mandibular incisors

Which of the following teeth most often refer pain to the ear? A. Maxillary molars B. Mandibular molars C. Maxillary premolars D. Maxillary anteriors E. Mandibular anteriors

B. Mandibular molars

Which of the following mesial root surfaces are unlikely to be thoroughly cleaned with the use of floss? A. Maxillary central incisor B. Mandibular first premolar C. Maxillary first premolar D. Mandibular first molar

B. Maxillary first premolar

Facial edema, cheilitis granulomatosa, and a fissured tongue characterize which of the following syndromes? A. Frey's B. Melkersson-Rosenthal C. Treacher Collins

B. Melkersson-Rosenthal

Which of the following is the most likely indication for splinting? A. Primary occlusal trauma B. Mobility with patient discomfort C. Mobility with a decrease in tissue quality, secondary to hormonal imbalance D. Mobility related to a unilateral "skid" from centric relation to centric occlusion

B. Mobility with patient discomfort

The depth of the clinical gingival sulcus is the distance from the gingival margin to the A. Cementoenamel junction. B. Most apical penetration of the periodontal probe. C. Most apical extension of the junctional epithelium. D. Bottom of the sulcus in well-preserved histologic block sections.

B. Most apical penetration of the periodontal probe.

Which of the following is a "third generation" cephalosporin? A. Cephamandole B. Moxalactam C. Amikacin D. Cephalexin E. Clavulanic acid

B. Moxalactam

When using an occlusal separator, a practitioner can expect which of the following types of TMJ disorders to respond most favorably? A. Chronic dislocation B. Muscle spasm C. Capsular fibrosis D. Unilateral condylar hyperplasia

B. Muscle spasm

A defect in neuromuscular transmission causes which of the following? A. Bell's palsy B. Myasthenia gravis C. Muscular dystrophy D. Multiple sclerosis E. Trigeminal neuralgia

B. Myasthenia gravis

A 26-year-old male patient complains of a tender swelling in his palate. The radiograph of this area (seen in the radiograph) strongly suggests the presence of which type of cyst? A. Radicular B. Nasopalatine C. Nasolabial D. Residual

B. Nasopalatine

An adult patient presents with multiple, soft nodules and with macular pigmentation of the skin. Which of the following best represents this condition? A. Lipomatosis B. Neurofibromatosis C. Metastatic malignant melanoma D. Polyostotic fibrous dysplasia E. Nevoid basal cell carcinoma syndrome

B. Neurofibromatosis

A 4-year-old child is shy, timid, and fearful. Which of the following will be MOST appropriate for the restorative appointments for this child? A. Naloxone (Narcan®) B. Nitrous oxide/oxygen C. Promethazine (Phenergan®) alone D. Hydroxyzine hydrochloride (Atarax®) or hydroxyzine pamoate (Vistaril®) in divided doses E. Meperidine (Demerol®), promethazine (Phenergan®) and chlorpromazine (Thorazine®) combined

B. Nitrous oxide/oxygen

Which of the following represents the drug-of-choice in the treatment of candidiasis for an HIV-infected patient? A. Acyclovir B. Nystatin C. AZT D. Chlorhexidine

B. Nystatin

When both nitrous oxide and oxygen tanks are used to one-half of their capacities, what are their gas pressures in psi (The starting gas pressures are 02 [2,000]; N20 [750])? A. O2 = 1000; N2O = 325 B. O2 = 2000; N2O = 750 C. O2 = 1000; N2O = 750 D. O2 = 2000; N2O = 325

B. O2 = 2000; N2O = 750

Which of the following computer databases contains references to dental literature electronically? A. LEXUS B. OVID-MEDLINE C. Dental Abstracts D. Index to Dental Literature

B. OVID-MEDLINE

Which of the following should be done when a patient presents with a low, attached maxillary frenum, accompanied by a 3 mm diastema in the early transitional dentition? A. Perform a frenectomy when first observed B. Observe until after eruption of permanent canines C. Render treatment before complete eruption of maxillary incisors D. Instruct patient to wear an elastic around teeth to close diastema

B. Observe until after eruption of permanent canines

Which of the following statements is true concerning anticholinesterase? A. Agents such as DEP inhibit only plasma cholinesterase. B. Organophosphates are readily absorbed through the skin. C. Agents may cause paroxysmal supraventricular tachycardia. D. Reactivators such as 2-PAM reactivate ACHE which has undergone "aging". E. Alkaloid physostigmine has the greatest number of side effects unrelated to ACHE inhibition.

B. Organophosphates are readily absorbed through the skin.

A 50-year-old male presents with deep fibrotic pockets and angular bone loss. The dentist reflects a flap, debrides the two and three-walled defects, and performs scaling and root planing. Which procedure should the dentist do next? A. Gingivectomy B. Osseous grafts C. Modified Widman flap D. Apically repositioned flap

B. Osseous grafts

A patient requires tooth extractions from an area that has been subjected to radiation therapy. Which of the following represents the greatest danger to this patient? A. Alveolar osteitis B. Osteoradionecrosis C. Prolonged healing D. Fracture of the mandible

B. Osteoradionecrosis

When administered as oral centrally acting analgesics, which of the following is considered to have the highest dependence liability? A. Codeine B. Oxycodone (in Percodan®) C. Propoxyphene (Darvon®) D. Pentazocine (Talwin®)

B. Oxycodone (in Percodan®)

Which of the following conditions indicates that a periodontal, rather than an endodontic problem, exists? A. Acute pain to percussion with no swelling B. Pain to lateral percussion with a wide sulcular pocket C. A deep narrow sulcular pocket to the apex with exudate D. Pain to palpation of the buccal mucosa near the tooth apex

B. Pain to lateral percussion with a wide sulcular pocket

For patients who have AIDS, which of the following locations is the MOST common for Kaposi's sarcoma? A. Tongue B. Palate C. Gingiva D. Buccal mucosa E. Floor of the mouth

B. Palate

New attachment is the reunion of connective tissue and root separated by which of the following? A. Incision B. Pathology C. Injury D. Iatrogenic causes

B. Pathology

Each of the following will assist a dentist in diagnosing or assessing Sjogren's syndrome EXCEPT one. Which one is this EXCEPTION? A. Obtaining sialograms B. Performing exfoliative cytology C. Obtaining blood samples for serology D. Determining salivary flow rate E. Obtaining labial gland biopsy

B. Performing exfoliative cytology

Which of the following symptoms is the most distinct characteristic of morphine poisoning? A. Comatose sleep B. Pin-point pupils C. Depressed respiration D. Deep, rapid respiration E. Widely dilated, non-responsive pupils

B. Pin-point pupils

A child has a very sore ulceration of the lower lip. There is no history of obvious trauma. The ulceration appeared several hours after the patient received dental treatment. Which of the following represents the most likely diagnosis? A. Herpes labialis B. Postanesthetic lip bite C. Chemical burn from aspirin D. Aphthous ulcer secondary to stress E. Allergic stomatitis to the rubber dam

B. Postanesthetic lip bite

How does the growth of the cranial base generally relate in time to the growth of the jaws? A. Follows B. Precedes C. Unrelated D. Accompanies E. Initially follows, then accompanies

B. Precedes

For an extension-base removable partial denture, which of the following is the most important to maintain the remaining supporting tissues? A. Using stress releasing clasps B. Preserving denture base support C. Limiting eccentric occlusal contacts D. Using plastic teeth

B. Preserving denture base support

A dentist is considering the use of nitrous-oxide conscious sedation for a patient. However, this type of sedation will be CONTRAINDICATED, should the patient have a history of which of the following? A. Dental anxiety B. Psychotic care C. Controlled hypertension

B. Psychotic care

The dentist is performing a block of the maxillary division of the trigeminal nerve. Into which anatomical area must the local anesthetic solution be deposited or diffused? A. Pterygomandibular space B. Pterygopalatine space C. Retropharyngeal space D. Retrobulbar space E. Canine space

B. Pterygopalatine space

Which of the following diagnostic criteria is the least reliable in assessing the pulp status in the primary dentition? A. Swelling B. Pulp testing C. Spontaneous pain D. Internal resorption

B. Pulp testing

A cooperative 5-year-old child has a carious lesion in her primary mandibular second molar. There is no tooth mobility, but the practitioner notes a small, draining sinus tract adjacent to the tooth. Which of the following is the treatment-of -choice? A. Extraction B. Pulpectomy C. Indirect pulp therapy D. Calcium hydroxide pulpotomy E. Formocresol pulpotomy

B. Pulpectomy

Which of the following is most clearly associated with the appearance of "moon facies"? A. Hyperthyroidism B. Regular corticosteroid use C. Diabetes mellitus D. Pancreatic insufficiency

B. Regular corticosteroid use

A mandibular first molar crown is severely decayed. Each of the following procedures should be used to remove this tooth EXCEPT one. Which one is this EXCEPTION? A. Using a cowhom forceps B. Removing the thin lingual cortical plate of bone C. Elevating the root sections with an east-west elevator D. Splitting the tooth into mesial and distal root sections E. Reflecting a modest buccal flap and removing the buccal bone

B. Removing the thin lingual cortical plate of bone

An 82-year-old woman presents with a large four-surface pin-retained amalgam restoration on Tooth #3. The dentist notes minor recurrent caries along the faciocervical amalgam margin. Which of the following is the treatment-of-choice? A. Replace the restoration B. Repair the defect C. Prepare the tooth for a crown D. Observe at recall

B. Repair the defect

A teen-aged patient presents with numerous proximal carious lesions that undermine the occlusal enamel. Which of the following is the treatment-of-choice? A. Restore involved teeth with onlays to preserve occlusion B. Restore involved teeth as rapidly as possible using dental amalgam C. Place the patient on a prevention regimen and delay treatment until the effectiveness of home care is evaluated

B. Restore involved teeth as rapidly as possible using dental amalgam

A practitioner is restoring the mesio-occlusal marginal ridge of a maxillary left second molar. If the marginal ridge is higher than the adjacent tooth, then it can create a problem in which of the following excursions? A. Protrusive B. Retrusive C. Working D. Non-working

B. Retrusive

Which of the following antibiotics has been implicated in the vast majority of cases of oral contraceptive failure during antibiotic use? A. Penicillin B. Rifampin C. Keflex D. Erythromycin E. Tetracycline

B. Rifampin

A dentist is resecting (amputating) a root for periodontal reasons. Which of the following represents the usual treatment sequence? A. Periodontal therapy followed by root canal therapy B. Root canal therapy followed by periodontal therapy C. Root canal therapy followed by a period of observation D. Periodontal therapy followed by a period of observation

B. Root canal therapy followed by periodontal therapy

To evaluate drugs and medical devices, the Food and Drug Administration uses which of the following criteria? A. Cost-effectiveness B. Safety and efficacy C. Reliability and validity

B. Safety and efficacy

During which trimester of pregnancy does calcification of teeth begin? A. First B. Second C. Third

B. Second

An individual has just received the first in a series of three hepatitis B vaccines. For the seroconversion of this series to be the most effective, the person should receive the second and third vaccinations how many months later? A. Second vaccination one month later, third vaccination three months later B. Second vaccination one month later, third vaccination six months later C. Second vaccination two months later, third vaccination three months later D. Second vaccination three months later, third vaccination six months later

B. Second vaccination one month later, third vaccination six months later

In performing normal dental diagnostic procedures, the operator receives the greatest hazard from which type of radiation? A. Direct primary-beam B. Secondary and scatter C. Gamma

B. Secondary and scatter

A patient of record calls late Saturday night because of severe, throbbing pain, aggravated by "heat, biting, and touching" in a mandibular premolar. What procedure is recommended? A. Instruct the patient to apply ice intermittently, take aspirin, and call Monday for an appointment. B. See the patient at the office and initiate endodontic treatment. C. See the patient at the office, remove the carious dentin and place sedative zinc oxide-eugenol cement. D. Prescribe an analgesic and refer the patient to an endodontist. E. Refer the patient to the hospital oral surgery department for extraction.

B. See the patient at the office and initiate endodontic treatment.

Which of the following flap designs allows the best surgical access to the apical aspect of a tooth root with the least reflection of soft tissue? A. Envelope B. Semilunar C. Vertical release D. Rotation pedicle

B. Semilunar

Which of the following represents the primary symptom to demonstrate that midazolam (Versed®) has been injected intra- arterially rather than intravenously? A. Severe cardiac arrhythmia B. Severe pain in the extremity C. Rapid decrease in blood pressure D. Rapid decrease in respiration rate E. Exaggerated sedation due to the more direct effect of the drug

B. Severe pain in the extremity

Bilateral enlargement of the parotid glands characterizes each of the following conditions EXCEPT one. Which one is the EXCEPTION? A. Malnutrition B. Sialolithiasis C. Sjogren's syndrome D. Benign lymphoepithelial lesion E. Acute epidemic parotitis

B. Sialolithiasis

Which of the following properties of a gold alloy exceeds a base-metal alloy in numerical value? A. Hardness B. Specific gravity C. Casting shrinkage D. Fusion temperature E. Modulus of elasticity

B. Specific gravity

A maxillary central incisor of an adult patient is traumatized in an accident. The tooth is slightly tender to percussion, is in good alignment, and responds normally to pulp vitality tests. Radiographic examination shows a horizontal fracture of the apical third of the root. The best treatment is which of the following? A. Root canal treatment B. Splint and re-evaluate the tooth for pulpal vitality at a later time C. Apexification D. Apicoectomy to remove the fractured apical section of the root followed by root canal treatment

B. Splint and re-evaluate the tooth for pulpal vitality at a later time

A patient that is having general anesthesia using an IV barbiturate rapidly passes through A. Stage I. B. Stage II. C. Stage III. D. Stage IV.

B. Stage II.

Which of the following represents the variability about the mean-value of a group of observations? A. Sensitivity B. Standard deviation C. t-Statistic D. Specificity

B. Standard deviation

Each of the following is a characteristic of tramadol (Ultram®) EXCEPT one. Which one is the EXCEPTION? A. Centrally acting analgesic B. Structurally similar to morphine C. Binds to the mu-opioid receptor D. Biotransformed into a more active metabolite E. Inhibits uptake of norepinephrine and serotonin

B. Structurally similar to morphine

Each of the following is a part of the initial preparation of a periodontal treatment plan EXCEPT one. Which one is this EXCEPTION? A. Root planing B. Surgical pocket elimination C. Home-care instructions D. Occlusal adjustment E. Extractions of hopeless teeth

B. Surgical pocket elimination

A patient has a skeletal deformity with a Class III malocclusion. This deformity is the result of a maxillary deficiency. The treatment-of -choice is A. Orthodontics. B. Surgical repositioning of the maxilla. C. Anterior maxillary osteotomy. D. Posterior maxillary osteotomy. E. Surgical repositioning of the mandible.

B. Surgical repositioning of the maxilla.

Which of the following safety techniques provides the GREATEST DECREASE in overall radiation-risk to patients? A. Changing from Group D to Group E film B. Switching from round to rectangular collimation C. Using an automatic rather than manual processing switch D. Adding a cervical collar to a leaded apron

B. Switching from round to rectangular collimation

Which of the following best describes sweat gland innervation? A. Sympathetic--adrenergic B. Sympathetic--cholinergic C. Parasympathetic--adrenergic D. Parasympathetic--cholinergic E. Adrenergic--cholinergic

B. Sympathetic--cholinergic

Each of the following drugs produces vasoconstriction of vessels if injected into the gingiva EXCEPT one. Which one is this EXCEPTION? A. Epinephrine (EpiPen®) B. Terazosin (Hytrin®) C. Levonordefrin (Neo-Nedfrin®) D. Phenylephrine (Neo-Synephrine®) E. Norepinephrine (Levophed®),

B. Terazosin (Hytrin®)

When a patient opens his mouth, there is an audible click. Which of the following represents the MOST plausible explanation? A. The joint capsule has thickened. B. The disk and condyle lack functional coordination. C. The articular eminence has flattened. D. The medial pterygoid muscle is in spasm.

B. The disk and condyle lack functional coordination.

Each of the following statements concerning the administration technique of inhalation sedation is correct, EXCEPT one. Which one is the EXCEPTION? A. If the patient is wearing contact lenses, they should be removed prior to administration of the sedation. B. The patient should be seated in an upright (85-90 degree) position in the dental chair . C. The initial flow rate of oxygen should be at least 6 L/min via the nasal hood. D. As the flow of nitrous oxide through a mixing dial style unit is increased, the flow rate of oxygen is reduced, so the correct percentage of each is delivered at a constant flow rate.

B. The patient should be seated in an upright (85-90 degree) position in the dental chair .

Which of the following precautions should the dentist observe when providing dental care for a child with insulin-dependent diabetes? A. No special precautions are necessary. B. The patient should be seen after a meal and insulin injection. C. The patient should have an empty stomach and should take the insulin before the procedure. D. The patient should have an empty stomach and postpone the insulin injection until after the procedure. E. The patient should be seen after a meal and postpone the insulin injection until after the procedure.

B. The patient should be seen after a meal and insulin injection.

Which of the following best explains why the renal excretion of salicylic acid is increased in an alkaline urine? A. The urine volume is decreased. B. The percentage of salicylic acid that is ionized in urine is increased. C. The percentage of salicylic acid that is ionized in urine is decreased. D. Decreasing urine pH usually increases the renal excretion of weak acids.

B. The percentage of salicylic acid that is ionized in urine is increased.

Plaque accumulation has a direct effect on which of the following? A. The level of materia alba B. The severity of gingivitis C. The accumulation of calculus D. The severity of periodontitis

B. The severity of gingivitis

You are doing some preprosthetic surgery on a patient in preparation for a new denture. Which of the following statements about removal of her moderate-sized mandibular lingual tori is correct? A. The tori should be removed using a large pear-shaped acrylic bur. B. The tori should be removed by grooving the superior surface then shearing the torus off with a mono-beveled chisel. The area is then smoothed with a bone file. C. The tori should be removed by grooving the superior surface of the torus with a bur, then shear the torus off with a bibeveled chisel. D. The tori should be removed by inserting a rongeurs vertically under a lingual flap and "snipping off' the tori.

B. The tori should be removed by grooving the superior surface then shearing the torus off with a mono-beveled chisel. The area is then smoothed with a bone file.

Which of the following is the best aid in the diagnosis of an irreversible pulpitis? A. Electric pulp test B. Thermal test C. Percussion D. Test cavity

B. Thermal test

Which of the following generally describes children with defiant behavior? A. They are usually around 3 years of age or younger. B. They are also referred to as stubborn or spoiled. C. They usually exhibit tremors in voice when speaking. D. They exhibit quiet sobbing with profuse tears.

B. They are also referred to as stubborn or spoiled.

Which of the following describes osseointegrated implants? A. They have a direct structural and functional connection with bone only at the radiographic level of detection. B. They are anchored directly to living bone as determined by radiographic and light microscopic analyses. C. They form a junctional epithelium with the surrounding tissue. D. They form a pseudo-periodontal ligament.

B. They are anchored directly to living bone as determined by radiographic and light microscopic analyses.

What happens to the profiles of a majority of individuals as they mature? A. They become more convex. B. They become less convex. C. They retain the same degree of convexity.

B. They become less convex.

Which of the following tissues is MOST sensitive to radiation-induced carcinogenesis? A. Liver B. Thyroid C. Salivary gland D. Prostate

B. Thyroid

Lesions of recurrent herpetic stomatitis are usually found mainly on A. Loosely attached areas of the oral mucosa, such as the buccal mucosa and the floor of the mouth. B. Tightly attached areas of the oral mucosa, such as the hard palate and the attached gingiva. C. Loosely attached areas of the oral mucosa, such as the soft palate and the alveolar mucosa. D. Tissue anywhere in the oral mucosa, including both loosely- and tightly-attached areas.

B. Tightly attached areas of the oral mucosa, such as the hard palate and the attached gingiva.

Each of the following, EXCEPT one, is a good reason for using sedation. Which one is this EXCEPTION? A. To allay apprehension, anxiety or fear B. To decrease the amount of local anesthesia that is required for a given procedure C. To alleviate stress in a severely medically compromised patient D. To accomplish certain procedures that a practitioner would not normally be able to do on an anxious patient

B. To decrease the amount of local anesthesia that is required for a given procedure

Which of the following represents the main objective of the maintenance phase of periodontal therapy? A. To maintain rapport with the patient B. To prevent the recurrence of disease C. To perform touch-up of secondary surgical procedures D. To complete all remaining aspects of the treatment plan E. To re-evaluate the results of initial therapeutic effects

B. To prevent the recurrence of disease

Which of the following properties of sodium hypochlorite is the most undesirable? A. Smell B. Toxicity to vital tissue C. Tendency to bleach dentin D. Corrosive action on endodontic files E. Reaction with chelating agents

B. Toxicity to vital tissue

Which of the following outlines best describes the access cavity preparation on a mandibular molar with four canals? A. Oval B. Trapezoidal C. Triangular D. Rectangular

B. Trapezoidal

Which of the following is necessary to allow for apically positioning a flap margin on the palatal surface of molar teeth? A. Use only a sulcular incision for the initial incision B. Trim the flap margin to the proper length during the procedure C. Avoid making the initial incision deep enough to reach bone D. Extend vertical releasing incision beyond the mucogingival junction

B. Trim the flap margin to the proper length during the procedure

Use of infection control procedures with all patients, rather than with selected patients, is properly referred to as A. OSHA requirements. B. Universal precautions. C. ADA guidelines. D. CDC recommendations.

B. Universal precautions.

A dental office employee wishes to verify that instruments have become sterilized. To obtain accurate results, this individual should do which of the following? A. Place indicator tape on the instruments B. Use biological monitors C. Examine the sterilizer packages for color changes D. Observe the indicator light on the sterilizing unit

B. Use biological monitors

Individuals who are cognitively disabled (e.g. those who have Down syndrome) sometimes have difficulty accepting dental care. In attempting to improve their cooperation, the dentist should do which of the following as an initial therapy? A. Administer sedative agents B. Use graduated exposures to the dental setting C. Use physical restraints D. Refer the patient to a special care clinic

B. Use graduated exposures to the dental setting

Which of the following is the LEAST effective for enhancing orthodontic anchorage? A. Use of extraoral force B. Use of .018 round wire in a .022 rectangular slot C. Attachment to a greater number of teeth D. Wider force distribution in the periodontal ligament

B. Use of .018 round wire in a .022 rectangular slot

Through the Bloodborne Pathogen Standard, the Occupational Safety and Health Administration (OSHA) directs activity for each of the following EXCEPT one. Which one is this EXCEPTION? A. Using barrier techniques B. Using material safety data sheets (MSDS) C. Obtaining hepatitis B vaccinations D. Communicating hazards to employees E. Performing housekeeping

B. Using material safety data sheets (MSDS)

An evaluation of which of the following represents the most important aspect in shade selection (for the restoration to match an existing dentition)? A. Hue B. Value C. Chroma D. Size E. Shape

B. Value

Which of the following cannot be used to calculate the dosage of a drug for a child? A. Clark's rule B. Vital signs C. Body surface area D. Body weight (mg/kg)

B. Vital signs

Which of the following is the least reliable finding from a clinical examination of teeth subjected to traumatic injury? A. Mobility of teeth B. Vitality of teeth C. Displacement of teeth D. Loss of tooth structure

B. Vitality of teeth

The Centers for Disease Control (CDC) recommends that sterilizing units be monitored at which of the following intervals? A. Daily B. Weekly C. Bi-weekly D. Monthly

B. Weekly

What determines the maximum dose of local anesthesia for a child? A. Age B. Weight C. The procedure to be accomplished D. The desired degree of pulpal anesthesia

B. Weight

A member of the dental staff routinely fails to clean up his work area. Which of the following best exemplifies an assertive message that could be made by a co-worker? A. If you refuse to do your part, I'm just going to have to report the situation to the boss. B. When you don't help with the clean-up, it makes me angry because I have to do both your work and my own. C. I'm tired of cleaning up your mess! D. I hope you'll remember to help me clean up today. E. Is something bothering you? You sometimes forget to clean up.

B. When you don't help with the clean-up, it makes me angry because I have to do both your work and my own.

A male patient who is receiving Coumadin® therapy presents for an elective extraction. His prothrombin time (PT) is prolonged. Which of the following methods is preferred for reducing the PT to an acceptable level? A. Administering vitamin K (Aqua Mephyton®) B. Withdrawing Coumadin® for two days C. Reducing Coumadin® to one-half the usual dose for two days D. Administering a Coumadin® antagonist, such as heparin E. Administering a platelet transfusion to enhance coagulability

B. Withdrawing Coumadin® for two days

The relationship of methacholine to acetylcholine is as A. a vehicle. B. a congener. C. an isomer. D. an adjuvant. E. an antagonist.

B. a congener.

When high gold content alloys are compared to base metal alloys, the base metal alloys exhibit A. a higher melting point, increased specific gravity, and generally higher yield strength and hardness. B. a higher melting point, decreased specific gravity, and generally higher yield strength and hardness. C. a higher melting point, generally higher yield strength and hardness, and more consistent bonding to porcelain. D. increased specific gravity, generally lower yield strength, and more consistent bonding to porcelain. E. decreased specific gravity, generally higher yield strength and hardness, and more consistent bonding to porcelain.

B. a higher melting point, decreased specific gravity, and generally higher yield strength and hardness.

After obturation of a traumatized central incisor, a horizontal line of material in the midroot area extending both mesially and distally from the canal to the periodontal ligament space is noted. This indicates the presence of A. accessory canals. B. a midroot fracture. C. internal resorption. D. external resorption.

B. a midroot fracture.

Compared to codeine, morphine is A. safer. B. a more potent analgesic. C. a more efficacious analgesic. D. ineffective when administered orally.

B. a more potent analgesic.

The most common cause of fracture at the isthmus of a Class II restoration is A. delayed amalgam expansion. B. a sharp axio-pulpal line angle. C. isthmus width too narrow. D. moisture contamination of the amalgam during placement. E. lack of interproximal contact.

B. a sharp axio-pulpal line angle.

The onset of action of a drug is primarily determined by the rate of A. excretion. B. absorption. C. distribution. D. biotransformation.

B. absorption.

A 35-year-old patient who has an acute suppurative pulpitis is scheduled for tooth extraction. Due to this patient's history of rheumatic heart disease, the practitioner administered penicillin chemoprophylaxis. Fifteen minutes later, this practitioner notices that the periorbital tissue and the lips of the patient are edematous. These findings are consistent with a diagnosis of A. hematoma formation. B. angioneurotic edema. C. congestive heart failure. D. an acute anaphylactoid reaction.

B. angioneurotic edema.

Replacement resorption is characterized by A. pain. B. ankylosis. C. apical pathosis. D. rapid progression. E. acute inflammation.

B. ankylosis.

In removable partial denture design, guiding planes serve to A. aid in balancing occlusion. B. assure predictable clasp retention. C. form right angles to the occlusal plane. D. eliminate the necessity for precision attachments. E. eliminate the necessity for a back-action clasp.

B. assure predictable clasp retention.

Atrophine A. stimulates sweat and saliva secretion. B. at moderate to high doses usually increases heart rate. C. relieves ptosis in myasthenia gravis D. is devoid of central nervous system effects because it does not cross the blood-brain barrier.

B. at moderate to high doses usually increases heart rate.

When finishing the occlusal portion of a posterior composite restoration, the dentist should carefully A. eliminate contacts in the fossae. B. avoid altering the centric contact on enamel. C. develop centric contacts on cavosurface margins. D. leave a 1 mm protective "fin" of material over the cavosurface margin.

B. avoid altering the centric contact on enamel.

Antibiotics are obtained from growth of A. fungi and viruses. B. bacteria and fungi. C. tissue cell cultures. D. viruses and mycoplasmas. E. mycoplasmas and rickettsiae.

B. bacteria and fungi.

A patient, who has been taking penicillin for three weeks, presents with slightly painful white lesions on the tongue, palate, and cheeks. A dentist wipes the lesions with gauze and notes that they have red surfaces. The most likely diagnosis is A. lichen planus. B. candidiasis. C. erythema multiforme. D. primary herpetic gingivostomatitis.

B. candidiasis.

When surgically entering the superior joint space, one must incise through the A. discal ligament only. B. capsular ligament only. C. capsular and discal ligaments. D. capsular and retrodiscal lamina.

B. capsular ligament only.

Adrenal insufficiency during major stress results in A. gingival hyperplasia. B. cardiovascular collapse. C. hypotension. D. ketoacidosis.

B. cardiovascular collapse.

The nerve plexus of Rashkow is located in the A. cell-rich layer. B. cell-free layer. C. central pulp core. D. odontoblast layer.

B. cell-free layer.

Before deciding whether to use a general anesthetic for surgical procedures, the two preoperative studies that should be considered first are A. bleeding time and clotting time. B. complete blood count and urinalysis. C. hemoglobin level and prothrombin time. D. complete blood count and bleeding time. E. total white blood cell count and prothrombin time.

B. complete blood count and urinalysis.

The literature suggests that the majority of vertical root fractures of endodontically treated teeth result from A. traumatic occlusion. B. condensation forces during gutta-percha fill. C. locking temporaries into prepared teeth. D. cementing the cast post and core. E. permanent cementing of crowns.

B. condensation forces during gutta-percha fill.

A patient who is receiving an I.V. diazepam sedation has upper-eyelid ptosis (Verrill's sign). The dentist should A. assist respiration immediately. B. consider the patient to be adequately sedated. C. place the patient in the Trendelenburg position. D. administer one more increment of diazepam and proceed with the treatment.

B. consider the patient to be adequately sedated.

A dentist adjusts the shade of a restoration using a complementary color. This procedure will result in A. increased value. B. decreased value. C. intensified color. D. increased translucency.

B. decreased value.

When a patient bites on a hard object on the left mandibular molar, the interarticular pressure of the right temporomandibular joint is A. increased. B. decreased. C. remains unchanged. D. varied but it is unpredictable.

B. decreased.

The depth of light penetration into the tooth or restoration before it is reflected outward is the A. metamerism. B. degree of translucency. C. color perception. D. value.

B. degree of translucency.

Dental elevators are single blade instruments that are A. used to retract the crestal gingiva prior to the application of the dental forceps. B. designed to engage the tooth apical to the cementoenamel junction. C. designed to engage the tooth coronal to the cementoenamel junction. D. used to reflect a full thickness mucoperiosteal flap prior to the application of the dental forceps.

B. designed to engage the tooth apical to the cementoenamel junction.

Dental plaque is believed to adhere to teeth because A. levans are soluble and sticky. B. dextrans are insoluble and sticky. C. the surface tension of plaque is low. D. bacteria secrete a lipoprotein-cementing substance.

B. dextrans are insoluble and sticky.

Water irrigation devices in oral health regimens are useful in A. eliminating plaque. B. diluting bacterial products. C. reducing periodontal pockets. D. eliminating gingival inflammation.

B. diluting bacterial products.

During a soldering procedure, flux serves to A. provide an oxidizing environment in the area to be soldered. B. displace gases and dissolve corrosion products. C. remove any debris that may remain in the area to be soldered. D. hold the solder in place during heating.

B. displace gases and dissolve corrosion products.

When communicating with a patient, the dentist can express involvement and interest primarily with A. body orientation. B. eye contact. C. verbal agreement. D. positive reinforcement.

B. eye contact.

Non-working interferences usually occur on the inner aspects of the A. facial cusps of maxillary molars. B. facial cusps of mandibular molars. C. lingual cusps of mandibular molars. D. facial cusps of maxillary premolars.

B. facial cusps of mandibular molars.

Alveoplasty following closed extraction is usually performed with A. a bur. B. fingers. C. a rongeur. D. a bone file.

B. fingers.

To obtain the best results when using an elastomeric impression material, the dentist must ensure that the prepared tooth is A. very dry. B. free of surface moisture. C. covered thinly with a water soluble lubricant. D. covered with a surface tension-reducing agent and air dried. E. lubricated with a petroleum-free gel exhibiting minimal film thickness and low viscosity.

B. free of surface moisture.

To prove its clinical effectiveness, an antimicrobial agent must demonstrate that it A. reduces plaque mass. B. helps to reduce disease. C. specifically kills aerobic bacteria. D. specifically kills anaerobic bacteria.

B. helps to reduce disease.

A patient who has exophthalmos may be diagnosed as having A. hyperglycemia. B. hyperthyroidism. C. hyperadrenalism. D. hyperparathyroidism.

B. hyperthyroidism.

The thinnest portion of the wax pattern should be placed A. against the ring for support. B. in the deepest part of the ring. C. opposite the direction of rotation of the casting arm. D. in the same direction as the rotation of the casting arm.

B. in the deepest part of the ring.

A patient presents with an amalgam restoration fractured at the isthmus six months after placement. The most likely cause is A. recurrent caries. B. inadequate depth of the preparation. C. excessive width of the preparation. D. premature occlusal contact.

B. inadequate depth of the preparation.

In radiography, a longer gray scale of contrast can be achieved by A. decreasing the filtration. B. increasing the kilovoltage. C. increasing the milliamperage. D. increasing the focal spot-skin distance.

B. increasing the kilovoltage.

A patient has pain over the left pre-auricular area; this patient can open approximately 45 mm and has a "pop-and-click" in the joint area. The most likely diagnosis is A. myofascial pain dysfunction syndrome. B. internal derangement with reduction. C. auriculotemporal syndrome. D. coronoid hyperplasia.

B. internal derangement with reduction.

Following immediate denture surgery, the best suture technique to use would be A. continuous. B. interrupted. C. vertical mattress. D. never use sutures. E. horizontal mattress.

B. interrupted.

A patient receives a heavy blow to the chin. On clinical examination, the mandible deviates to the left side on opening. The most likely site of the fracture is the A. right subcondylar region. B. left subcondylar region. C. right body of the mandible. D. left body of the mandible.

B. left subcondylar region.

A tender swelling in the soft tissue of the submandibular triangle is most likely caused by A. lipoma. B. lymphadenopathy. C. an obstruction of Stensen's duct. D. n infected thyroglossal duct cyst.

B. lymphadenopathy.

The mechanism of action of the muscle relaxation induced by diazepam most clearly resembles that produced by A. d-tubocurarine. B. meprobamate. C. succinylcholine. D. decamethonium. E. gallamine.

B. meprobamate.

A cast gold restoration might be indicated for the replacement of a faulty amalgam to obtain A. better margins. B. more ideal contours. C. less trauma to the pulp. D. less removal of tooth structure.

B. more ideal contours.

When compared therapeutically to penicillin G, penicillin V has a A. slower renal excretion. B. more reliable oral absorption. C. broader antibacterial spectrum. D. greater resistance to penicillinase. E. lower potential for allergic reaction.

B. more reliable oral absorption.

The majority of patients with "temporomandibular joint pain/dysfunction syndrome" have A. normal dentitions. B. no obvious TMJ pathology on panorex. C. physical findings of rheumatoid arthritis in other joints. D. evidence of destruction of cortical bone of the condylar head on the affected side.

B. no obvious TMJ pathology on panorex.

Incorrect horizontal angulation of the x-ray tubehead during exposure of a molar bite-wing leads to A. elongation of teeth. B. overlapping of interproximals C. overexposure of film D. foreshortening of roots. E. foreshortening of crowns.

B. overlapping of interproximals

A young man, who is a hemophiliac, has a maxillary first molar with a necrotic pulp. This tooth also has a radiolucent lesion at the palatal root apex. The dentist has obtained approval to proceed with the treatment from the patient's physician. The best course of treatment is to A. extract the tooth. B. perform a conventional root canal treatment. C. perform a conventional root canal treatment in conjunction with systemic vitamin K therapy. D. instrument and obturate the canals to midroot level in order to avoid vascular trauma. E. seal formocresol in the pulp chamber in order to fix the necrotic material in the canals.

B. perform a conventional root canal treatment.

To increase the anterior mandibular vestibular depth, the practitioner places the mucosal graft directly on the A. mucosa. B. periosteum. C. muscle. D. bone.

B. periosteum.

Repeated abuse of nitrous oxide results in A. systemic myopathy. B. peripheral neuropathy. C. thrombocytopenia. D. acute narrow angle glaucoma.

B. peripheral neuropathy.

A composite restoration is wider than the diameter of the light tip of the curing unit. In this situation, the restoration is cured by A. moving the tip over the surface for the required time. B. placing the tip stepwise over each area and exposing each area for the required time. C. positioning the tip far enough from the surface to illuminate the entire surface. D. centering the tip on the surface and curing the entire restoration from this position.

B. placing the tip stepwise over each area and exposing each area for the required time.

A 43-year-old patient who has mitral stenosis, secondary to rheumatic fever, requests extraction of two periodontally involved mandibular teeth. Initially, the dentist should A. premedicate the patient with cephalosporin. B. premedicate the patient with amoxicillin. C. premedicate the patient with ampicillin and gentamicin. D. consult with the patient's physician to determine the antibiotic-of-choice.

B. premedicate the patient with amoxicillin.

A 7-year-old patient fractured the right central incisor three hours ago. A clinical examination reveals a 2-mm exposure of a "bleeding pulp." The treatment-of-choice is A. pulpectomy and apexification. B. pulpotomy with calcium hydroxide. C. direct pulp cap with calcium hydroxide. D. one-appointment root canal treatment.

B. pulpotomy with calcium hydroxide.

Dental patients who are anxious or fearful have a tendency to A. become phobic when things don't go well. B. put off making appointments. C. be interested in prevention.

B. put off making appointments.

A 33-year-old female patient states that her mandibular first molar has been hurting since the recent placement of an amalgam restoration. She describes. the pain as mild-to-moderate, which is not spontaneous, but is provoked by cold, heat, and sweets. These symptoms most likely correspond with A. pulp necrosis. B. reversible pulpitis. C. internal resorption. D. irreversible pulpitis.

B. reversible pulpitis.

The beak of an extraction forceps is designed so that most of the extraction pressure is transmitted to the A. alveolar bone. B. root of the tooth. C. crown of the tooth. D. cementoenamel junction.

B. root of the tooth.

For MOST effective cutting and long usefulness of a tungsten carbide bur, it should be A. rotating slowly before contacting the tooth. B. rotating rapidly before contacting the tooth. C. placed in contact with the tooth before starting.

B. rotating rapidly before contacting the tooth.

In removing four or more adjacent teeth, the dentist removes the inter-radicular bone and compresses the alveolar plates. This is a method of treating a A. severe exostosis. B. severe labial undercut. C. mandibular protrusion. D. posterior vertical hyperplasia. E. severe maxillary protrusion.

B. severe labial undercut.

On the basis of diagnostic test results, a dentist correctly classifies a group of patients as being free from disease. These results possess high A. sensitivity. B. specificity. C. generalizability. D. repeatability.

B. specificity.

Electron microscopic examination of the bacterial flora of necrotizing ulcerative gingivitis indicates the presence of microorganisms within non-necrotic tissues in advance of other bacteria. The organisms involved are A. cocci. B. spirochetes. C. bacteriophages. D. filamentous rods.

B. spirochetes.

The only joint structure that exerts posterior traction on the articular disc is the A. inferior retrodiscal lamina. B. superior retrodiscal lamina. C. inferior head of the lateral pterygoid. D. superior head of the lateral pterygoid.

B. superior retrodiscal lamina.

A removable partial denture made of a base metal alloy is resistant to tarnish and corrosion because of its A. high hardness. B. surface oxide layer. C. noble metal content. D. grain structure.

B. surface oxide layer.

The character and individuality of teeth are largely determined by A. translucency. B. surface texture. C. color. D. shape.

B. surface texture.

Dentists usually send their metal ceramic restoration cases to dental laboratory technicians for fabrication. Upon receiving the cases, these technicians most frequently complain that the A. margins have been ill-defined. B. teeth have been insufficiently reduced. C. shades for the restorations have been inadequately described. D. impressions or models are inaccurate.

B. teeth have been insufficiently reduced.

The optimal concentration of fluoride for community water depends upon the A. proportion of residents who are children. B. temperature of the air. C. caries rate of the total population.

B. temperature of the air.

Speech problems associated with cleft lip and palate are usually the result of A. poor tongue control that produces lisping. B. the inability of soft palate to close air flow into the nasal area. C. the inability of the tongue to close air flow from the epiglottis. D. missing teeth that make formation of articulation sounds by the tongue difficult. E. poor lip musculature or heavy scars in the lips that limit vowel sound production.

B. the inability of soft palate to close air flow into the nasal area.

When a nonrigid connector is used in a fixed partial denture, the path of insertion of the key into the keyway should be parallel to the path(s) of insertion of A. the retainer carrying the keyway. B. the retainer not involved with the keyway. C. both retainers.

B. the retainer not involved with the keyway.

Group function occlusion is characterized by having A. non-working contacts. B. working contacts. C. protrusive contacts. D. a long centric.

B. working contacts.

The apical foramen most often exits the root A. at the anatomic apex B. less than 0.5 mm from the anatomic apex C. 0.5-1.0 mm from the anatomic apex D. more than 1 mm from the anatomic apex

C. 0.5-1.0 mm from the anatomic apex

According to the American Heart Association, which of the following prophylactic antibiotic regimens is recommended for a 20-kg child who has congenital heart disease? A. 1.0 gram amoxicillin orally one hour before the dental procedure and 500 mg 4 times a day for 2 days postoperatively B. 1.0 gram penicillin V orally one hour before the dental procedure and 500 mg orally 6 hours later C. 1.0 gram amoxicillin one hour before the dental procedure and 500 mg orally 6 hours later D. 3.0 grams amoxicillin orally one hour before the dental procedure and 1.5 grams orally 6 hours later

C. 1.0 gram amoxicillin one hour before the dental procedure and 500 mg orally 6 hours later

A tablespoon contains how many ml? A. 5 B. 10 C. 15 D. 20

C. 15

Cavity liners should have a minimal thickness of A. 1 mm. B. 1.5 mm. C. 15 mm. D. 125 mm.

C. 15 mm.

The cusps to be restored with dental amalgam should be reduced by A. 1 mm while forming a flattened surface. B. 1 mm while following the original contour of the cusps. C. 2 mm while forming a flattened surface. D. 2 mm while following the original contour of the cusps.

C. 2 mm while forming a flattened surface.

Which of the following represents the optimal incisal reduction for a metal-ceramic crown preparation? A. 1.0 mm B. 1.5 mm C. 2.0 mm D. 2.5 mm E. 3.0 mm

C. 2.0 mm

The casting shrinkage of cobalt-chromium alloys is approximately A. 1.25%. B. 1.75%. C. 2.2%. D. 3.2%.

C. 2.2%.

When a permanent tooth clinically emerges, how much of the root structure is most likely to have developed? A. 1/4 B. 1/3 C. 2/3 D. 4/5

C. 2/3

A dentist is restoring an endodontically treated posterior tooth with a pulp chamber-retained amalgam restoration. Ideally, the dentist should place the amalgam to what depth into each root canal to obtain satisfactory retention? A. 1 mm B. 2 mm C. 3 mm D. 4 mm

C. 3 mm

Debridement of an intraoral wound is accomplished with A. a betadine solution. B. 1% tincture of iodine. C. 3% hydrogen peroxide. D. 10% hydrogen peroxide.

C. 3% hydrogen peroxide.

An analgesic compound #3 contains how many mg of codeine? A. 7.5 B. 15 C. 30 E. 60

C. 30

The number of persons who die each year from oral cancer in the United States approximates A. 2000 B. 4000 C. 8000 D. 16000

C. 8000

Which of the following is the LEAST likely to cause rampant caries in geriatric patients? A. Poor oral hygiene B. A decreased salivary flow C. A change in the oral microflora D. The side effects of medication

C. A change in the oral microflora

A bevel is CONTRAINDICATED on the cavosurface angles of a Class I dental amalgam cavity preparation. Which of the following best explains why? A. This type of margin is prone to microleakage. B. The cavosurface bevel makes burnishing more difficult. C. A thin flange of the amalgam restorative material might fracture. D. As the tooth undergoes natural attrition, the amalgam margin can abrade.

C. A thin flange of the amalgam restorative material might fracture.

The best measure of the potential clinical performance of a casting alloy is its A. castability. B. burnishability. C. ADA certification. D. tarnish susceptibility. E. mechanical properties.

C. ADA certification.

Each of the following drugs has a significant anti-inflammatory property EXCEPT one. Which one is this EXCEPTION? A. Aspirin B. Cortisol C. Acetaminophen D. Ibuprofen (Motrin®) E. Indomethacin (Indocin®)

C. Acetaminophen

Each of the following is known to contribute to orofacial clefting EXCEPT one. Which one is the EXCEPTION? A. Valium® B. Poor diet C. Acetaminophen D. Vitamin deficiency or excess

C. Acetaminophen

Which of the following lesions has the greatest malignant potential? A. Leukoedema B. Lichen planus C. Actinic cheilitis D. White sponge nevus

C. Actinic cheilitis

Which of the following is the most likely postoperative complication of bleaching a tooth that has not been adequately obturated? A. Fracture B. Discoloration C. Acute apical periodontitis D. Chronic apical periodontitis E. External cervical root resorption

C. Acute apical periodontitis

Which of the following instruments is used to hold soft tissue which will not be removed from the body? A. Hemostat B. Allis forceps C. Adson forceps D. Drain inserter

C. Adson forceps

A patient presents with spontaneous necrotizing ulcers of the oral cavity. The WBC is 1,986, while the differential CBC reveals lymphocytes--68%, monocytes--28%, polymorphonuclear leukocytes--2%, eosinophils--l%, and basophils--l%. Which of the following diagnoses is the MOST appropriate for this patient? A. Leukemia B. Infectious mononucleosis C. Agranulocytosis D. Recurrent aphthous ulcers

C. Agranulocytosis

Each of the following is an electromagnetic radiation EXCEPT one. Which one is the EXCEPTION? A. Radar B. X rays C. Alpha rays D. Gamma rays E. Visible light

C. Alpha rays

When 50 mg. of chlorpromazine (Thorazine®) is administered to a patient, on standing the patient might experience a fall in blood pressure due to which of the following? A. Anticholinergic action B. Decrease in heart rate C. Alpha-adrenergic blockade D. Negative inotropic action E. Stimulation of autonomic ganglia

C. Alpha-adrenergic blockade

All-ceramic crowns should NOT be used for patients who are associated with each of the following conditions EXCEPT one. Which one is the EXCEPTION? A. Severe bruxism B. Previous history of ceramic crown fractures C. Amalgam restorations with marginal breakdown D. Extensive wear of tooth structure or restorations

C. Amalgam restorations with marginal breakdown

Which of the following exhibits neoplasia of both the epithelial and connective tissue components? A. Ameloblastoma B. Cementoblastoma C. Ameloblastic fibroma D. Odontogenic adenomatoid tumor

C. Ameloblastic fibroma

Which of the following drugs induced the first dramatic remission in acute leukemia in children? A. Mechlorethamine B. Carmustine C. Aminopterin D. Streptozocin E. Mercaptopurine

C. Aminopterin

Which of the following factors is the most critical in the determination of prognosis of a tooth with periodontal disease? A. Probing depth B. Mobility of the tooth C. Amount of attachment loss D. Presence of furcation involvement

C. Amount of attachment loss

A patient presents with slight chipping of the enamel along the incisal edges of Teeth #8 and #9. When choosing between restoring the incisal edges or reshaping by selective grinding, which of the following important factors should the dentist consider? A. Location of proximal contacts B. Shape of incisal embrasures C. Amount of translucent enamel present

C. Amount of translucent enamel present

Which of the following direct retainers for a removable partial denture is the MOST esthetic? A. A bar clasp B. A circumferential clasp C. An intracoronal attachment D. A round, wrought-wire clasp

C. An intracoronal attachment

Loss of intermaxillary space, infection, or avitaminosis B can each cause which of the following? A. Atrophic glossitis B. Xerostomia C. Angular cheilitis D. Recurrent aphthae E. Periadenitis mucosa necrotica recurrens

C. Angular cheilitis

Which of the following planes of space are used to classify malocclusion? A. Horizontal, vertical, diagonal B. Antero-posterior, sagittal, vertical C. Antero-posterior, transverse, vertical D. Antero-posterior, sagittal, coronal

C. Antero-posterior, transverse, vertical

To prevent cross-contamination to patients from dental operatory units, a dentist should use handpieces and water spray hoses that are fitted with which type of valves? A. Retraction B. Pressure C. Anti-retraction D. Depressurization

C. Anti-retraction

Which of the following groups of drugs is CONTRAINDICATED for patients who have glaucoma? A. Adrenergic B. Cholinergic C. Anticholinergic D. Adrenergic blocking

C. Anticholinergic

Which of the following groups of drugs is contraindicated for patients who have glaucoma? A. Adrenergic B. Cholinergic C. Anticholinergic D. Adrenergic blocking

C. Anticholinergic

When compared to other materials, which of the following is the main disadvantage of using polyether elastomeric impression materials? A. Tear easily B. Stick to the teeth C. Are much stiffer D. Are not as accurate E. Have longer working time

C. Are much stiffer

The greatest risk of oral cancer is in persons who A. Have poor oral hygiene and nutritional deficiencies; and, frequently use alcohol. B. Have poor oral hygiene and nutritional deficiencies; and, regularly use tobacco. C. Are older, frequently use alcohol, and regularly use tobacco. D. Are older, have nutritional deficiencies, and have poor oral hygiene.

C. Are older, frequently use alcohol, and regularly use tobacco.

Which of the following is the most common surgical procedure for treating chronic recurrent dislocation of the temporomandibular joint? A. Total joint replacement B. Corrective orthognathic surgery C. Articular eminectomy, usually combined with capsular plication D. An interpositional arthroplasty, often combined with a coronoidectomy E. Coronoidectomy and release of temporalis muscle from the mandible

C. Articular eminectomy, usually combined with capsular plication

Which of the following is a beta-adrenergic receptor blocking agent used for the treatment of hypertension? A. Prazosin (Minipress) B. Clonidine (Catapres) C. Atenolol (Tenormin) D. Hydralazine (Aprezoline) E. Verapamil (Calan)

C. Atenolol (Tenormin)

Which of the following is a beta-adrenergic receptor blocking agent used for the treatment of hypertension? A. Prazosin (Minipress®) B. Clonidine (Catapres®) C. Atenolol (Tenormin®) D. Hydralazine (Aprezoline®) E. Verapamil (Calan®)

C. Atenolol (Tenormin®)

In selecting an impression tray for a preliminary maxillary impression, how far should the tray extend posteriorly? A. To the tuberosity B. To the fovea palatine C. Beyond the vibrating line D. To the pterygomaxillary notches

C. Beyond the vibrating line

A patient with a posterior ridge relationship of a bilateral crossbite will most likely require that the posterior denture teeth be set in which of the following relationships? A. Class I normal horizontal overlap B. Unilateral crossbite C. Bilateral crossbite D. Class II occlusion

C. Bilateral crossbite

Accepted methods for closing a diastema between maxillary central incisors include using either a removable appliance with finger springs or A. A rubber elastic around the two teeth. B. A steel ligature around the two teeth. C. Bonded brackets with intertooth traction.

C. Bonded brackets with intertooth traction.

Tooth bleaching affects a color change in which of the following tooth parts? A. Dentin only B. All of the enamel C. Both dentin and enamel D. Only the surface of enamel

C. Both dentin and enamel

Each of the following is an endogenous opioid peptide EXCEPT one. Which one is this EXCEPTION? A. Endorphin B. Dynorphin C. Bradykinin D. Enkephalin

C. Bradykinin

Nystatin (Mycostatin®) is used in the treatment of infections caused by A. Peptostreptococci spp. B. Bacteroides spp. C. Candida albicans. D. Streptococcus mutans. E. Pseudomonas aeruginosa.

C. Candida albicans.

Before extracting a patient's premolar, the dentist administers an inferior alveolar block. Three minutes after receiving this block, the patient develops paralysis of his forehead muscles, of his eyelids, and of the upper and lower lips on the same side of his face. These findings are the most likely to be associated with the diffusion of the anesthetic solution into which of the following? A. Otic ganglion B. Auriculotemporal nerve C. Capsule of the parotid gland D. Ophthalmic division of the trigeminal nerve E. Motor branches of the mandibular nerve supplying the masticatory muscles

C. Capsule of the parotid gland

Which of the following is recommended for treating the pain of tic douloureux (trigeminal neuralgia)? A. Oxycodone B. Ibuprofen C. Carbamazepine D. Hydrocortisone E. Acetylsalicylic acid

C. Carbamazepine

In young patients, stains are usually more prominent in which areas of the teeth? A. Incisal B. Occlusal C. Cervical D. Facial

C. Cervical

Which of the following is the earliest clinical sign of a carious lesion? A. Radiolucency B. Patient sensitivity C. Change in enamel opacity D. Rough surface texture E. Cavitation of enamel

C. Change in enamel opacity

A 35-year-old woman has an acute dentoalveolar abscess. Which of the following conditions will be the most likely to complicate the management of this patient? A. Asthma B. Epilepsy C. Chemotherapy D. Hypertension

C. Chemotherapy

Aplastic anemia is a serious toxic effect associated most frequently with which of the following antibiotics? A. Streptomycin B. Tetracycline C. Chloramphenicol D. Chlortetracycline

C. Chloramphenicol

A patient who has which of the following conditions is most likely to have postoperative bleeding after multiple extractions? A. Angina B. Diabetes C. Cirrhosis D. Rheumatic fever E. Chronic bronchitis

C. Cirrhosis

Which of the following periapical conditions is most often associated with a vital pulp? A. Apical cyst B. Apical scar C. Condensing osteitis D. Chronic apical periodontitis E. Suppurative apical periodontitis

C. Condensing osteitis

An adult healthy patient has a marked indurated swelling. He has a temperature of 100 degrees Fahrenheit and has been in considerable pain for 24 hours. Percussion of the maxillary right central incisor causes discomfort as does palpation at its apex. Pulp vitality tests are negative for this tooth, while adjacent control teeth test within normal limits. A radiograph reveals that the maxillary right central incisor has a deep unbased restoration and a widened apical periodontal ligament space. Which of the following is the best emergency treatment for this patient? A. Extracting the maxillary right central incisor B. Administering an antibiotic and analgesic and initiating root canal therapy when symptoms subside C. Debriding the root canal of the maxillary right central incisor and prescribing antibiotics and analgesics D. Incising and draining the swelling and prescribing antibiotics and analgesics

C. Debriding the root canal of the maxillary right central incisor and prescribing antibiotics and analgesics

Which of the following will result when using a thinner mix of a gypsum-bonded casting investment? A. Produce a smoother casting B. Increase setting expansion C. Decrease setting expansion D. Increase thermal expansion E. Decrease required burn-out temperature

C. Decrease setting expansion

Chroma is that aspect of color that signifies the A. Degree of grayness. B. Degree of translucency. C. Degree of saturation of the hue. D. Combined effect of the hue and value.

C. Degree of saturation of the hue.

The lateral incisor (seen in the radiograph) exemplifies which of the following? A. Fusion B. Gemination C. Dens invaginatus D. Odontodysplasia E. Pulpal dysplasia

C. Dens invaginatus

At birth, the palate is relatively flat; in adults, it is vault-shaped. By which of the following does this change occur? A. Bone resorption in the palatal vault B. Growth of the maxillary sinuses C. Deposition of the alveolar crestal bone D. Bone deposition on the posterior wall of the maxillary tuberosity

C. Deposition of the alveolar crestal bone

Which of the following is the treatment of choice for lidocaine-induced seizures? A. Epinephrine (EpiPen¨) B. Naloxone (Narcan¨) C. Diazepam (Valium¨) D. Flumazenil (Romazicon¨) E. Succinylcholine (Anectine¨)

C. Diazepam (Valium¨)

Which of the following physical signs indicates severe CNS oxygen deprivation? A. Dilated pupils with increased light reflex B. Pinpoint pupils with increased light reflex C. Dilated pupils with an absence of light reflex D. Pinpoint pupils with an absence of light reflex

C. Dilated pupils with an absence of light reflex

Which of the following is the most common cause of intracapsular restraint of mandibular movement? A. Infection B. Ankylosis C. Disc interference disorders

C. Disc interference disorders

Which of the following mechanical properties limits the amount of adjustment of a base metal removable clasp arm? A. Hardness B. Stiffness C. Elongation D. Tensile strength

C. Elongation

Through the Bloodborne Pathogens Standard, the Occupational Safety and Health Administration (OSHA) directs that the uniform clothing of office personnel be laundered at each of the following locations EXCEPT one. Which one is this EXCEPTION? A. Dental office B. Outside laundry C. Employee's home

C. Employee's home

Which of the following instruments should be used to plane the facio-proximal cavosurface margin of a standard Class II preparation on a mandibular molar? A. Straight chisel B. Binangle chisel C. Enamel hatchet D. Bibeveled hatchet

C. Enamel hatchet

How should a dentist treat a patient who has a generalized acute herpetic stomatitis? A. Promptly prescribe 300,000 units of penicillin B. Begin a series of treatments with diluted chickenpox vaccine C. Encourage liquid intake, sustain oral hygiene, and gently debride the mouth D. With bacterial cultures, immediately distinguish from acute necrotizing ulcerative gingivitis

C. Encourage liquid intake, sustain oral hygiene, and gently debride the mouth

Each of the following represents an advantage of oral sedation EXCEPT one. Which one is the EXCEPTION? A. Need for specialized training and equipment B. High incidence and severity of adverse reactions C. Erratic and incomplete absorption of drugs from the GI tract D. Short duration of action

C. Erratic and incomplete absorption of drugs from the GI tract

The facial surface of the posterior mandible often presents limiting factors to proper treatment of periodontal defects. Which of the following is primary among these factors? A. Genial tubercles B. Mentalis muscle C. External oblique ridge D. Internal pterygoid muscle

C. External oblique ridge

Two hours elapsed before a patient with an avulsed tooth was able to see a dentist. The dentist replanted and splinted the tooth, and performed nonsurgical endodontic therapy. Which of the following probably resulted after the treatment? A. Radicular cyst formation B. Condensing osteitis C. External root resorption E. Chronic periapical periodontitis

C. External root resorption

Which of the following is the principal nonverbal cue that two or more people can use to regulate verbal communication? A. Posture B. Movement C. Eye contact D. Body position

C. Eye contact

During the preparation of a Class II cavity, which of the following permanent teeth pulp horns will be the most subject to accidental exposure? A. Distofacial of a maxillary first molar B. Distofacial of a mandibular first molar C. Facial of a mandibular first premolar D. Lingual of a mandibular first premolar

C. Facial of a mandibular first premolar

Which of the following bacteria responsible for odontogenic infections are capable of adapting to either a high or low oxygen-containing environment? A. Enteric organisms B. Obligate anaerobes C. Facultative organisms D. Synergistic organisms E. Microaerophilic organisms

C. Facultative organisms

Each of the following nonsedating antihistamines would be contraindicated in an individual taking cimetidine for heartburn EXCEPT one. Which one is the EXCEPTION? A. Astemizole (Hismanal®) B. Diphenhydramine (Benadryl®) C. Fexofenadine (Allegra®) D. Hydroxyzine (Vistaril®) E. Terfenadine (Seldane®)

C. Fexofenadine (Allegra®)

Each of the following could be ruled out for the lesion distal to the mandibular third molar EXCEPT one. Which one is this EXCEPTION? A. Cementoblastoma B. Osteoma C. Fibroma D. Salivary gland inclusion defect (Stafne defect)

C. Fibroma

Which of the following is most related to the initiation of caries in the elderly? A. Erosion B. Attrition C. Gingival recession D. A defective restoration

C. Gingival recession

The dentist uses a palatal flap to close an oroantral fistula in the area of Tooth #15. This flap receives its principal blood supply from which of the following arteries? A. Facial B. Nasopalatine C. Greater palatine D. Posterior superior alveolar

C. Greater palatine

Which of the following is the most serious toxic effect that is associated with overdose of acetaminophen? A. Hemorrhage B. Renal necrosis C. Hepatic necrosis D. Gastric ulceration E. Respiratory alkalosis

C. Hepatic necrosis

Which of the following best describes the drug-receptor activity of naloxone? A. High affinity, high intrinsic activity B. Low affinity, high intrinsic activity C. High affinity, no intrinsic activity D. No affinity, low intrinsic activity

C. High affinity, no intrinsic activity

While the dentist is preparing a large carious lesion in Tooth #30 for a restoration, a pulp exposure occurs. The patient angrily shouts at the dentist, "You incompetent 'creep'--you're responsible for this problem!"- Of the following possible responses the dentist could make, which one is the most emphatic? A. Calm down, I can still restore your tooth adequately. B. Not when I'm preparing a tooth with caries like you had. C. I can see that you're very upset. You thought the tooth could be restored and now this problem has occurred. D. If you took care of your mouth the way you should, I wouldn't have been close to the pulp. E. I'm sorry this happened, but we must get on with the procedure.

C. I can see that you're very upset. You thought the tooth could be restored and now this problem has occurred.

A patient is experiencing a lancinating, paroxysmal pain. This pain is occurring in the posterior portions of his tongue, tonsil, nasopharynx, and pharynx. This condition MOST likely represents a neuralgia of which of the following cranial nerves? A. V B. VII C. IX D. X E. XII

C. IX

What do Gardner syndrome and cleidocranial dysplasia have in common? A. Intestinal polyps B. Intraoral pigmentation C. Impacted supernumerary teeth D. Osteomas of the skull and jaws

C. Impacted supernumerary teeth

Which of the following is a major disadvantage of immediate complete denture therapy? A. Trauma to extraction sites B. Increased potential for infection C. Impossibility for an anterior try-in D. Excessive resorption of residual ridges

C. Impossibility for an anterior try-in

A patient presents for treatment of a large fluctuant mass in the submandibular space as a result of extension of odontogenic infection. He has a temperature of 38.5 degrees Celsius (101 degrees Fahrenheit). Initially, the dentist should treat this patient with which of the following? A. Salicylate therapy to reduce the temperature B. Alternate application of heat and cold to the area to improve circulation C. Incision and drainage and a culture for antibiotic sensitivity D. Antibiotic therapy to reduce the swelling and infection

C. Incision and drainage and a culture for antibiotic sensitivity

In which of the following categories are ephedrine, tyramine, and amphetamine classified? A. Anticholinesterases B. Alpha-adrenergic blocking agents C. Indirect-acting sympathomimetics D. Direct-acting parasympathomimetics

C. Indirect-acting sympathomimetics

In lateral movements, the non-working condyle moves in what direction? A. Laterally, posteriorly, and superiorly B. Superiorly, posteriorly, and inferiorly C. Inferiorly, anteriorly, and medially D. Inferiorly, anteriorly, and laterally

C. Inferiorly, anteriorly, and medially

A patient presents with continuous, spontaneous pain associated with the maxillary right central incisor. This tooth has a large Class V composite restoration. Cold testing produces lingering pain. There is no sensitivity to percussion or palpation. The most likely diagnosis is A. Necrotic pulp, chronic apical periodontitis. B. Reversible pulpitis, normal periapex. C. Irreversible pulpitis, normal periapex. D. Irreversible pulpitis, acute apical periodontitis.

C. Irreversible pulpitis, normal periapex.

A patient is experiencing a throbbing pain in a specific tooth. This pain is aggravated by heat and relieved by cold. The tooth is sensitive to percussion. The most likely diagnosis is A. Occlusal trauma. B. Periodontal abscess. C. Irreversible pulpitis. D. Hyperemia of the pulp.

C. Irreversible pulpitis.

Each of the following is true regarding drug biotransformation EXCEPT one. Which one is this EXCEPTION? A. The rate may differ significantly in various animal species. B. It primarily occurs in the liver microsomal enzyme system. C. It usually converts a drug to its lipid-soluble, nonionized form. D. It generally involves alterations of the chemical structure of the drug.

C. It usually converts a drug to its lipid-soluble, nonionized form.

Which of the following jaw-relation records should be used for setting both the medial and superior condylar guides on an arcon articulator? A. Intercuspation B. Centric relation C. Lateral interocclusal records D. Protrusive interocclusal records

C. Lateral interocclusal records

A dentist anticipates the possibility of a pulpal exposure of a vital, asymptomatic tooth during a cavity preparation. In this situation, what should the dentist do with the carious material? A. Remove the carious material laterally first and then remove completely from the deeper areas of the cavity B. Remove the carious material completely and then treat the tooth endodontically C. Leave the carious material in the deeper areas, base, and restore appropriately D. Leave the carious material in the deeper areas, temporize appropriately, and observe in two weeks

C. Leave the carious material in the deeper areas, base, and restore appropriately

The dentist is preparing a primary mandibular first molar for a stainless steel crown. Which of the following tooth surfaces will require the least amount of reduction? A. Buccal B. Mesial C. Lingual D. Occlusal

C. Lingual

The soft tissue-tooth interface that forms most frequently after flap surgery in an area previously denuded by inflammatory disease is a A. Collagen adhesion. B. Reattachment by scar. C. Long junctional epithelium. D. Connective tissue attachment.

C. Long junctional epithelium.

An elderly patient has worn a maxillary complete denture against 6 mandibular anterior teeth for an extended period of time. In examining the edentulous mouth of this patient, the dentist would probably see which of the following? A. Intrusion of the mandibular anterior teeth B. Flabby ridge tissue in the posterior maxillary arch C. Loss of osseous structure in the anterior maxillary arch D. Cystic degeneration of the foramina of the anterior palatine nerve

C. Loss of osseous structure in the anterior maxillary arch

In the normal dental pulp, which of the following histologic features is (are) the least likely to appear? A. Cell-free zone of Weil B. Palisade odontoblastic layer C. Lymphocytes and plasma cells D. Undifferentiated mesenchymal cells

C. Lymphocytes and plasma cells

An endodontically-treated permanent mandibular first molar has incipient lesions on its mesial and distal surfaces. During previous treatment, a minimal amount of tooth structure was removed. The appropriate treatment for this tooth is a (an) A. MOD amalgam. B. MOD cast gold inlay. C. MOD cast gold onlay. D. 3/4 crown. E. Full crown.

C. MOD cast gold onlay.

Which of the following teeth have the most consistent canal morphology? A. Mandibular incisors B. Maxillary canines C. Mandibular premolars D. Maxillary premolars E. Mandibular molars

C. Mandibular premolars

Because of its root structure, which of the following teeth is the LEAST responsive to rotation forces during extraction? A. Mandibular canine B. Maxillary central incisor C. Maxillary first premolar D. Mandibular first premolar E. Mandibular second premolar

C. Maxillary first premolar

Which of the following is the current method -of -choice for removing or disrupting bacterial masses in the mouth? A. Biologic B. Chemical (mouthwashes) C. Mechanical (brush and floss) D. Antibiotic (penicillin, tetracycline)

C. Mechanical (brush and floss)

The displacement of the proximal segment in an unfavorable angle fracture of the mandible would be caused by which of the following muscles? A. Digastric B. Geniohyoid C. Medial pterygoid D. Lateral pterygoid

C. Medial pterygoid

Which of the following penicillin drugs cannot be given orally? A. Ampicillin B. Cloxacillin C. Methicillin D. Penicillin G E. Penicillin VK

C. Methicillin

In which stage of tooth development does tetracycline discoloration occur? A. Apposition B. Proliferation C. Mineralization D. Histodifferentiation

C. Mineralization

Which of the following is the most effective way to reduce injury to the pulp during a restorative procedure? A. Prepare dentin with slow-speed burs B. Use anesthetics without vasoconstrictors C. Minimize dehydration of the dentinal surface D. Keep the dentinal surface clean by frequent irrigation

C. Minimize dehydration of the dentinal surface

Which of the following is the most effective way to reduce injury to the pulp during a restorative procedure? A. Prepare dentin with slow-speed burs. B. Use anesthetics without vasoconstrictors. C. Minimize dehydration of the dentinal surface. D. Keep the dentinal surface clean by frequent irrigation. E. Use light, short duration contact with the bur.

C. Minimize dehydration of the dentinal surface.

Which of the following describes primary occlusal trauma? A. It is the first incidence of trauma that a tooth experiences. B. It is a trauma that produces irreversible damage to the periodontium. C. Mobility is caused by excessive forces on a normal periodontium. D. Mobility is caused by excessive forces on a reduced periodontium.

C. Mobility is caused by excessive forces on a normal periodontium.

With an overdose of a cholinergic drug, one would expect to see each of the following signs EXCEPT one. Which one is this EXCEPTION? A. Sweating B. Urination C. Mydriasis D. Bradycardia E. Copious serous saliva

C. Mydriasis

When lowering the floor of the mouth of a patient, a practitioner detaches all or part of which muscles? A. Mylohyoid and geniohyoid B. Mylohyoid and buccinator C. Mylohyoid and genioglossus D. Geniohyoid and genioglossus E. Buccinator and genioglossus

C. Mylohyoid and genioglossus

Which of the following groups of cells are significantly phagocytic? A. Neutrophil and basophil B. Basophil and eosinophil C. Neutrophil and histiocyte D. Neutrophil and lymphocyte E. Plasma cell and lymphocyte

C. Neutrophil and histiocyte

Which of the following agents found in tobacco products cause addiction? A. Tar B. Formaldehyde C. Nicotine D. Carbon monoxide

C. Nicotine

If an incipient carious lesion were inadvertently covered with sealant, the lesion would most likely A. Progress at a slower rate. B. Progress at a more rapid rate. C. Not continue to progress.

C. Not continue to progress.

Which of the following anomalies can occur with a disturbance during the initiation and proliferation stages in tooth development? A. Size B. Shape C. Number D. Calcification

C. Number

Which of the following is an acronym for a marketing strategy to use for reluctant patients? A. SUCCESS B. RECRUIT C. OPTIMEM D. EIEIO

C. OPTIMEM

Which of the following usually causes the slow progress in molar uprighting in an adult patient? A. Appliance friction B. Excessive deep bite C. Occlusal interferences D. Poor patient cooperation E. Lack of anchorage control

C. Occlusal interferences

When cementing a cast restoration, where should the dentist apply the cement? A. Only to the tooth B. Only to the restoration C. On both the restoration and the preparation

C. On both the restoration and the preparation

When is the most appropriate time to permanently modify/alter the occlusion of an acute TMD patient? A. During the first visit, while the patient has the acute symptoms B. Prior to initiating splint therapy, one week after the first visit C. Only after the patient is symptom-free and has shown improvement during splint therapy D. Never, until definitive orthodontic treatment has been started

C. Only after the patient is symptom-free and has shown improvement during splint therapy

Which of the following best describes adjunctive orthodontic treatment? A. Orthodontic therapy performed only with removable appliances B Limited orthodontic treatment to align the front teeth for maximum esthetics C. Orthodontic treatment to enhance restorative and periodontal rehabilitation D. Early treatment of orthodontic problems to prevent more serious malocclusion

C. Orthodontic treatment to enhance restorative and periodontal rehabilitation

Which of the following best describes adjunctive orthodontic treatment? A. Orthodontic therapy performed only with removable appliances B. Limited orthodontic treatment to align the front teeth for maximum esthetics C. Orthodontic treatment to enhance restorative and periodontal rehabilitation D. Early treatment of orthodontic problems to prevent more serious malocclusion

C. Orthodontic treatment to enhance restorative and periodontal rehabilitation

Which of the following groups of muscles are influential in molding the lingual border of the final mandibular impression for a complete denture? A. Hyoglossus, mylohyoid, medial pterygoid, digastric B. Mylohyoid, styloglossus, medial pterygoid, depressor anguloris C. Palatoglossus, superior pharyngeal constrictor, mylohyoid, genioglossus D. Genioglossus, mylohyoid, middle pharyngeal constrictor, posterior belly of the digastric

C. Palatoglossus, superior pharyngeal constrictor, mylohyoid, genioglossus

Which of the following groups of muscles influences the molding of the lingual border of the final mandibular impression for an edentulous patient? A. Hyoglossus, mylohyoid, medial pterygoid, digastric B. Mylohyoid, styloglossus, medial pterygoid, depressor anguli oris C. Palatoglossus, superior pharyngeal constrictor, mylohyoid, genioglossus D. Genioglossus, mylohyoid, middle pharyngeal constrictor, posterior belly of the digastric

C. Palatoglossus, superior pharyngeal constrictor, mylohyoid, genioglossus

Which of the following is the least helpful in determining the anatomical integrity of the temporomandibular joint? A. Arthrography B. Arthroscopy C. Panoramic films D. Computerized tomography E. Magnetic resonance imaging

C. Panoramic films

Methylparaben preservative most likely shows cross-sensitivity with which of the following anesthetics? A. Xylidides B. Ortho-tuluidide derivatives C. Para-aminobenzoic acid esters D. Meta-aminobutyric acid esters

C. Para-aminobenzoic acid esters

During the mixed dentition stage, which of the following appliances should be used as a space maintainer for missing primary molars in the mandibular arch? A. Distal shoe B. Nance holding arch C. Passive lingual arch D. Removable acrylic functional

C. Passive lingual arch

During experimental gingivitis, supragingival plaque undergoes which of the following changes? A. Plaque mass decreases. B. Plaque microflora become more aerobic. C. Plaque microflora become more gram-negative. D. Plaque microflora become predominantly spirochetal.

C. Plaque microflora become more gram-negative.

Which of the following combinations of agents would be necessary to block the cardiovascular effects produced by the injection of a sympathomimetic drug? A. Atropine and prazosin B. Atropine and propranolol C. Prazosin and propranolol D. Phenoxybenzamine and curare E. Amphetamine and propranolol

C. Prazosin and propranolol

For a population, the researcher divides the number of disease cases by the number of people. By so doing, this investigator will have calculated which of the following rates? A. Incidence B. Odds ratio C. Prevalence D. Specificity

C. Prevalence

A properly trimmed wooden wedge will do each of the following EXCEPT one. Which one is this EXCEPTION? A. Protect the gingival tissue B. Provide space for the matrix band C. Prevent overcontouring of the contact area D. Reduce moisture leakage into the cavity preparation

C. Prevent overcontouring of the contact area

Which of the following agents is the least effective in producing topical anesthesia? A. Tetracaine (Pontocaine®) B. Butacaine (Butyn®) C. Procaine (Novocaine®) D. Lidocaine (Xylocaine®) E. Benzocaine

C. Procaine (Novocaine®)

The ideal bone graft should do each of the following EXCEPT one. Which one is this EXCEPTION? A. Induce osteogenesis B. Withstand mechanical forces C. Produce an immunologic response D. Become replaced by host bone

C. Produce an immunologic response

Thrombophlebitis, which occurs after intravenous administration of diazepam, is usually attributed to which of the following substances in the mixture? A. Benzoic acid B. Ethyl alcohol C. Propylene glycol D. Sodium metabisulfite

C. Propylene glycol

Which of the following is derived from the unsaturated fatty acids in cell membranes? A. Histamine B. Serotonin C. Prostaglandins D. Bradykinin E. Endorphins

C. Prostaglandins

A patient who is on dicumarol therapy requires a tooth extraction. Which laboratory test is the most valuable in evaluating this patient's surgical risk? A. Clotting time B. Bleeding time C. Prothrombin time D. Sedimentation rate E. Complete blood cell count

C. Prothrombin time

Which of the following is contraindicated for a formocresol pulpotomy on a primary molar? A. Tooth that is sensitive to sweets B. Exposure of pulp during excavation of caries C. Radiographic evidence of internal resorption D. Radiographic evidence of deep caries approximating the coronal pulp

C. Radiographic evidence of internal resorption

A new patient had root canal therapy performed seven months ago in another country. No historical radiographs are available. The root canal filling appears to be satisfactory, the tooth is asymptomatic, and there is no associated sinus tract. However, a small periapical radiolucency is evident. Which of the following is indicated? A. Incision and drainage B. Nonsurgical retreatment C. Re-evaluation in six months D. Apicoectomy and apical amalgam E. Prescription of an appropriate antibiotic

C. Re-evaluation in six months

A patient returns 10 days following incision and drainage of a submandibular space abscess. The swelling appears to have enlarged despite high doses of antibiotic therapy. What should the dentist do next? A. Insert a larger drain B. Order a CBC with differential C. Repeat the culture and sensitivity tests D. Use proteolytic enzymes to allow the antibiotic to penetrate the abscess

C. Repeat the culture and sensitivity tests

Which of the following sequences will a dentist most likely use in replantating mature avulsed teeth? A. Root canal therapy, repositioning, and splinting B. Repositioning, splinting, and root canal therapy C. Repositioning, splinting, canal debridement, calcium hydroxide therapy, and gutta-percha fill D. Canal debridement, calcium hydroxide therapy, repositioning, splinting, and gutta-percha fill

C. Repositioning, splinting, canal debridement, calcium hydroxide therapy, and gutta-percha fill

Which of the following is an incorrect statement regarding a mucous retention phenomena? A. Is dome shaped B. Appears radio-opaque C. Requires surgical removal D. Lined with respiratory epithelium E. Protrudes from the floor of the maxillary sinus

C. Requires surgical removal

The space for the eruption of permanent mandibular second and third molars is created by the A. Apposition of the alveolar process. B. Apposition at the anterior border of the ramus. C. Resorption at the anterior border of the ramus. D. Resorption at the posterior border of the ramus.

C. Resorption at the anterior border of the ramus.

The light from visible-fight polymerization units can cause which of the following? A. Iritis B. Cataracts C. Retinal damage D. Corneal ulcerations

C. Retinal damage

A static positional record can be used to adjust posterior articulator controls. In order to set the left condylar inclination and the left sideshift, which of the following are minimally required? A. Protrusive checkbite in incisal edge-to-edge position B. Protrusive checkbite in extreme protrusive position C. Right lateral excursive record D. Left lateral excursive record

C. Right lateral excursive record

A 12-year-old boy has a skeletal Class III malocclusion. As he gets older, which of the following cephalometric values generally increases? A. ANB B. SNA C. SNB D. Y-axis

C. SNB

Which of the following effects are common to pentobarbital, diazepam, and meperidine? A. Anticonvulsant and hypnotic B. Analgesia and relief of anxiety C. Sedation and ability to produce dependence D. Amnesia and skeletal muscle relaxation

C. Sedation and ability to produce dependence

The decrease in ventilation caused by morphine and by some of the related opioids depends chiefly upon a decrease in A. Blood PC02 concentration. B. Medullary endorphin activity. C. Sensitivity of the medullary respiratory center to carbon dioxide. D. Sensitivity of aortic and carotid chemoreceptors to blood oxygen concentration.

C. Sensitivity of the medullary respiratory center to carbon dioxide.

What does an anterior crossbite in a primary dentition usually indicate? A. Digit sucking habit B. Self-correcting condition C. Skeletal growth problem D. Developing Class II malocclusion

C. Skeletal growth problem

Which of the following best describes the usual behavioral characteristics of a child who has Down syndrome? A. Crying and stubborn B. Hyperactive and disoriented C. Smiling, affectionate, and easily distracted D. Fearful and unresponsive to verbal communication

C. Smiling, affectionate, and easily distracted

Which of the following best describes a vestibuloplasty? A. Bone is added to the mandible to augment its height. B. Osseointegrated dental implants are placed to support prosthesis. C. Soft tissue is repositioned in order to increase the area on which the denture may be supported. D. The chin is augmented to enhance esthetics. E. Periodontal tissue is removed from around a tooth to establish a biologic width.

C. Soft tissue is repositioned in order to increase the area on which the denture may be supported.

Respiratory difficulty due to blockade of the neuromuscular junction can be produced by which of the following? A. Penicillin B. Sulfonamide C. Streptomycin D. Cephalosporin E. Chloramphenicol

C. Streptomycin

A patient who has a Class III malocclusion presents for treatment. Each of the following is a helpful diagnostic aid to distinguish between maxillary retrusion and mandibular protrusion EXCEPT one. Which one is this EXCEPTION? A. Photography B. Cephalometry C. Study models D. Clinical evaluation

C. Study models

Which of the following toothbrushing techniques is theoretically the most effective? A. Charter B. Stillman C. Sulcular D. Side-to-side

C. Sulcular

Amalgam scrap should be stored in a tightly-sealed container and covered with which of the following? A. Water B. Glycerin C. Sulfide solution D. Sodium hypochlorite solution

C. Sulfide solution

The "S" in DMFS signifies which of the following? A. Score B. Standard C. Surfaces D. Simplified E. Significant

C. Surfaces

A primary advantage of intravenous sedation is which of the following? A. Fewer side-effects from the sedation B. Slower biotransformation for prolonged action C. The ability to titrate individualized dosage D. A smooth and more gradual onset of sedation

C. The ability to titrate individualized dosage

When determining the appropriate dose of systemic fluoride supplement for a child, it is most important for the dentist to consider which of the following? A. The fluoride content of the drinking water B. The child's diet and caries activity C. The child's age and the fluoride content of the drinking water D. The child's weight and the fluoride content of the drinking water

C. The child's age and the fluoride content of the drinking water

When determining the appropriate dose of systemic fluoride supplement for a child, it is most important for the dentist to consider which of the following? A. The fluoride content of the drinking water. B. The child's diet and caries activity. C. The child's age and the fluoride content of the drinking water. D. The child's weight and the fluoride content of the drinking water.

C. The child's age and the fluoride content of the drinking water.

Which of the following is the MOST convenient method for examining an 18-month-old child? A. The child sitting in the dental chair and a parent at the chair side B. A parent sitting in the dental chair and the child sitting on the parent's lap facing forward C. The dentist and parent in a knee-to-knee position with the child's head in the dentist's lap D. The dentist and parent in a knee-to-knee position with the child's head in the parent's lap

C. The dentist and parent in a knee-to-knee position with the child's head in the dentist's lap

A split thickness skin graft or mucosal graft used to complete a vestibuloplasty primarily receives its nourishment and oxygenation from which of the following? A. The normal mucosa at the graft periphery B. The exposed bone directly beneath the graft C. The exposed periosteum that forms the graft bed D. The vasculature in the subepithelial or submucosal layer moved to the site with the graft E. The graft is serving to protect the site and is expected to necrose.

C. The exposed periosteum that forms the graft bed

In the eruption sequence of a primary dentition, which of the following usually occurs? A. The first molar erupts before the central incisor. B. The canine erupts before the lateral incisor. C. The first molar erupts before the canine. D. The lateral incisor erupts before the central incisor. E. The second molar erupts before the first molar.

C. The first molar erupts before the canine.

A minor traumatic injury can result in the enamel shearing-off of the underlying dentin. Traumatized odontoblasts will eventually lead to pulpal degeneration. A. Both statements are TRUE. B. Both statements are FALSE. C. The first statement is TRUE, the second is FALSE. D. The first statement is FALSE, the second is TRUE.

C. The first statement is TRUE, the second is FALSE.

In the process of protecting the host tissue, PMNs also cause tissue destruction. Overall, the role of PMNs is primarily a destructive one. A. Both statements are TRUE. B. Both statements are FALSE. C. The first statement is TRUE, the second is FALSE. D. The first statement is FALSE, the second is TRUE.

C. The first statement is TRUE, the second is FALSE.

Most epidemiologic studies indicate that gingivitis in children is relatively common. A strong positive association between specific nutritional deficiencies and the presence of periodontal disease in children and adults has been demonstrated. A. Both statements are TRUE. B. Both statements are FALSE. C. The first statement is TRUE, the second is FALSE. D. The first statement is FALSE, the second is TRUE.

C. The first statement is TRUE, the second is FALSE.

The Occupational Safety and Health Administration (OSHA) is concerned with regulated waste within the office. OSHA regulates the transportation of waste from the office. A. Both statements are true. B. Both statements are false. C. The first statement is true, the second is false. D. The first statement is false, the second is true.

C. The first statement is true, the second is false.

The use of a rubber dam is considered an essential component of endodontic armamentarium. Exception to its use in a given situation is acceptable if the patient is informed and consents. A. Both statements are true. B. Both statements are false. C. The first statement is true, the second is false. D. The first statement is false, the second is true.

C. The first statement is true, the second is false.

Dentists often use barrier membranes (teflon membranes) to treat osseous defects in an attempt to block which of the following? A. Osseous regeneration B. Connective tissue attachment C. The formation of a long junctional epithelium D. The coronal migration of periodontal ligament cells

C. The formation of a long junctional epithelium

Your patient was referred to an oral and maxillofacial surgeon for an implant, and you were advised that she was going to need a sinus lift procedure with placement of an autogenous bone graft. What is the definition of that graft? A. The graft will use an artificial, bone-like material. B. The graft uses bone from another human being. C. The graft uses the patient's own bone, taken from another site. D. The graft uses bovine bone, or bone from another animal species.

C. The graft uses the patient's own bone, taken from another site.

Which of the following best indicates that a removable partial denture needs to be relined? A. There is a loss of retention. B. There is soreness on the crest of the ridges. C. The indirect retainers are not seated as the extension bases are depressed. D. The acrylic resin teeth are abraded and the occlusal vertical dimension is decreased.

C. The indirect retainers are not seated as the extension bases are depressed.

The presence of a periradicular radiolucency becomes apparent only after which of the following is (are) destroyed? A. The facial or lingual periosteum B. Both facial and lingual cortical plates C. The junction between the cortical and cancellous bone D. The junction between facial and cortical plates

C. The junction between the cortical and cancellous bone

For a complete denture balanced occlusion, the lingual cusps of maxillary posterior teeth on the non-working side contacts which areas of the mandibular posterior teeth? A. The facial inclines of the lingual cusps B. The lingual inclines of the lingual cusps C. The lingual inclines of the facial cusps D. The central fossae

C. The lingual inclines of the facial cusps

A dentist relined a patient's maxillary complete denture. This patient returned repeatedly for adjustments of the erythematous areas on the ridge crest. Which of the following is the most likely cause? A. An allergy to the acrylic resin that was used for relining B. Presence of a pressure spot in the relined impression C. The loss of even centric relation contacts D. A decrease in the occlusal vertical dimension

C. The loss of even centric relation contacts

In constructing a fixed partial denture for a patient, the dentist will use a hygienic pontic. Which of the following will primarily determine the faciolingual dimension of the occlusal portion of this pontic? A. The length of the pontic B. The masticatory force of the patient C. The position of the opposing contact areas D. The width and crestal position of the edentulous ridge

C. The position of the opposing contact areas

In complete denture fabrication, which of the following regulate(s) the paths of the condyles in mandibular movements? A. The height of the cusps of posterior teeth B. The amount of horizontal and vertical overlap C. The size and shape of the bony fossae and the menisci and muscular influence D. The vertical occlusion, centric relation, and degree of compensating curve

C. The size and shape of the bony fossae and the menisci and muscular influence

In developing a canine-protected articulation where the anterior vertical overlap is determined to be less than 2 mm, the posterior cusp height should be kept shallow because the buccal cusps of the posterior teeth will have to assist in protrusive disclusion. A. Both the statement and the reason are correct and related. B. Both the statement and the reason are correct but NOT related. C. The statement is correct but the reason is NOT. D. The statement is NOT correct, but the reason is correct. E. NEITHER the statement NOR the reason is correct.

C. The statement is correct but the reason is NOT.

The hamular notch is important in complete denture construction BECAUSE it aids the dentist in positioning the maxillary posterior teeth. A. Both the statement and the reason are correct and related. B. Both the statement and the reason are correct but NOT related. C. The statement is correct, but the reason is NOT. D. The statement is NOT correct, but the reason is accurate. E. NEITHER the statement NOR the reason is correct.

C. The statement is correct, but the reason is NOT.

The retentive tip of a clasp arm is placed in an undercut because a flexible arm is best suited for reciprocation. A. Both the statement and the reason are correct and related. B. Both the statement and the reason are correct but NOT related. C. The statement is correct, but the reason is NOT. D. The statement is NOT correct, but the reason is correct. E. NEITHER the statement NOR the reason is correct.

C. The statement is correct, but the reason is NOT.

In selecting a dental base, the dentist should give greatest consideration to which of the following? A. The biocompatibility of the base B. The strength of the base C. The thickness of the remaining dentin

C. The thickness of the remaining dentin

How many roots does a primary maxillary first molar have? A. One B. Two C. Three D. Four

C. Three

Low-dose aspirin therapy prevents the formation of thromboemboli by preferentially inhibiting which of the following? A. Phospholipase A2 in the blood vessel walls B. Prostacyclin synthetase in the blood vessel walls C. Thromboxane synthetase in the platelets D. Vitamin K in the liver

C. Thromboxane synthetase in the platelets

When removing an impacted third molar, a dentist divides (sections) the tooth for each of the following reasons EXCEPT one. Which one is this EXCEPTION? A. To shorten the surgical procedure B. To minimize the amount of bone to be removed C. To allow the tooth to be delivered to the occlusal aspect D. To minimize exertion of force necessary to remove the tooth

C. To allow the tooth to be delivered to the occlusal aspect

The crown of an endodontically-treated maxillary lateral incisor is fractured near the gingival margin. The coronal end of the silver cone used in filling that canal is visible at that level. The findings reveal that the existing root canal filling meets all criteria to be judged successful. Which of the following is the best way to obtain the needed post space? A. To prepare it alongside the silver cone using burs and Peeso reamers. B. To carefully grind away the coronal part of the silver cone using round burs or end-cutting burs. C. To remove the silver cone and re-treat the canal using a gutta-percha technique, and then to create the needed post space. D. To remove the silver cone, notch it 4 mm from the apical end, coat it with freshly mixed sealer, replace it in the canal, and "twist-off" the coronal segment.

C. To remove the silver cone and re-treat the canal using a gutta-percha technique, and then to create the needed post space.

Which of the following is the MOST likely to cause gingival recession? A. Occlusal trauma B. Systemic disease C. Toothbrush abrasion D. Improper deflecting contour E. Necrotizing ulcerative gingivitis

C. Toothbrush abrasion

In communicating with children, which of the following should the dental team do? A. Allow the parent to answer questions asked by the child at chair side B. Allow both the parent and the dentist to communicate simultaneously with the child C. Transmit word substitutes for dental procedures and equipment during the appointment D. Attain voice control to build the groundwork for future instructions

C. Transmit word substitutes for dental procedures and equipment during the appointment

In communicating with children, which of the following should the dental team do? A. Allow the parent to answer questions asked by the child at chairside. B. Allow both the parent and the dentist to communicate simultaneously with the child. C. Transmit word substitutes for dental procedures and equipment during the appointment. D. Attain voice control to build the groundwork for future instructions.

C. Transmit word substitutes for dental procedures and equipment during the appointment.

The most common type of active tooth movement in the primary dentition is correction of which type of problem? A. Vertical B. Sagittal C. Transverse D. Space regaining in the maxillary arch E. Space regaining in the mandibular arch

C. Transverse

A mandibular radiolucency can be seen in the radiograph. This radiolucency could be diagnosed as each of the following EXCEPT one. Which one is this EXCEPTION? A. Odontogenic myxoma B. Ameloblastoma C. Traumatic bone cyst D. Central cell granuloma E. Odontogenic keratocyst

C. Traumatic bone cyst

For access preparation, the root canal anatomy of a maxillary second molar with 4 canals requires which outline form? A. Ovoid B. Round C. Triangular D. Trapezoidal E. Square

C. Triangular

Which of the following most often causes the dental trauma involving the primary dentition of children 1 1/2 - 2 1/2 years of age? A. Physical child abuse B. Lack of parental supervision C. Underdeveloped motor coordination D. High frequency of accident-prone profiles

C. Underdeveloped motor coordination

A dentist is restoring a patient's mandibular arch with a removable partial denture (RPD). The RPD will replace the second premolars and all molars on both sides. Which of the following is the best method for recording centric relation? A. Use a plaster record of the interdigitation of teeth B. Manually articulate the casts and secure with sticky wax C. Use the framework that has an occlusal rim attached D. Use a wax registration that covers premolars of the mandibular arch and anterior teeth E. Use an occlusal rim made on the master cast to which is added a soft wax for the registration

C. Use the framework that has an occlusal rim attached

Which of the following provides the best guarantee for sterilization in a heat sterilizer? A. Using a chemical indicator strip or pouch B. Recording the temperature and/or pressure readings from the sterilizer gauges C. Using bacterial spore tests D. Determining the ability of the sterilizer to kill the hepatitis B virus E. Demonstrating inactivation of the tuberculosis bacterium

C. Using bacterial spore tests

A practitioner pickles gold alloy restorations by heating them to redness and plunging them into an acid bath. This procedure can result in which of the following? A. Oxidation of the metal B. Porosity in the casting C. Warpage of the restoration D. Surface roughness of the restoration

C. Warpage of the restoration

The practitioner would like to learn whether or not a patient understands the treatment plan. Which of the following questions is the MOST likely to elicit this information? A. Do you understand what I mean? B. Do you have any questions about this approach? C. What benefits or drawbacks do you see in this approach? D. Is this approach OK with you?

C. What benefits or drawbacks do you see in this approach?

Under which of the following conditions will it be critical to mount a patient's casts on the true hinge axis? A. When the patient has a severe Class II occlusal relation B. When the patient requires several fixed partial dentures C. When the dentist plans to change the vertical dimension through restorations D. When the dentist plans to fabricate dentures with high-cusped teeth on a fully adjusted articulator

C. When the dentist plans to change the vertical dimension through restorations

What dimension of the face has reached the greatest percentage of its adult size at birth? A. Depth B. Height C. Width

C. Width

Which of the following agents is used for HIV infections? A. Amantadine (Symmetrel®) B. Acyclovir (Zovirax®) C. Zidovudine (Retrovir®) D. Ribavirin (Virazole®) E. Isoniazid (Nydrazid®)

C. Zidovudine (Retrovir®)

If there is insufficient tissue from the oral mucosa to close an alveolar cleft, the MOST common method for obtaining soft tissue coverage is by using A. dermis. B. fascia lata. C. a tongue flap. D. Teflon-proplast. E. freeze-dried dura.

C. a tongue flap.

A dentist restored an endodontically treated tooth with a cast post-and-core and a metal ceramic crown. Three months later, the patient calls and complains of pain, especially on biting. Tooth mobility is normal, as are the radiographs. The most probable cause of pain is A. a loose crown. B. psychosomatic. C. a vertical root fracture. D. a premature eccentric contact.

C. a vertical root fracture.

The base of the incision in the gingivectomy technique is located A. in the alveolar mucosa. B. at the mucogingival junction. C. above the mucogingival junction. D. coronal to the periodontal pocket. E. at the level of the cementoenamel junction.

C. above the mucogingival junction.

Sulfur granules are of diagnostic value in suspected cases of A. sarcoidosis. B. candidiasis. C. actinomycosis. D. histoplasmosis.

C. actinomycosis.

Allergic reactions to local anesthetics are caused by A. rapid absorption. B. slow detoxification. C. an antigen-antibody reaction. D. improper administration technique.

C. an antigen-antibody reaction.

A tray for a polysulfide rubber impression that lacks occlusal stops may result in A. holding the tray for the entire time in the mouth. B. a more prolonged period of setting in the mouth before the tray is removed. C. an inaccurate final impression because of permanent distortion during polymerization. D. an inaccurate final impression because of elastic distortion during removal from the mouth.

C. an inaccurate final impression because of permanent distortion during polymerization.

Which of the following statements best describes the action of local anesthetics in inflamed tissue? Local anesthetics A. diffuse or penetrate more rapidly to the nerve due to their ionic charge. B. diffuse or penetrate more rapidly to the nerve because they are predominantly non-ionized. C. are less efficacious because of the decreased pH of extracellular fluid. D. are quickly inactivated by inflammatory mediators. E. generally have a longer duration of action.

C. are less efficacious because of the decreased pH of extracellular fluid.

In MOST instances, intraoral verruca vulgaris develops as the result of A. trauma. B. pipe smoking. C. autoinoculation. D. chronic alcoholism.

C. autoinoculation.

Facial pain is mediated through all of the following pathways EXCEPT A. fifth cranial nerve. B. ninth cranial nerve. C. autonomic nerves. D. cervical spinal nerves.

C. autonomic nerves.

A patient states that for almost a year now, she has had a rubbery, firm, painless nodule within the substance of the parotid gland. The condition described is MOST likely a A. mucocele. B. lymph node. C. benign mixed tumor. D. squamous cell carcinoma. E. sialolith with encapsulation.

C. benign mixed tumor.

To be effective in translating the roots of teeth, an orthodontic appliance must be A. very gentle in action. B. augmented with extraoral force. C. capable of exerting a torque or moment. D. capable of exerting positive, intermittent forces.

C. capable of exerting a torque or moment.

Nausea and vomiting that are associated with administration of opioid analgesics is the result of stimulation of the A. limbic system. B. emetic center. C. chemoreceptor trigger zone (CTZ). D. opioid receptors in the G.I. tract.

C. chemoreceptor trigger zone (CTZ).

The virus that causes acute herpetic gingivostomatitis is closely related to the virus that causes A. mumps. B. measles. C. chickenpox. D. cat-scratch disease.

C. chickenpox.

The primary advantage of an external splint over an internal splint is A. increased rigidity. B. increased retention. C. increased durability. D. conservation of tooth structure.

C. conservation of tooth structure.

To ensure better thermal and protective insulation of the pulp during a capping procedure, calcium hydroxide should be A. applied to a thickness of 3.0 mm. B. placed in all cavity preparations. C. covered with a stronger base. D. preceded by application of a cavity varnish. E. preceded by application of a zinc phosphate cement.

C. covered with a stronger base.

Lymphangioma is MOST closely related to A. hemangioma. B. angiosarcoma. C. cystic hygroma. D. hemangiopericytoma.

C. cystic hygroma.

Establishing drainage is an important aspect of emergency treatment for an acute periradicular abscess. An equally important aspect is A. adjusting the occlusion. B. prescribing appropriate analgesics. C. debriding the canal system of necrotic tissue. D. introducing antimicrobial medicaments into the pulp space.

C. debriding the canal system of necrotic tissue.

Erythromycin is responsible for numerous drug interactions, some of which are fatal. This is because erythromycin A. increases the absorption of many drugs. B. decreases the absorption of many drugs. C. decreases cytochrome P-450 metabolism of other drugs. D. increases renal reabsorption of many drugs. E. decreases distribution of many drugs.

C. decreases cytochrome P-450 metabolism of other drugs.

Succinylcholine blocks neuromuscular transmission by A. inhibiting cholinesterase. B. inhibiting the central nervous system. C. depolarizing the motor endplate of skeletal muscle. D. inducing formation of cholinesterase at the endplate. E. blocking release of acetylcholine at the endplate.

C. depolarizing the motor endplate of skeletal muscle.

After a free gingival graft, the primary source of nutrition for the graft during the first 24 hours is the A. primary vascular anastomosis. B. residual nutrients within the graft. C. diffusion of nutrients from the underlying connective tissue. D. diffusion of nutrients from adjacent vessels of the periodontal ligament

C. diffusion of nutrients from the underlying connective tissue.

The blade type dental implant is most useful for A. single tooth replacement. B. anterior maxillary edentulous areas between first bicuspids. C. edentulous posterior areas without posterior abutment teeth. D. rehabilitation of a totally edentulous ridge (especially the mandible) with a fixed appliance.

C. edentulous posterior areas without posterior abutment teeth.

The maximal or "ceiling" effect of a drug is also correctly referred to as the drug's A. agonism. B. potency. C. efficacy. D. specificity.

C. efficacy.

The duration of activity of diazepam depends mainly on A. tolerance to the drug. B. redistribution of the drug. C. elimination of active metabolites.

C. elimination of active metabolites.

Excessive heating of the acrylic resin during processing should be avoided to prevent A. exothermic heat buildup. B. acrylic resin shrinkage. C. evaporation of the monomer. D. excessive acrylic resin expansion. E. discoloration of the acrylic resin.

C. evaporation of the monomer.

The most important anticoagulant effect of heparin is to interfere with the conversion of A. PTA to PTC. B. PTC to Factor VIII. C. fibrinogen to fibrin. D. prothrombin to thrombin. E. proaccelerin to accelerin.

C. fibrinogen to fibrin.

Aging of the pulp is evidenced by an increase in A. vascularity. B. cellular elements. C. fibrous elements. D. pulp stones.

C. fibrous elements.

Neostigmine is different from physostigmine because neostigmine A. is not a clinically useful drug. B. is well absorbed from the gastrointestinal tract. C. has an additional direct effect at the neuromuscular junction. D. is a reversible cholinesterase inhibitor while physostigmine is an irreversible one. E. has the specificity to selectively block cholinesterase at the neuromuscular junction.

C. has an additional direct effect at the neuromuscular junction.

A first order bend in an orthodontic wire is A. a twist in the wire. B. in the vertical plane. C. in the horizontal plane. D. a horizontal bend with a twist.

C. in the horizontal plane.

A patient presents for try-in evaluation of balanced occlusion of complete maxillary and mandibular dentures. A dentist notes that protrusive excursion results in separation of posterior teeth. This dentist can best correct this problem by A. changing the condylar inclination. B. increasing the incisal guidance. C. increasing the compensating curve. D. using a flat plane cusp for the posterior teeth.

C. increasing the compensating curve.

Loss of the apical seat caused by overinstrumentation is best managed by A. prescribing analgesics and antibiotics. B. filling with a warm gutta-percha technique. C. increasing the file size and decreasing the file length. D. using a chelating agent and packing with calcium hydroxide. E. filling with a gutta-percha cone that is larger than the apical perforation.

C. increasing the file size and decreasing the file length.

There are four basic stages in the pathogenesis of the periodontal lesion. In progressing order, they are A. initial, established, early, and advanced. B. early, incipient, established, and advanced. C. initial, early, established, and advanced. D. incipient, early, established, and advanced.

C. initial, early, established, and advanced.

In an edentulous patient, the coronoid process can A. limit the distal extension of the mandibular denture. B. affect the position and arrangement of the posterior teeth. C. limit the thickness of the denture flange in the maxillary buccal space. D. determine the location of the posterior palatal seal of the maxillary denture.

C. limit the thickness of the denture flange in the maxillary buccal space.

Osteomyelitis of the mandible is most likely to develop because of A. failure of pus to localize. B. a resistant strain of bacteria. C. low resistance of the patient. D. lack of drainage from the infected area.

C. low resistance of the patient.

Pain threshold refers to the A. highest level of pain a patient will tolerate. B. average level of pain a patient will experience. C. lowest level of pain a patient will detect.

C. lowest level of pain a patient will detect.

A dentist surgically removes a maxillary first molar, fracturing a large portion of the palatal root. The root is forced into the antrum and cannot be visualized. Adjacent teeth and bone are normal. The appropriate approach to recover this root is through the A. hard palate in the canine area. B. maxillary incisive fossa medial to the canine. C. maxillary canine fossa above the level of the premolar roots. D. nasoantral wall above the middle concha or turbinate. E. first molar alveolus by enlarging the antral opening.

C. maxillary canine fossa above the level of the premolar roots.

In a restorative problem involving all teeth in the mouth, the protrusive condylar path inclination has a primary influence on the A. incisal guidance. B. anterior teeth only. C. mesial inclines of the mandibular cusps. D. mesial inclines of the maxillary cusps.

C. mesial inclines of the mandibular cusps.

Modifying fear by familiarization is a successful method to use with children who A. are handicapped. B. have a behavior problem. C. mirror their parents' fear of the dentist. D. have had previous traumatic medical experience.

C. mirror their parents' fear of the dentist.

Secondary trauma from occlusion is seen in cases where A. healthy gingival and osseous tissues are present. B. traumatic changes are occurring in periodontal tissues of teeth with normal supporting bone. C. normal occlusal forces cause trauma to the attachment apparatus of teeth with inadequate bone support.

C. normal occlusal forces cause trauma to the attachment apparatus of teeth with inadequate bone support.

The fenestration technique for mandibular vestibuloplasty is based on the premise that the mucosal attachment will A. not cross the skin graft. B. not cross the mucosal graft. C. not cross a fibrous scar band. D. be maintained in its lowered position by the new denture.

C. not cross a fibrous scar band.

Incomplete removal of bacteria, pulp debris, and dentinal shavings is commonly caused by failure to irrigate thoroughly. Another reason is failure to A. use broaches. B. use a chelating agent. C. obtain a straight line access. D. use Gates-Glidden burs.

C. obtain a straight line access.

The thickness of coronal dentin in primary teeth, compared with that in corresponding permanent teeth, is approximately A. one-fourth. B. one-third. C. one-half.

C. one-half.

Shy, submissive children are often the product of parents who are A. empathetic. B. overindulgent. C. overprotective. D. under-affectionate.

C. overprotective.

The drug of choice to treat overdosage with tricyclic antidepressants is A. atropine. B. phenytoin. C. physostigmine. D. pentobarbital. E. an amphetamine.

C. physostigmine.

Which of the following best explains why the dentist should provide a postpalatal seal in a complete maxillary denture? The seal will compensate for A. errors in fabrication. B. tissue displacement. C. polymerization and cooling shrinkage. D. deformation of the impression material.

C. polymerization and cooling shrinkage.

Green and orange stains on maxillary incisors can usually be attributed to A. drugs. B. diet. C. poor oral hygiene. D. fluoride consumption.

C. poor oral hygiene.

A 50-year-old man complains of a burning, aching pain on the side of his face. He experiences paresthesia and itching in the same area. An examination reveals small white scars in the pre-auricular region. The MOST probable diagnosis is A. Bell's palsy. B. psychoneurosis. C. post-herpetic neuralgia. D. auriculotemporal syndrome

C. post-herpetic neuralgia.

Before beginning tooth preparation, the dentist should visualize the outline form to A. establish the convenience form. B. establish the resistance and retention form. C. prevent overcutting and overextension. D. aid in the finish of enamel walls and margins.

C. prevent overcutting and overextension.

Epinephrine antagonizes the effects of histamine by A. preventing the release of histamine. B. acting on the central nervous system. C. producing physiologic actions opposite to that of histamine. D. competitively blocking histamine at the cellular receptor site.

C. producing physiologic actions opposite to that of histamine.

Chlortetracycline acts by interfering with A. cell wall synthesis. B. nucleic acid synthesis. C. protein synthesis on bacterial but not mammalian ribosomes. D. protein synthesis on mammalian but not bacterial ribosomes.

C. protein synthesis on bacterial but not mammalian ribosomes.

When seating a casting, the practitioner usually finds the initial interferences at (on) the A. margins. B. axial walls. C. proximal contacts.

C. proximal contacts.

The endplate potential of the neuromuscular junction and the excitatory postsynaptic potential of the autonomic ganglion is generated by A. the hydrolysis of acetylcholine. B. a greater mobility of potassium ions in the postsynaptic membrane. C. reduction of the resting potential at the postsynaptic area by acetylcholine. D. a nonspecific increase in the conductances of sodium, chloride and potassium ions at the postsynaptic membrane.

C. reduction of the resting potential at the postsynaptic area by acetylcholine.

An edentulous patient has mobile, hyperplastic tissue in her maxillary anterior region. In making impressions for this patient, the dentist should A. use the closed-mouth technique. B. use a high-fusing impression compound. C. register the tissue in its passive position. D. involve maximum pressure.

C. register the tissue in its passive position.

The dentist plans to make an impression of an onlay preparation. To prepare the tissues around the preparation for the procedure, this dentist will use electrosurgery. When correctly used, electrosurgery will A. reduce the gingival height. B. permanently alter the contour of the gingival tissues. C. remove a thin layer of crevicular gingival tissue.

C. remove a thin layer of crevicular gingival tissue.

Fixing solution functions to A. accelerate the film processing. B. remove the atomic silver. C. remove undeveloped silver salts. D. reduce silver ions to metallic silver.

C. remove undeveloped silver salts.

Rigid splinting of an avulsed permanent incisor will most likely lead to A. normal healing response. B. calcific metamorphosis. C. replacement resorption. D. internal resorption. E. root fracture.

C. replacement resorption.

During extraction, the maxillary tuberosity is fractured but remains attached to the periosteum with an intact blood supply. The treatment of choice is to A. remove the tuberosity. B. elevate the flap and place transosseous wires. C. reposition the fragment and stabilize with sutures. D. place the patient on antibiotic therapy and stabilize with titanium miniplate.

C. reposition the fragment and stabilize with sutures.

Non-perforating internal resorption is best managed by A. surgical curettage. B. incision and drainage. C. root canal treatment. D. calcium hydroxide pulpotomy. E. periodic evaluation and recall.

C. root canal treatment.

The anterior maxillary incisors of a middle-aged patient can be given a younger appearance by selective grinding of the enamel. The dentist performs this procedure by A. flattening the incisal edges. B. shortening the incisal edges. C. rounding the incisal point angles. D. moving the facial line angles proximally. E. moving the facial height of curvature gingivally.

C. rounding the incisal point angles.

Trigeminal neuralgia (tic douloureux) is characterized by A. paralysis of one side of the face. B. uncontrollable twitching of one eye. C. sharp, excruciating pain of short duration. D. prolonged episodes of pain in one side of the face. E. dull pain when pressure is applied over the affected area.

C. sharp, excruciating pain of short duration.

The copal resin varnish that is placed in the cavity preparation before the amalgam is condensed provides A. sealing of the margins for the lifetime of the restoration. B. long-term sealing of several years duration. C. short-term sealing of the margins. D. no sealing of the margins.

C. short-term sealing of the margins.

The needle holder has beaks which when compared to the hemostat are A. longer and thicker. B. longer and thinner. C. shorter and stronger. D. shorter and more delicate.

C. shorter and stronger.

The usual metabolic pathway of ingested fluoride involves urinary excretion primarily, with the remaining portion found largely in A. teeth. B. muscle tissues. C. skeletal tissues. D. epidermal tissues.

C. skeletal tissues.

Shrinkage porosity in a gold casting is associated with A. the use of excessive flux. B. the use of no flux. C. sprue diameter. D. alloy oxidation. E. excessive burnout time.

C. sprue diameter.

The correct total liter flow of nitrous oxide-oxygen is determined by A. a standard 6 liter per minute flow. B. the patient's metabolic oxygen requirements. C. the amount necessary to keep the reservoir bag 1/3 to 2/3 full. D. the largest volume that the patient can exchange within 1 minute.

C. the amount necessary to keep the reservoir bag 1/3 to 2/3 full.

Bone marrow for grafting defects in the jaws is generally obtained from A. a rib. B. the tissue bank. C. the iliac crest. D. maxillary tuberosity. E. the mandibular parasymphysis.

C. the iliac crest.

The split thickness skin graft vestibuloplasty provides A. an excellent vestibule, but carries high patient morbidity. B. limited vestibular extension in the anterior lingual mandible. C. the maximum ridge that is available for use in any given patient. D. only temporary increase in vestibular depth as the skin graft contracts duripg healing.

C. the maximum ridge that is available for use in any given patient.

A corrected anterior crossbite is best retained by using A. a bonded lingual or palatal retainer. B. a mandibular acrylic guide plane. C. the overbite achieved during treatment. D. a maxillary Hawley retainer for stabilization.

C. the overbite achieved during treatment.

Radiographs of an asymptomatic, 20-year-old patient reveal a sharply outlined radiolucent lesion in the mandibular first molar area. This 2 cm lesion scallops between the roots of vital teeth. The most probable diagnosis is A. radicular cyst. B. dentigerous cyst. C. traumatic bone cyst. D. odontoma.

C. traumatic bone cyst.

The major reason for loss of transplanted autogenous third molars is A. infection. B. immune rejection. C. traumatic occlusion. D. inadequate root length.

C. traumatic occlusion.

The most likely reason for a sudden temperature elevation six days after surgery is A. S.B.E. B. atelectasis. C. wound infection. D. urinary tract infection.

C. wound infection.

A patient presents with malocclusion and a unilateral, slowly progressing elongation of her face. This elongation has caused her chin to deviate away from the affected side. The MOST probable diagnosis is which of the following? Ankylosis Osteoarthritis Myofascial pain Condylar hyperplasia

Condylar hyperplasia

Each of the following EXCEPT one is an indication for the use of systemic antibiotics in dentistry. Which one is this EXCEPTION? A. Extraction (under local anesthesia) of a tooth for a patient who has acute periapical abscess B. Full mouth extractions for a patient who has acute periodontal disease C. Extraction of a mandibular third molar in a patient who has an acute pericoronitis D. "Dry socket" developing following the extraction of a mandibular third molar

D. "Dry socket" developing following the extraction of a mandibular third molar

When making a bite-wing exposure of the posterior teeth, the recommended vertical angulation may vary from A. -10 degrees to -5 degrees B. -5 degrees to 10 degrees C. 0 degrees to +5 degrees D. +5 degrees to +10 degrees

D. +5 degrees to +10 degrees

In most cases, which of the following is the accepted dose schedule of penicillin in the treatment of actinomycosis of the jaw? A. 250 mg. oral tablets q.i.d., 7 days B. 500 mg. oral tablets q.i.d., 7 days C. 600,000 units intramusculary twice daily for two weeks followed by oral medication D. 10 million units intravenously per day for 10-14 days followed by long-term oral medication.

D. 10 million units intravenously per day for 10-14 days followed by long-term oral medication.

3.6 ml solution of 4% prilocaine contains how many mg? A. 3.6 B. 36 C. 72 D. 144 E. 360

D. 144

An employee has just terminated employment in a dental office. The dentist-employer should retain this individual's medical records for how many more years? A. 1 B. 3 C. 10 D. 30

D. 30

What percentage of the U.S. population does not have dental insurance? A. 35-40% B. 45-50% C. 55-60% D. 65-70%

D. 65-70%

A dentist is examining a tooth that has a probing depth of 5 mm and gingival recession of 3 mm. The total amount of attachment loss is A. 2 mm. B. 3 mm. C. 5 mm. D. 8 mm.

D. 8 mm.

A patient presents with a restricted floor of the mouth, only 6 mandibular anterior teeth, and diastema between several teeth. Which of the following major connectors is appropriate for this patient? A. A lingual bar B. A lingual plate C. A lingual bar with a Kennedy bar D. A lingual plate with interruptions (spaces) in the plate at the diastemas

D. A lingual plate with interruptions (spaces) in the plate at the diastemas

A patient has a large cyst-like lesion in his posterior mandibular region. Which of the following is MOST likely to facilitate an accurate diagnosis of this lesion? A. Exfoliative cytology B. Aspiration C. A radiographic examination D. A microscopic examination of incised tissue E. A periodic clinical examination

D. A microscopic examination of incised tissue

Each of the following represents an appropriate rationale for obtaining a dental radiographic examination EXCEPT one. Which one is this EXCEPTION? A. A patient experiences intermittent pain in a tooth. B. A patient's tooth is mobile. C. A patient's maxillary canine has not erupted. D. A patient has presented for a six- month, periodic examination.

D. A patient has presented for a six- month, periodic examination.

Which of the following is the least effective in removing crevicular plaque? A. Toothpick B. Dental floss C. A soft, nylon toothbrush D. A water-irrigating device

D. A water-irrigating device

Each of the following diseases has been associated with the Epstein-Barr virus EXCEPT one. Which one is this EXCEPTION? A. Burkitt's lymphoma B. Oral hairy leukoplakia C. Infectious mononucleosis D. Adenoid cystic carcinoma

D. Adenoid cystic carcinoma

Which of the following clinical findings has the greatest influence on the type of incision to use in periodontal flap surgery? A. Probing depth B. Frenum attachment C. Depth of the vestibule D. Amount of attached gingiva E. Presence of intrabony defects

D. Amount of attached gingiva

In determining the psychological prognosis for a complete denture patient, one of the most valuable aids is which of the following? A. A classification of the ridge relation B. An evaluation of the resorption of the ridges C. An evaluation of the patient's occupational background D. An evaluation of the patient's present dentures and attitudes

D. An evaluation of the patient's present dentures and attitudes

Which of the following will most likely cause an autogenous tooth transplant to fail during the third year? A. Development of root caries B. Ankylosis of the tooth C. An inflammatory resorption of supporting bone D. An idiopathic, chronic progressive root resorption

D. An idiopathic, chronic progressive root resorption

Which of the following is a local contraindication for the surgical removal of an impacted tooth? A. A nonvital tooth B. Fever of unknown origin C. A history of bleeding disorder D. An undiagnosed ulcer in the oral cavity E. Pale gingiva with a confirmed history of anemia

D. An undiagnosed ulcer in the oral cavity

What is the major difference between a Class V cavity preparation for amalgam and one for composite resin by the acid-etch technique? A. Depth B. Convenience form C. Position of retention points D. Angulation of the enamel cavosurface margins

D. Angulation of the enamel cavosurface margins

Orthodontic correction of which of the following is the most easily retained? A. Diastema B. Rotation C. Expansion D. Anterior crossbite E. Generalized spacing

D. Anterior crossbite

Vesicular lesions precede the formation of ulcers in each of the following EXCEPT one. Which one is this EXCEPTION? A. Herpangina B. Herpes zoster C. Herpetic stomatitis D. Aphthous stomatitis E. Hand-foot-and-mouth disease

D. Aphthous stomatitis

A complete denture patient returns for a 24-hour post-op appointment after insertion of new dentures. The patient points to a sore area on top of the mandibular ridge crest on the posterior right side but no sore spot is visually evident. Which of the following is the first procedure the dentist should perform? A. Reduce the centric contacts on all posterior teeth on the right side of the mandibular denture. B. Using a large acrylic bur, grossly reduce the internal surface of the denture. C. Reduce all eccentric contacts on the right side of the maxillary and mandibular dentures. D. Apply pressure indicator paste to the internal surface of the mandibular denture and check for pressure spots. E. Tell the patient this is normal for the first day of wearing new dentures and to return in one week for another post-op check.

D. Apply pressure indicator paste to the internal surface of the mandibular denture and check for pressure spots.

Premature exfoliation of a primary mandibular canine is most often the sequela of which of the following? A. Caries B. Trauma C. Serial tooth extraction D. Arch length inadequacy

D. Arch length inadequacy

What space management treatment should be planned for a child age 4, missing both primary mandibular first molars with no primate space present? A. Lingual arch B. Distal shoe(s) C. Nance holding arch D. Band-and-loop space maintainer(s) E. Observation; no space maintainer necessary

D. Band-and-loop space maintainer(s)

What space management treatment should be planned for a child age 4, missing both primary mandibular first molars with no primate space present? A. Lingual arch B. Distal shoe(s) C. Nance holding arch D. Band-and-loop space maintainer(s) E. Observation; no space maintainer necessary.

D. Band-and-loop space maintainer(s)

To reduce a patient's salivary flow, a dentist has prescribed atropine. As a result of this medication, the patient might experience which of the following side effects? A. Sedation B. Diarrhea C. Bradycardia D. Blurred vision E. Stomach cramping

D. Blurred vision

Which of the following permanent maxillary teeth is the MOST likely to be crowded out of the arch? A. Lateral incisor B. First premolar C. Second premolar D. Canine

D. Canine

Which of the following is a bactericidal antibiotic used specifically in the treatment of infections caused by Pseudomonas species and indole-positive Proteus species? A. Penicillin V B. Tetracycline C. Dicloxacillin D. Carbenicillin

D. Carbenicillin

Each of the following statements is correct regarding phenytoin EXCEPT one. Which one is the EXCEPTION? A. Causes gingival hyperplasia B. Corrects certain cardiac dysrhythmias C. Is ineffective when administered orally D. Causes toxic effects related to the cerebellum and vestibular system E. Exerts antiseizure activity at doses which do not cause general CNS depression

D. Causes toxic effects related to the cerebellum and vestibular system

In radiographs, which of the following regions of the tooth crown is the MOST often mistaken for a carious lesion? A. Pulp horn B. Marginal ridge C. Cingulum D. Cementoenamel junction

D. Cementoenamel junction

Ameloblastoma of the mandible is similar radiographically to which of the following conditions? A. Osteosarcoma B. Pindborg tumor C. Ameloblastic fibro-odontoma D. Central giant cell granuloma

D. Central giant cell granuloma

Squamous cell carcinoma of the tongue MOST commonly metastasizes to which of the following? A. Lungs B. Liver C. Bones D. Cervical nodes

D. Cervical nodes

An unconscious patient is suspected of having an obstructed airway. How should this patient be managed? A. Protrude the tongue, clear the pharynx, extend the neck, and protrude the mandible B. Extend the neck, clear the pharynx, protrude the tongue, and protrude the mandible C. Extend the neck, protrude the mandible, protrude the tongue, and clear the pharynx D. Clear the pharynx, extend the neck, protrude the mandible, and protrude the tongue

D. Clear the pharynx, extend the neck, protrude the mandible, and protrude the tongue

Which of the following is associated with the condition portrayed in the radiograph? A. Odontodysplasia B. Peutz-Jeghers syndrome C. Amelogenesis imperfecta D. Cleidocranial dysplasia

D. Cleidocranial dysplasia

Five hours ago, a 12-year-old boy fell and fractured his maxillary right central incisor at the level of the gingival tissue. The exposed pulp is vital. For this tooth, which of the following represents the treatment of choice? A. Pulp cap B. Pulpotomy C. An apexogenesis procedure D. Complete root canal treatment E. Pulpectomy and placement of calcium hydroxide

D. Complete root canal treatment

Five hours ago, a 12-year-old boy fell and fractured his maxillary right central incisor at the level of the gingival tissue. The exposed pulp is vital. For this tooth, which of the following represents the treatment-of -choice? A. Pulp cap B. Extraction C. Pulpotomy D. Complete root-canal treatment E. Pulpectomy and placement of calcium hydroxide

D. Complete root-canal treatment

An endodontic instrument separated in the apical third of a root canal. The fragment is 3 mm long and is tightly lodged. No radiographic changes at the apex are evident. The practitioner should A. Extract the tooth. B. Resect the apical section of the root containing the broken instrument. C. Perform an apicoectomy and place a reverse filling. D. Complete the root canal filling to the level of the instrument and observe.

D. Complete the root canal filling to the level of the instrument and observe.

For complete dentures, which of the following three factors affect the correct positioning of lips? A. Face-bow transfer, position of teeth, and correct rest vertical dimension B. Face-bow transfer, thickness of the anterior border, and correct occlusal vertical dimension C. Correct rest vertical dimension, thickness of the anterior border, and position of teeth D. Correct occlusal vertical dimension, thickness of the anterior border, and position of teeth

D. Correct occlusal vertical dimension, thickness of the anterior border, and position of teeth

Each of the following pharmacologic effects is produced by ethanol EXCEPT one. Which one is the EXCEPTION? A. Diuresis B. CNS depression C. Enzyme induction D. Cutaneous vasoconstriction E. Increased gastric acid secretion

D. Cutaneous vasoconstriction

A patient has a mandibular molar with pulp necrosis, pain to percussion, and with no periradicular lesions. The emergency treatment. for this tooth is which of the following? A. Opening the tooth, performing apical trephination, temporizing, and checking the occlusion B. Prescribing analgesics and antibiotics and rescheduling the patient C. Performing pulpotomy, temporizing, and checking the occlusion D. Debriding the canals, temporizing, and checking the occlusion E. Opening the tooth for drainage and leaving the tooth open

D. Debriding the canals, temporizing, and checking the occlusion

According to the American Dental Association (ADA) publication entitled Principles of Ethics and Code of Professional Conduct, a dentist can announce specialization in which of the following? A. Implantology B. Hospital dentistry C. Esthetic dentistry D. Dental public health E. Geriatric dentistry

D. Dental public health

An ameloblastoma is MOST likely to develop in the wall of which of the following cysts? A. Radicular B. Dentigerous C. Nasopalatine D. Branchial cleft

D. Dentigerous

A radiographic examination reveals a radiolucent area in the region of the mandibular left third molar. The third molar is not present. The clinical differential diagnoses should include each of the following EXCEPT one. Which one is this EXCEPTION? A. Ameloblastoma B. Residual cyst C. Odontogenic keratocyst D. Dentigerous cyst E. Odontogenic myxoma

D. Dentigerous cyst

Each of the following are narcotics used in outpatient anesthesia EXCEPT one. Which one is this EXCEPTION? A. Fentanyl B. Sufentanil C. Meperidine D. Diazepam E. Morphine

D. Diazepam

A patient is allergic to both amide and ester anesthetic derivatives. For this patient, the dentist should find which of the following infiltrative local anesthetics to be safe and effective? A. Bupivacaine B. Phenylephrine C. Nitrous oxide D. Diphenhydramine E. Ethylaminobenzoate

D. Diphenhydramine

Among cholinergic agents, a quatemary ammonium group in the structure of the drug appears necessary for which of the following actions? A. Nicotinic anticholinesterase B. Direct nicotinic stimulation C. Muscarinic anticholinesterase D. Direct muscarinic stimulation E. Central nervous system activity

D. Direct nicotinic stimulation

Which of the following root surfaces is the most likely to be strip-perforated during canal instrumentation of the mesial root of a mandibular first molar? A. Facial B. Lingual C. Mesial D. Distal

D. Distal

In patients with Parkinson's disease, which transmitter is deficient? A. Serotonin (5 hydroxy-tryptamine) B. Gaba-amino-butyric acid (GABA) C. Glycine D. Dopamine E. Norepinephrine

D. Dopamine

The dentist would like to compare biologic-risk estimates due to radiation damage. For this procedure, which of the following will be the correct unit of measurement? A. Exposure B. Dose C. Effective dose D. Dose equivalent

D. Dose equivalent

Superficial decalcification and staining are evident in the buccal groove of a mandibular molar. Which of the following is the treatment of choice? A. Oral prophylaxis at 3-month intervals B. Daily irrigation with pressurized water C. Complete elimination of the area by preparation and restoration D. Enameloplasty, limited to the superficial depth of the decalcified enamel

D. Enameloplasty, limited to the superficial depth of the decalcified enamel

Which of the following clinical lesions MOST often reveals histologic evidence of severe dysplasia or carcinoma in situ? A. Leukoedema B. Leukoplakia C. Lichen planus D. Erythroplakia E. White sponge nevus

D. Erythroplakia

In a complete maxillary denture, accurate adaptation of the border of the maxillary facial flange affects which of the following the most? A. Speech B. Support C. Stability D. Esthetics

D. Esthetics

The day after a routine Class V composite was placed, the patient reports discomfort from the tooth. Which of the following is most likely responsible for this complaint? A. No liner or base used B. Over-etching with phosphoric acid C. Too deep an axial depth of the preparation D. Exposure of root dentin during the finishing procedures

D. Exposure of root dentin during the finishing procedures

Which of the following accounts for the main cause of failure of replanted teeth? A. Ankylosis B. Infection C. Pulpal necrosis D. External resorption E. Internal resorption

D. External resorption

Which of the following abnormal findings is diagnostic of a trochlear nerve dysfunction? A. Proptosis B. Dilated pupil C. Eyes fail to move laterally D. Eyes fail to move down and out

D. Eyes fail to move down and out

Which of the following is a CONTRADICTION in terms when describing a patient's profile for orthodontic purposes? A. Convex, posterior divergent B. Concave, anterior divergent C. Concave, posterior divergent D. Flat, bimaxillary dentoalveolar protrusion

D. Flat, bimaxillary dentoalveolar protrusion

Each of the following can occur as a result of a successful root canal treatment EXCEPT one. Which one is this EXCEPTION? A. Apical seal of cementum B. Regeneration of alveolar bone C. Regeneration of the periodontal ligament D. Formation of reparative dentin

D. Formation of reparative dentin

Following a bilateral mandibular fracture in the canine region, the anterior fragment of the mandible is displaced posteriorly by the action of the geniohyoid and by which other muscles? A. Thyrohyoid and mylohyoid only B. Genioglossus and mylohyoid C. Thyrohyoid, mylohyoid, and anterior bellies of the digastric D. Genioglossus and anterior bellies of the digastric

D. Genioglossus and anterior bellies of the digastric

Which of the following is NOT a complication of radiation therapy? A. Mucositis B. Xerostomia C. Osteoradionecrosis D. Granuloma

D. Granuloma

Malignant melanoma most commonly occurs in which of the following intraoral sites? A. Buccal mucosa and vestibule B. Lateral and ventral tongue mucosa C. Soft palate and tonsillar pillar complex D. Hard palate and maxillary gingiva E. Floor of mouth and anterior lingual gingiva

D. Hard palate and maxillary gingiva

Which of the following is MOST likely to play a role in the development of dental anxiety? A. Hearing upsetting stories about going to the dentist B. Being anxious with all health care providers C. Being criticized by the dentist or hygienist for neglect of oral hygiene D. Having had a specific frightening or painful experience in the dental office

D. Having had a specific frightening or painful experience in the dental office

Which of the following represent(s) an early effect of primary traumatic occlusion? A. Vertical pocket formation B. Generalized alveolar bone loss C. Undermining resorption of alveolar bone D. Hemorrhage and thrombosis of blood vessels in the periodontal ligament

D. Hemorrhage and thrombosis of blood vessels in the periodontal ligament

When the dentist enters the operatory, the patient, who is new to the office, stands close to the wall, has his arms folded, and is looking at the floor. The dentist should initiate communication by saying which of the following? A. Let's get going; I've got a lot to do. B. What are you angry about? Didn't my assistant get you seated? C. You seem uncomfortable; did you have a bad dental experience? D. Hi, I'm Doctor Wilson, what brings you here today?

D. Hi, I'm Doctor Wilson, what brings you here today?

The dentist is using polyether material to make impressions. The physical property of this material that should concern this dentist is the A. Coefficient of thermal conductivity. B. Hydrophilic nature. C. Extended setting time. D. High modulus of elasticity.

D. High modulus of elasticity.

High-flow 100% 02 is indicated for treating each of the following types of syncope EXCEPT one. Which one is this EXCEPTION? A. Vasovagal B. Neurogenic C. Orthostatic D. Hyperventilation syndrome

D. Hyperventilation syndrome

A PATIENT SPEAKING ON THE TELEPHONE: "I am terribly embarrassed that I have cancelled both of the appointments I have made with you. Because the last dentist I saw hurt me so much, I am just terrified about coming in." Which of the following responses will be the MOST appropriate for the dentist to make? A. How long has it been since you saw that dentist? Perhaps, it would be helpful to know that modern dentistry is much less painful than dentistry was a few years ago. B. You can be assured that my dental procedures are performed as painlessly as possible. C. If you wish, I can prescribe a tranquilizer for you to take before you come in and, perhaps, do the work under nitrous oxide. D. I can tell that you are very fearful about this. Perhaps you would like to come in just to chat, to get acquainted, and to tell more about it.

D. I can tell that you are very fearful about this. Perhaps you would like to come in just to chat, to get acquainted, and to tell more about it.

Which of the following cranial nerves have parasympathetic activity? A. II, III, VII, IX B. II, V, IX, X C. III, V, VII, IX D. III, VII, IX, X E. V, VII, VIII,

D. III, VII, IX, X

The use of study casts in orthognathic surgery does each of the following EXCEPT one. Which one is the EXCEPTION? A. Constructs splints B. Performs model surgery C. Determines the postoperative occlusion D. Identifies the type of skeletal deformity E. Aids in explaining the surgical procedure to the patient

D. Identifies the type of skeletal deformity

Which of the following represents the appropriate way for a practitioner to manage a patient's hostility and anger? A. Using one's authority as a dentist and issuing a firm warning B. Allowing the patient to control the situation C. Attempting to extinguish the behavior by ignoring it D. Identifying the underlying source of the patient's anger

D. Identifying the underlying source of the patient's anger

In an ideal Class V cavity preparation for amalgam in a mandibular premolar, retention form is gained A. Into the mesial and distal walls. B. In the mesial and occlusal line angles. C. In the axio-occlusal and axiogingival line angles at the expense of the axial wall. D. In the axio-occlusal and axiogingival line angles at the expense of the occlusal and gingival walls.

D. In the axio-occlusal and axiogingival line angles at the expense of the occlusal and gingival walls.

An incorrect occlusal vertical dimension causes a patient to overclose and to have a poor facial profile. To correct this problem, the dentist should do which of the following? A. Increase the rest vertical dimension and increase the interocclusal distance B. Decrease the rest vertical dimension and diminish the interocclusal distance C. Decrease the occlusal vertical dimension and increase interocclusal distance D. Increase the occlusal vertical dimension and diminish the interocclusal distance

D. Increase the occlusal vertical dimension and diminish the interocclusal distance

The presence of Ag in metal-ceramic alloys A. Contributes to which of the following? B. Increases melting temperature C. Increases alloy strength D. Increases alloy corrosion resistance E. May cause green discoloration of the porcelain

D. Increases alloy corrosion resistance

The lower one third of a patient's face appears too short and there is an apparent loss of the vermilion border of the lips. Which of the following procedures is indicated to correct this situation? A. Moving the anterior teeth facially B. Increasing the interocclusal distance C. Decreasing the occlusal vertical dimension D. Increasing the occlusal vertical dimension

D. Increasing the occlusal vertical dimension

Hepatitis B virus infection commonly occurs by each of the following routes EXCEPT one. Which one is this EXCEPTION? A. Sexual intercourse B. Prenatal transfer C. Percutaneous inoculation D. Ingestion

D. Ingestion

Which of the following sympathomimetic agents is the MOST potent bronchodilator? A. Amphetamine B. Norepinephrine C. Phenylephrine D. Isoproterenol E. Methoxamine

D. Isoproterenol

Which of the following characterizes freeze-dried bone? A. It is usually rejected by the host. B. It is exfoliated after a short time. C. It contains cells with osteogenic potential. D. It is eventually replaced by the host bone.

D. It is eventually replaced by the host bone.

Each of the following describes the experimental gingivitis model EXCEPT one. Which one is this EXCEPTION? A. It demonstrates the relationship between plaque formation and gingivitis. B. It supports the non-specific plaque hypothesis. C. It demonstrates that gingivitis is a reversible disease. D. It proves that gingivitis progresses to periodontitis.

D. It proves that gingivitis progresses to periodontitis.

Which of the following lesions is the MOST likely to become malignant? A. Papilloma B. Leiomyoma C. Lymphangioma D. Junctional nevus E. Granular cell tumor

D. Junctional nevus

Which of the following analgesics can be given either orally or by intramuscular injection? A. Aspirin B. Acetaminophen (Tylenol®) C. Ibuprofen (Motrin®, Advil®) D. Ketorolac (Toradol®) E. Naproxen (Naprosyn®, Aleve®)

D. Ketorolac (Toradol®)

Increasing the amount of water in the mix of an improved gypsum die stone will most likely result in which of the following? A. More expansion and more strength B. More expansion and less strength C. Less expansion and more strength D. Less expansion and less strength

D. Less expansion and less strength

Each of the following is an advantage of midazolam over diazepam EXCEPT one. Which one is this EXCEPTION? A. Less incident of thrombophlebitis B. Shorter elimination half-life C. No significant active metabolites D. Less potential for respiratory depression E. More rapid and predictable onset of action when given intramuscularly

D. Less potential for respiratory depression

A healthy 3-year-old child has just had a routine extraction of a primary mandibular molar. Which of the following postoperative problems is the most likely to occur? A. Prolonged bleeding B. Pain C. A dry socket D. Lip bite

D. Lip bite

Which of the following symptoms is NOT commonly associated with acute suppurative osteomyelitis? A. Deep, intense pain B. High, intermittent fever C. Paresthesia or anesthesia of the inferior alveolar nerve D. Loose teeth with suppurative drainage from the periodontal area

D. Loose teeth with suppurative drainage from the periodontal area

Which of the following is the MOST common type of leukemia in children? A. Monocytic B. Granulocytic C. Eosinophilic D. Lymphoblastic

D. Lymphoblastic

In serial extraction procedures, concerns about the eruption sequence are usually related to the eruption pattern of which permanent teeth? A. Maxillary canines and first premolars B. Mandibular first and second premolars C. Mandibular first molars and incisors D. Mandibular canines and first premolars

D. Mandibular canines and first premolars

The amalgam preparation outline for the primary mandibular second molar closely resembles that for which permanent tooth? A. Maxillary second premolar B. Mandibular first premolar C. Mandibular second premolar D. Mandibular first molar

D. Mandibular first molar

A fracture through the angle of the mandible can result in an upward displacement of the proximal fragment. Which of the following groups of muscles produces this movement? A. Digastric and geniohyoid B. Masseter, digastric, and lateral pterygoid C. Masseter, temporal, and lateral pterygoid D. Masseter, temporal, and medial pterygoid

D. Masseter, temporal, and medial pterygoid

Which of the following teeth are the most susceptible to root fracture during extraction? A. Maxillary first premolars B. Maxillary second premolars C. Maxillary third molars D. Mandibular first premolars

D. Maxillary first premolars

An apically displaced flap is generally impossible in which of the following areas? A. Mandibular facial B. Mandibular lingual C. Maxillary facial D. Maxillary palatal

D. Maxillary palatal

In the radiograph, a radiolucency appears at the apex of the maxillary canine. This finding represents a (an) A. Apical rarefying osteitis. B. Nasal fossa. C. Ameloblastoma. D. Maxillary sinus.

D. Maxillary sinus.

The characteristics of effective punishment include each of the following EXCEPT one. Which one is this EXCEPTION? A. Immediacy B. Consistency C. Authority D. Maximum Intensity

D. Maximum Intensity

Each of the following is a common reason for early retirement of dentists EXCEPT one. Which one is this EXCEPTION? A. Carpal tunnel syndrome B. Chronic back pain C. Hypertension D. Mercury toxicity syndrome E. Latex powder sensitivity

D. Mercury toxicity syndrome

Which of the following represents an amphetamine-like drug that is widely used in the treatment of hyperkinetic children? A. Doxapram (Dopram®) B. Hydroxyzine (Atarax®) C. Theophylline (Theobid®) D. Methylphenidate (Ritalin®)

D. Methylphenidate (Ritalin®)

Which of the following is a disadvantage of glass ionomer cement? A. Difficulty in mixing B. Irritation of the pulp C. Low bond strength to dentin D. Moisture sensitivity during initial set

D. Moisture sensitivity during initial set

Examination reveals a soft, fluctuant, tender swelling in the midline of the hard palate. The teeth test vital. Radiographs reveal a radiolucent area projected between the roots of the maxillary central incisors. Which of the following cysts represents the most likely diagnosis? A. Nasolabial B. Median palatal C. Periapical D. Nasopalatine duct E. Incisive papilla

D. Nasopalatine duct

A patient placed an aspirin directly in the mandibular facial sulcus. Shortly afterward, a well-circumscribed white patch appeared on the mucosa. Which of the following is the most likely diagnosis? A. Keratosis B. Hyperplasia C. Atrophy D. Necrosis

D. Necrosis

A patient placed an aspirin directly in the mandibular facial sulcus. Shortly afterward, a well-circumscribed white patch appeared on the mucosa. Which of the following is the most likely diagnosis? A. Keratosis B. Hyperplasia C. Atrophy D. Necrosis E. Hypertrophy

D. Necrosis

In shaping and cleansing the canal of a vital maxillary central incisor, a practitioner has inadvertently perforated the apical foramen. This error can result in each of the following EXCEPT one. Which one is this EXCEPTION? A. Pain to the patient B. Enlargement of the foramen C. Trauma to the apical tissue D. Necrotic tissue being forced into the apical tissues

D. Necrotic tissue being forced into the apical tissues

Antibiotics are useful in the treatment of which of the following? A. Herpangina B. Angina pectoris C. Recurrent aphthous stomatitis D. Necrotizing ulcerative gingivitis

D. Necrotizing ulcerative gingivitis

Which of the following has been associated with localized juvenile periodontitis? A. Cyclic eosinophilia B. Lysis of neutrophils C. Increased phagocytosis D. Neutrophil chemotactic defects

D. Neutrophil chemotactic defects

The lesion in the sinus, seen in the radiograph, has been present for an undetermined amount of time. The patient has neither symptoms nor palpable lymph nodes. The dentist should perform A. Endodontic retreatment. B. Incisional biopsy. C. Excisional biopsy. D. No treatment at this time.

D. No treatment at this time.

Increasing mA setting of an x-ray unit raises which of the following? A. Photon speed B. Photon wavelength C. Number of photons generated D. Voltage between anode and cathode E. Effective energy of the resultant beam

D. Number of photons generated

A Class II cavity preparation in a primary molar for dental amalgam restoration will not require a gingival bevel, because the enamel rods in the area incline A. Facially. B. Lingually. C. Gingivally. D. Occlusally. E. Vertically.

D. Occlusally.

Each of the following is a disadvantage of oral sedation EXCEPT one. Which one is the EXCEPTION? A. The administrator has little control over the ultimate clinical actions of the drug(s). B. There can be significant differences in the bioavailability of oral drugs. C. With most oral drugs, there is a relatively slow onset of clinical activity and a prolonged duration of action. D. Oral anxiolytic drugs are effective in relieving anxiety in the hours immediately preceding the dental appointment.

D. Oral anxiolytic drugs are effective in relieving anxiety in the hours immediately preceding the dental appointment.

Radiographs of a patient's teeth reveal that the crowns are bulbous; the pulps, obliterated; and the roots, shortened. These findings are associated with which of the following? A. Porphyria B. Pierre Robin syndrome C. Amelogenesis imperfecta D. Osteogenesis imperfecta E. Erythroblastosis fetalis

D. Osteogenesis imperfecta

An 18-year-old man complains of tingling in his lower lip. An examination discloses a painless, hard swelling of his mandibular premolar region. The patient first noticed this swelling 3 weeks ago. Radiographs indicate a loss of cortex and a diffuse, radiating pattern of trabeculae in the mass. Which of the following is the most likely diagnosis? A. Leukemia B. Dentigerous cyst C. Ossifying fibroma D. Osteosarcoma E. Hyperparathyroidism

D. Osteosarcoma

When providing endodontic treatment for a patient who has a history of rheumatic heart disease, the dentist should especially avoid which of the following? A. Underinstrumentation of a vital tooth B. Overinstrumentation of a vital tooth C. Underinstrumentation of a necrotic tooth D. Overinstrumentation of a necrotic tooth

D. Overinstrumentation of a necrotic tooth

Which of the following factors is a clinician trying to control by using nitrous oxide? A. Referred pain B. Pain stimulus C. Pain reaction D. Pain threshold

D. Pain threshold

To obtain the best long-term result, which of the following procedures for genioplasty is recommended? A. Silastic® onlay implant B. Proplast® chin implant C. Autogenous onlay bone graft D. Pedicled horizontal sliding osteotomy E. Allogeneic freeze-dried onlay bone graft

D. Pedicled horizontal sliding osteotomy

Corticosteroid therapy for arthritis is contraindicated for a patient who also has which of the following conditions? A. Anemia B. Nephritis C. Alcoholism D. Peptic ulcer E. Rheumatic heart disease

D. Peptic ulcer

A panoramic examination reveals a radiopaque lesion in a patient's maxillary sinus. The patient is asymptomatic. Each of the following should be considered in the differential diagnosis EXCEPT one. Which one is this EXCEPTION? A. Osteoma B. Mucus retention pseudocyst C. Odontoma D. Periapical cyst E. Antrolith

D. Periapical cyst

In assessing a patient's dental fears, the dentist should use each of the following patient factors EXCEPT one. Which one is this EXCEPTION? A. Verbal statements B. Physiological responses C. Behavior D. Personality

D. Personality

Tardive dyskinesia is a neurological side effect of which of the following classes of drugs? A. Alcohols B. Tricyclic antidepressants C. Barbiturate antiepileptics D. Phenothiazine antipsychotics E. Monoamine oxidase inhibitors

D. Phenothiazine antipsychotics

Gingival overgrowth, hirsutism, coarsening of facial features, lymph gland hypertrophy, cognitive deficits, and fetal abnormalities are side effects associated with which of the following antiepileptic drugs? A. Phenobarbital B. Carbamazepine (Tegretol¨) C. Valproate (Depakene¨) D. Phenytoin (Dilantin¨) E. Ethosuximide (Zarontin¨)

D. Phenytoin (Dilantin¨)

Gingival overgrowth, hirsutism, coarsening of facial features, lymph gland hypertrophy, cognitive deficits, and fetal abnormalities are side effects associated with which of the following antiepileptic drugs? A. Phenobarbital B. Carbamazepine (Tegretol®) C. Valproate (Depakene®) D. Phenytoin (Dilantin®) E. Ethosuximide (Zarontin®)

D. Phenytoin (Dilantin®)

In obtaining an incisional biopsy of soft tissue, the dentist should do each of the following EXCEPT one. Which one is the EXCEPTION? A. Infiltrate local anesthetic solution around the intended site B. Place a suture through the intended specimen before removing it C. Obtain some adjacent normal tissue if possible D. Place the specimen in saline, if 10% formalin is unavailable

D. Place the specimen in saline, if 10% formalin is unavailable

Which of the following do polycarboxylate and glass ionomer have in common? A. Zinc oxide B. Polysiloxane C. Phosphoric acid D. Polyacrylic acid E. Ion-leachable glass

D. Polyacrylic acid

Which of the following is frequently accompanied by melanin pigmentation (cafe-au-lait spots)? A. Osteomalacia B. Hyperparathyroidism C. Osteogenesis imperfecta D. Polyostotic fibrous dysplasia

D. Polyostotic fibrous dysplasia

Three days after full mouth extraction, an elderly patient complains of black-and-blue marks on her neck. The most probable diagnosis is A. Thrombocytopenia. B. Capillary fragility. C. Hematoma formation. D. Postoperative ecchymosis.

D. Postoperative ecchymosis.

The cardiac glycosides will reduce the concentration of which ion in an active heart muscle cell? A. Sodium B. Calcium C. Chloride D. Potassium E. Magnesium

D. Potassium

A patient complains that a tooth with a recently placed large MOD restoration is tender to mastication and exhibits occasional thermal sensitivity. Which of the following is the most likely cause? A. Microleakage B. Recurrent caries C. Gingival recession D. Premature occlusion

D. Premature occlusion

Each of the following is a common cause of postsurgical atelectasis (decreased expansion of the alveoli) EXCEPT one. Which one is the EXCEPTION? A. Shallow inspirations B. Pain-limiting movement C. Inactivity after surgery D. Preoperative respiratory infection E. Narcotic analgesic that depress the respiratory drive

D. Preoperative respiratory infection

Which of the following represents the purpose of an open-ended question? A. Eliciting specific information B. Testing patient's truthfulness C. Gathering medical/legal information D. Producing maximal freedom of response E. Probing details of a specific topic

D. Producing maximal freedom of response

The drug-of-choice for the treatment of adrenergically-induced arrhythmias is A. Quinidine. B. Lidocaine. C. Phenytoin. D. Propranolol.

D. Propranolol.

Which of the following represents the classic sign or symptom of an anteriorly displaced disc with reduction? A. Pain B. Closed lock D. Reciprocal click E. Decreased range of motion

D. Reciprocal click

The treatment-of-choice for an external inflammatory root resorption on a non-vital tooth is which of the following? A. Extraction B. Surgical curettage of the affected tissue C. Pulpectomy and obturation with gutta-percha and sealer D. Removal of the necrotic pulp and placement of calcium hydroxide E. Observation since it is a self-limiting process

D. Removal of the necrotic pulp and placement of calcium hydroxide

Which of the following represents the sequence in the replantation of mature, avulsed teeth? A. Repositioning, splinting, and performing root canal therapy B. Performing root-canal therapy, repositioning, and splinting C. Performing canal debridement, performing calcium hydroxide therapy, repositioning, splinting, and filling with gutta- percha D. Repositioning, splinting, performing canal debridement, performing calcium hydroxide therapy, and filling with gutta- percha E. Repositioning and splinting only

D. Repositioning, splinting, performing canal debridement, performing calcium hydroxide therapy, and filling with gutta- percha

Which of the following represents the MOST likely cause of plastic denture teeth being dislodged from the acrylic resin of the denture base? A. Excessive occlusal forces applied B. Occlusal surfaces adjusted excessively C. Retentive undercuts not placed in the teeth D. Residual wax on teeth when processing

D. Residual wax on teeth when processing

Which of the following represents the cause of death in poisoning from an irreversible anticholinesterase, such as diisopropylfluorophosphate? A. Cardiac failure, resulting from excessive vagal stimulation B. Dehydration, resulting from hypermotility of the gastrointestinal tract C. Asphyxia, resulting from increased airway resistance D. Respiratory failure, resulting from paralysis of the intercostals and the diaphragm E. Central nervous system depression, following prolonged excitation

D. Respiratory failure, resulting from paralysis of the intercostals and the diaphragm

An individual's mandible is growing forward less than the maxilla. Each of the following will be an appropriate treatment strategy EXCEPT one. Which one is this EXCEPTION? A. Stimulating the mandibular growth B. Inhibiting the maxillary growth C. Redirecting the maxillary growth D. Retracting the mandibular teeth E. Retracting the maxillary teeth

D. Retracting the mandibular teeth

After the dentist has completed an etching procedure on a Class III composite preparation, the preparation becomes contaminated with saliva. In response, the dentist should do which of the following? A. Blow away the saliva with air, then proceed B. Rinse away the saliva with water, dry the preparation, then proceed C. Wipe away the saliva with a cotton pellet, rinse the preparation with water, dry it with air, then proceed D. Rinse away the saliva with water, dry the preparation with air, then repeat the etching procedure

D. Rinse away the saliva with water, dry the preparation with air, then repeat the etching procedure

Which of the following accounts for sudden relief of severe pain from a dentoalveolar abscess? A. The action of bacteria-produced neurotoxins B. The walling-off of the infection by the body C. The neutralizing effect of tissue enzymes D. Rupture through the periosteum into soft tissue

D. Rupture through the periosteum into soft tissue

An operator has chosen to use a shielded open-ended cone. Which of the following will contribute the most to patient gonadal dose? A. Leakage from the x-ray machine head B. Scatter from the operatory walls C. Scatter from the cone D. Scatter from the patient's face

D. Scatter from the patient's face

Which of the following represents the major route of excretion of penicillin V? A. Secretion in the bile and into the feces B. Excretion in the urine as inactive metabolites C. Total metabolism by the liver and excretion in the feces D. Secretion not metabolized in the urine

D. Secretion not metabolized in the urine

Each of the following is a mode of action of an ultrasonic instrument EXCEPT one. Which one is this EXCEPTION? A. Lavage B. Vibration C. Cavitation D. Sharp cutting edge of tip

D. Sharp cutting edge of tip

The integrity of the floor of the antrum is at greatest risk with surgery involving the removal of (a) A. Torus palatinus. B. Nasopalatine cyst. C. Maxillary third molar. D. Single remaining maxillary molar. E. Supernumerary teeth from the maxillary canine region.

D. Single remaining maxillary molar.

A properly executed posterior superior alveolar nerve block will anesthetize each of the following structures EXCEPT one. Which one is the EXCEPTION? A. Maxillary sinus membrane in the molar region B. Second and third molar teeth and a portion of the first molar tooth C. Buccal alveolar bone, soft tissue and periodontium in the posterior maxilla D. Soft palate mucosa on the side of the injection

D. Soft palate mucosa on the side of the injection

A decrease in which of the following causes an increase in radiographic density? A. Milliamperage (mA) B. Kilovoltage peak (kVp) C. Object-film distance D. Source-film distance E. Exposure time

D. Source-film distance

A decrease in which of the following causes an increase in radiographic density? A. mA B. kVp C. Object-film distance D Source-object distance

D. Source-object distance

Which of the following dimensions are compared in the transitional dentition analysis? A. Arch width to arch length B. Leeway space to freeway space C. Leeway space to size of tooth D. Space available to space required E. The arch perimeter of the primary and transitional dentition

D. Space available to space required

Handpiece stones can be used primarily to sharpen which of the following operative instruments? A. Curved chisels B. Enamel hatchets C. Binangle chisels D. Spoon excavators

D. Spoon excavators

Which of the following species is a usual constituent of floras that are associated with periodontal health? A. Campylobacter recta B. Eubacterium nodatum C. Porphyromonas gingivalis D. Streptococcus gordonii

D. Streptococcus gordonii

Which of the following is MOST appropriate for testing differences between the means of two groups? A. Chi-square test B. Multiple regression analysis C. Correlation coefficient analysis D. Students t-test

D. Students t-test

The 1997 American Heart Association recommendations for prevention of bacterial endocarditis includes which of the following dental procedures for antibiotic coverage? A. Placement of rubber dams B. Orthodontic appliance adjustment C. Local anesthetic injections (nonintraligamentary) D. Subgingival placement of antibiotic fibers or strips E. Restorative dentistry with or without retraction cord

D. Subgingival placement of antibiotic fibers or strips

Which of the following is the best treatment for a recurrent ranula? A. Cryosurgery B. Electrosurgery C. Marsupialization D. Sublingual gland excision

D. Sublingual gland excision

The distolingual extension of a mandibular impression for a complete denture is limited by the action of which of the following muscles? A. Stylohyoid B. Lateral pterygoid C. Medial pterygoid D. Superior constrictor

D. Superior constrictor

Which of the following soft tissue elements (fibers) are commonly associated with relapse following orthodontic rotation of teeth? A. Oblique B. Diagonal C. Horizontal D. Supracrestal

D. Supracrestal

Which of the following antibiotics is found at much higher concentrations in crevicular fluid than in serum? A. Clindamycin B. Penicillin C. Metronidazole D. Tetracycline

D. Tetracycline

In a Class V amalgam preparation for an incipient lesion, the ideal internal form of the preparation has which of the following features? A. The axial wall is flat. B. The mesial and distal walls converge. C. The occlusal and gingival walls converge. D. The axial wall is uniformly deep into dentin.

D. The axial wall is uniformly deep into dentin.

Changing several aspects of oral home-care behavior simultaneously is preferable to doing so sequentially. Focusing on successive approximations to the ideal, oral home-care goal, from the beginning, maximizes the likelihood of success. A. Both statements are TRUE. B. Both statements are FALSE. C. The first statement is TRUE, the second is FALSE. D. The first statement is FALSE, the second is TRUE.

D. The first statement is FALSE, the second is TRUE.

Dental fluorosis occurs in the permanent dentition only. It can be prevented by restricting children's intake of fluoride during the enamel's calcification period. A. Both statements are TRUE. B. Both statements are FALSE. C. The first statement is TRUE, the second is FALSE. D. The first statement is FALSE, the second is TRUE.

D. The first statement is FALSE, the second is TRUE.

When disease factors (pocketing, bone loss, inflammation) are equal in a younger patient and in an older patient, prognosis is usually better in the younger patient. The distribution of the remaining teeth can be an important factor in determining the prognosis. A. Both statements are TRUE. B. Both statements are FALSE. C. The first statement is TRUE, the second is FALSE. D. The first statement is FALSE, the second is TRUE.

D. The first statement is FALSE, the second is TRUE.

The use of a cervical headgear in a 12-year-old prepubertal child will result in each of the following EXCEPT one. Which one is this EXCEPTION? A. The child's bite will open. B. The permanent maxillary first molars will move distally. C. The forward growth of the maxilla will be inhibited. D. The forward growth of the mandible will be enhanced.

D. The forward growth of the mandible will be enhanced.

A fixed partial denture will be supported by both an osseointegrated implant and natural teeth. Which of the following is the MOST serious potential problem? A. The path of insertion will be difficult. B. The implant has no hydroxylapatite coating. C. Esthetics will be difficult to reproduce. D. The implant and natural teeth have different mobility.

D. The implant and natural teeth have different mobility.

Which of the following statements about the flap for the removal of a palatal torus is correct? A. The most optimal flap uses a midline incision which courses from the papilla between teeth #8 and 9 posteriorly to the junction of the hard and soft palates. B. The most optimal flap is a reflection of the entire hard palate mucoperiosteum back to a line between the 2 first molar teeth. C. The most optimal flap uses a midpalatal incision that courses from the palatal aspect of tooth #3 across to the palatal aspect of tooth #14 D. The most optimal flap is shaped like a "double-Y", with a midline incision and anterior and posterior side arms extending bilaterally from the ends of the midline incision.

D. The most optimal flap is shaped like a "double-Y", with a midline incision and anterior and posterior side arms extending bilaterally from the ends of the midline incision.

Which of the following is NOT an advantage of chemical vapor sterilization? A. Cycle time is short. B. It will not char fabrics. C. It will not cause corrosion. D. There is no need for special ventilation. E. Instruments are dry at the end of the cycle.

D. There is no need for special ventilation.

Each of the following describes the properties of improved zinc oxide-eugenol materials EXCEPT one. Which one is this EXCEPTION? A. They provide an excellent marginal seal. B. They have a palliative effect on the dental pulp. C. They have thermal insulation qualities that compare to those of dentin. D. They are easily removed from the cavity preparation.

D. They are easily removed from the cavity preparation.

Which of the following barbiturates MOST readily penetrates the blood-brain barrier? A. Barbital B. Phenobarbital C. Secobarbital D. Thiopental E. Pentobarbital

D. Thiopental

Which of the following adverse reactions of oral contraceptives is the most common and the most serious? A. Hypotension B. Hepatotoxicity C. Uterine neoplasia D. Thromboembolic disorder E. Decreased resistance to infection

D. Thromboembolic disorder

Which of the following is the first symptom that is usually perceived by the patient being administered nitrous oxide? A. Nausea B. Euphoria C. Giddiness D. Tingling of the hands

D. Tingling of the hands

The dentist bevels the gingival margins of a gold onlay preparation. This process serves each of the following EXCEPT one. Which one is this EXCEPTION? A. To remove loose enamel rods B. To facilitate finishing C. To minimize marginal opening D. To minimize the need for gingival extension

D. To minimize the need for gingival extension

Tripod marks are placed on the cast after surveying for which of the following reasons? A. To record the undercut areas on the abutment teeth B. To help mount the cast on the articulator C. To record the orientation of the cast to the articulator D. To record the orientation of the cast to the surveyor

D. To record the orientation of the cast to the surveyor

Which of the following exhibits the most personal behavior by the dentist? A. Leaning toward patient B. Facing directly toward patient C. Sitting 2 feet from patient D. Touching patient gently on arm

D. Touching patient gently on arm

Which of the following is a disorder associated with a chromosomal abnormality that can occur as the result of either chromosomal nondisjunction or translocation? A. Ehlers-Danlos syndrome B. Klinefelter syndrome C. Turner syndrome D. Trisomy 21(Down syndrome)

D. Trisomy 21(Down syndrome)

Hypersensitivity reactions to the penicillin derivatives include which of the following classical types? A. Type I only B. Types I, II, and IV C. Type IV only D. Types I, II, Ill, and IV

D. Types I, II, Ill, and IV

A freshly condensed Class II amalgam restoration has a deficient margin at the proximogingival cavosurface angle. This might have been caused by which of the following? A. Overtightening the matrix band B. Neglecting to wedge the matrix band C. Neglecting to contour the matrix band D. Using too large an initial increment of amalgam

D. Using too large an initial increment of amalgam

Which of the following conditions has the highest 5-year survival rate? A. Stage 1 well-differentiated squamous cell carcinoma involving the floor of the mouth B. Stage 1 well-differentiated squamous cell carcinoma involving the soft palate C. Stage 1 well-differentiated squamous cell carcinoma involving the right lateral tongue D. Verrucous carcinoma of the right lower posterior vestibule E. Verrucous carcinoma of the floor of the mouth

D. Verrucous carcinoma of the right lower posterior vestibule

Each of the following is a common side effect of prolonged tetracycline therapy EXCEPT one. Which one is this EXCEPTION? A. Diarrhea B. Superinfection C. Photosensitivity D. Visual disturbance E. Discoloration of newly forming teeth

D. Visual disturbance

Each of the following is associated with Sjogren's syndrome EXCEPT one. Which one is the EXCEPTION? A. Arthritis B. Xerostomia C. Cervical caries D. Warthin's tumor E. Keratoconjunctivitis sicca

D. Warthin's tumor

Which of the following responses made by the dentist is the MOST likely to increase a patient's adherence to oral hygiene prescriptions? A. You don't want lots of cavities, do you? B. If you take my advice, you can save yourself pain and money. C. You really should spend the time it takes to brush and floss. D. What do you see as some of the problems with this approach? E. I can assure you that this approach is quick and easy.

D. What do you see as some of the problems with this approach?

In the construction of a removable partial denture, when is a lingual plate preferred over a lingual bar connector? A. When more rigidity is required B. When the remaining teeth are widely spaced C. When the remaining anterior teeth are mobile D. When there is no space in the floor of the mouth

D. When there is no space in the floor of the mouth

Which of the following is inherited as an autosomal dominant trait? A. Lichen planus B. Bullous pemphigoid C. Pemphigus vulgaris D. White sponge nevus E. Epidermolysis bullosa acquisita

D. White sponge nevus

Daily cleaning of the root surface by the patient has been shown to A. cause root resorption. B. cause root sensitivity. C. stimulate the epithelial attachment. D. allow remineralization of the root surface.

D. allow remineralization of the root surface.

Following compression of acrylic into the denture flasks, placing the flasks into the processing tanks at curing temperature is delayed to A. assure complete flow of acrylic into the mold. B. allow the monomer to permeate all polymer crystals. C. establish an equalized and uniform pressure in the molds. D. allows the flasks and the acrylic to reach a stable temperature.

D. allows the flasks and the acrylic to reach a stable temperature.

Which of the following clinical findings has the greatest influence on the type of incision to use in periodontal flap surgery? A. probing depth B. frenum attachment C. depth of the vestibule D. amount of attached gingiva E. presence of intrabony defects

D. amount of attached gingiva

Ethambutol is A. an antiviral agent. B. an antifungal agent. C. an antityphoid agent. D. an antituberculous agent. E. a urinary tract antiseptic.

D. an antituberculous agent.

Isoniazid is A. an antiviral agent. B. an antifungal agent. C. an antityphoid agent. D. an antituberculous agent. E. a urinary tract antiseptic.

D. an antituberculous agent.

Porcelain failure in all-ceramic restorations can best be avoided if the A. occlusal forces are minimal. B. ceramic material is not over 1/2 mm in thickness. C. ceramic material is 1 1/2 mm in thickness. D. angles of the preparation are rounded.

D. angles of the preparation are rounded.

Dental phobias are very hard to eliminate because they A. are ego-syntonic. B. cannot be seen. C. become habitual. D. are self-reinforcing.

D. are self-reinforcing.

A 46-year-old male presents for multiple extractions. For the last 10 years, he has been taking 10 mg of prednisone daily for colitis. After consulting with the patient's physician, the practitioner should A. discontinue the patient's steroid therapy preoperatively. B. halve the patient's dose of prednisone to 5 mg on the day of surgery. C. switch the patient to another steroid for the perioperative period. D. ask the patient to continue taking the prednisone and consider temporarily increasing the dose. E. send the patient for serum prednisone levels.

D. ask the patient to continue taking the prednisone and consider temporarily increasing the dose.

A mercury spill in the laboratory or office is appropriately cleaned up by A. removing the mercury, then spraying the area with BAL. B. scrubbing the spill area with hot water and detergent. C. sweeping up the mercury and disposing of it in a plastic bag. D. aspirating the mercury into a wash bottle trap, then dusting the spill area with sulfur powder.

D. aspirating the mercury into a wash bottle trap, then dusting the spill area with sulfur powder.

The extrapyramidal syndrome seen with the antipsychotic agents is due to their action on the A. cerebellum. B. brain stem. C. hypothalamus. D. basal ganglia. E. cerebral cortex.

D. basal ganglia.

In adapting a pontic to the residual ridge, the dentist must maintain a proper biologic and hygienic environment. Therefore, the pontic must NOT A. be convex mesiodistally. B. touch the residual ridge. C. be concave faciolingually. D. be concave in two directions.

D. be concave in two directions.

Currently, the best oral sedative drugs for dentistry fall into the class of A. narcotics. B. barbiturates. C. phenothiazines. D. benzodiazepines.

D. benzodiazepines.

For oral sedation in the dental setting, the most ideal group of agents is A. narcotics. B. barbiturates. C. antihistamines. D. benzodiazepines. E. anticholinergics.

D. benzodiazepines.

During a stepback enlargement of the canal space, one reason for recapitulation after each increase in instrument size is to A. maintain coronal curvature of the canal. B. maintain the apical stop for filling with gutta-percha. C. create a coronal funnel to facilitate filling with gutta-percha. D. clean the apical segment of dentin filings that are not removed by irrigation.

D. clean the apical segment of dentin filings that are not removed by irrigation.

A change in continuity of the occlusal plane is often observed after ankylosis of a tooth. This change is caused by A. differential eruption sequences. B. the ankylosed teeth sinking into the alveolar bone. C. localized growth inhibition of the alveolar process. D. continued eruption of non-ankylosed teeth and growth of the alveolar process.

D. continued eruption of non-ankylosed teeth and growth of the alveolar process.

The axial walls in an MOD cavity prepared for a cast gold onlay should A. form acute angles with the pulpal wall. B. form acute angles with the proximal walls. C. diverge from the gingival walls to the pulpal wall. D. converge from the gingival walls to the pulpal wall.

D. converge from the gingival walls to the pulpal wall.

The central skeletal muscle relaxation produced by depressing the polysynaptic reflex arcs is brought about by all of the following drugs EXCEPT A. diazepam (Valium®). B. lorazepam (Ativan®). C. meprobamate (Equanil®). D. d-tubocurarine (Tubarine®).

D. d-tubocurarine (Tubarine®).

Intensifying screens are used with extraoral radiographic films to A. increase kVp. B. increase exposure time. C. improve image quality. D. decrease radiation to the patient.

D. decrease radiation to the patient.

Alcohol abuse occurs when someone A. drinks more than the legal limit of alcohol. B. experiences withdrawal symptoms when not drinking. C. gets seriously intoxicated on a regular basis. D. drinks enough that it puts their relationships, job, or health in jeopardy.

D. drinks enough that it puts their relationships, job, or health in jeopardy.

A researcher establishes the following null hypothesis: "There is no difference in student achievement between the programmed self -instructional strategy and a conventional lecture-discussion format." In this study, the DEPENDENT variable will be the A. type of instruction used. B. number of students who participated. C. relevance of course material. D. examination scores obtained.

D. examination scores obtained.

Before adhesion can take place between a liquid and a solid, it is essential that the liquid surface A. provide some mechanical interlocking with the solid. B. exhibit a large contact angle with the solid. C. enter into some form of chemical reaction with the solid. D. exhibit a small contact angle with the solid.

D. exhibit a small contact angle with the solid.

Auditory nerve deafness is associated with the use of A. polymyxin B. B. chloramphenicol (Chloromycetin®) C. amphotericin B. D. gentamycin (Garamycin®)

D. gentamycin (Garamycin®)

A 2-day-old developing plaque consists primarily of A. Treponema species. B. Bacteroides species. C. filamentous organisms. D. gram-positive cocci and rods. E. a structureless, non-mineralized pellicle.

D. gram-positive cocci and rods.

The Food and Drug Administration Guidelines for dental radiographic examinations suggest that radiographs should NOT be taken simply to screen for occult lesions, but that every radiograph taken should be evaluated for these lesions. Occult lesions are lesions that A. are painful. B. cause the jaw to swell. C. present with confusing signs or symptoms. D. have no signs or symptoms.

D. have no signs or symptoms.

Proximal grooves for a partial veneer crown preparation for a maxillary central incisor are placed parallel to the A. long axis of the tooth. B. long axis of the clinical crown. C. incisal one-third of the facial surface. D. incisal two-thirds of the facial surface.

D. incisal two-thirds of the facial surface.

A patient experiences numbness of the left upper lip, cheek, and the left side of the nose following a fracture of his midface. This symptom follows a fracture through the A. nasal bone. B. zygomatic arch. C. maxillary sinus. D. infraorbital rim.

D. infraorbital rim.

In a patient sedated with nitrous oxide, sudden, excessive and irrational merriment may mean that the patient A. likes you. B. likes nitrous oxide. C. is entering Stage I. D. is entering Stage II.

D. is entering Stage II.

The disk sensitivity assay for antibiotic activity is used because A. the results are obtainable in a few minutes. B. it distinguishes between bacteriostatic and bactericidal drugs. C. it indicates whether or not an antibiotic has good oral absorption. D. it allows for routine testing of sensitivity to a range of antibiotics.

D. it allows for routine testing of sensitivity to a range of antibiotics.

Acute pyogenic bacterial infections produce A. leukopenia. B. lymphopenia. C. neutropenia. D. leukocytosis. E. lymphocytosis.

D. leukocytosis.

Parallelism of abutment preparations is determined by the A. volume of chamfer. B. degree of convergence. C. angulation of finish lines. D. long axis of the preparations. E. long axis of natural teeth.

D. long axis of the preparations.

The MOST common location for a mucous retention swelling or mucocele is the A. tongue. B. gingiva. C. upper lip. D. lower lip. E. buccal mucosa.

D. lower lip.

A reservoir bag is used on an anesthesia machine in order to A. provide a market for used football bladders. B. provide for urinary incontinence in the sedated patient. C. provide a means of mixing N20 and 02 in measured amounts. D. make up for the difference between the constant gas flow from the machine and the cyclic breathing pattern of the patient. E. None of these

D. make up for the difference between the constant gas flow from the machine and the cyclic breathing pattern of the patient.

Prolonged use of amyl nitrite may result in A. aplastic anemia. B. thrombocytopenia. C. granulocytopenia. D. methemoglobinemia. E. hypoprothrombinemia.

D. methemoglobinemia.

The toxic impurity that can theoretically be found in nitrous oxide gas is A. ozone. B. helium. C. methane. D. nitric oxide. E. cyclopropane.

D. nitric oxide.

DFP and organophosphate insecticides produce their effects by A. blocking nicotinic receptors. B. blocking muscarinic receptors. C. competitively inhibiting cholinesterases. D. non-competitively inhibiting cholinesterases.

D. non-competitively inhibiting cholinesterases.

Loss of a primary right molar in a 3-year-old child requires placement of a A. band and loop. B. distal shoe. C. removable acrylic appliance. D. none of the above.

D. none of the above.

A dentist sees an accessory canal in the middle third of a root of a tooth with a necrotic pulp. A small, radiolucent lesion is adjacent to the accessory canal. After obturation, a radiograph shows no material in the accessory canal. The appropriate treatment is to A. expose the root surgically and curet the lesion. B. expose the root surgically and perform an apicoectomy at the level of the accessory canal. C. re-obturate the canal using a technique that will force the material into the accessory canal. D. observe the case radiographically during the next 6-12 months.

D. observe the case radiographically during the next 6-12 months.

Back pressure porosity is A. the same as occluded gas porosity. B. the result of using an oxidizing flame. C. most likely when the wax pattern is positioned very close to the open end of the ring. D. often evidenced by rounded margins on the casting. E. the same as suck-back porosity.

D. often evidenced by rounded margins on the casting.

Patients who have natural dentitions generate the greatest amount of occlusal force during A. swallowing. B. mastication. C. centric relation. D. parafunctional movements.

D. parafunctional movements.

Trismus, secondary to an inferior alveolar injection, most likely results from A. failure to use an aspirating syringe. B. accidental injection of the solution near a branch of the facial nerve. C. allowing the needle tip to rest beneath the periosteum during injection. D. passing the needle through the medial pterygoid muscle. E. accidental injection of the solution near a major motor branch of the trigeminal nerve.

D. passing the needle through the medial pterygoid muscle.

In pursuit of what the dentist believes is best for the patient, the dentist attempts to control patient behavior. This is known as A. autonomy. B. competence. C. maleficence. D. paternalism.

D. paternalism.

Pain that has no organic basis and that is fixed upon some portion of the anatomy is usually referred to as A. false. B. psychogenic. C. referred. D. phantom.

D. phantom.

A size-30 root canal file is broken 1 mm short of the apex in the distofacial canal of a maxillary second molar. The instrument can neither be removed nor by-passed. The remaining pulp is vital. In this situation, the dentist should complete the root canal treatment and A. amputate the distofacial root. B. perform an apicoectomy of the distofacial root. C. perform an apicoectomy and a retrofill of the distofacial root. D. place the patient on recall for further evaluation.

D. place the patient on recall for further evaluation.

The dentist tried-in the metal framework for a porcelain fused-to-metal crown and the margins were closed. When the completed crown was returned from the lab, the margins are all open. The most likely reason for this is the A. die was overtrimmed. B. lab cut off the margins. C. casting distorted during the porcelain application. D. porcelain proximal contact areas are over-contoured.

D. porcelain proximal contact areas are over-contoured.

Thiazides, which are used in the treatment of hypertension, may require supplemental administration of A. sodium. B. chloride. C. calcium. D. potassium.

D. potassium.

The outstanding advantage of phenytoin over phenobarbital in the treatment of epilepsy is that phenytoin A. does not bring about dependence. B. has a longer duration of action. C. is effective against both grand and petit map D. produces less sedation for a given degree motor cortex depression.

D. produces less sedation for a given degree motor cortex depression.

The chronic use (two or more weeks) of clindamycin (Cleocin ) is not recommended because it may cause A. blood dyscrasias. B. hepatotoxicity. C. renal toxicity. D. pseudomembranous colitis.

D. pseudomembranous colitis.

An 8-year-old child has an exposed vital but inflamed coronal pulp in Tooth #14. The treatment of choice is A. pulpectomy B. a direct pulp cap C. an indirect pulp cap D. pulpotomy

D. pulpotomy

In radiobiology, the "latent period" represents the period of time between A. cell rest and cell mitosis. B. the first and last dose in radiation therapy. C. film exposure and image development. D. radiation exposure and onset of symptoms.

D. radiation exposure and onset of symptoms.

Local anesthetics aid in reducing the flow of saliva during operative procedures by A. blocking the cholinergic nerve endings. B. blocking innervation to major salivary glands. C. blocking efferent parasympathetic nerve pathways. D. reducing sensitivity and anxiety during tooth preparation.

D. reducing sensitivity and anxiety during tooth preparation.

A patient needs relining of both maxillary complete denture and mandibular distal-extension partial denture. The recommended procedure is to A. make impressions of both simultaneously. B. start first on the maxillary complete denture. C. start first with the least stable prosthesis. D. relate first the mandibular partial denture frame.

D. relate first the mandibular partial denture frame.

Ostectomy is a procedure that involves the A. use of an autograft. B. use of an allograft. C. use of a contiguous graft. D. removal of tooth-supporting bone. E. removal of non-tooth supporting tooth.

D. removal of tooth-supporting bone.

Early stage periapical cemental dysplasia is best differentiated from chronic apical periodontitis by A. percussion. B. subjective symptoms. C. periodontal examination. D. results of pulp testing. E. radiographic appearance.

D. results of pulp testing.

A flabby, maxillary anterior ridge under a complete denture is frequently associated with A. V shaped ridges. B. Class II patients. C. osteoporosis. D. retained natural mandibular anteriors.

D. retained natural mandibular anteriors.

Which of the following is the primary reason for using plastic teeth in a removable partial denture? Plastic teeth are A. resistant to wear. B. resistant to stains. C. esthetically acceptable. D. retained well in acrylic resin.

D. retained well in acrylic resin.

Cross-sectional occlusal radiographs are useful for locating the A. hyoid bone. B. mandibular foramen. C. maxillary sinus polyp. D. sialoliths in Wharton's duct.

D. sialoliths in Wharton's duct.

Inflammation from periodontal disease usually extends to the bone marrow following A. the nutrient canals. B. primary trauma from occlusion. C. secondary trauma from occlusion. D. the course of the vascular channels. E. the course of the periodontal ligament.

D. the course of the vascular channels.

If the line of force were applied through a tooth's center of resistance, the tooth would A. tip. B. rotate. C. intrude. D. translate. E. extrude.

D. translate.

The one relation of the condyles to the fossae in which a pure hinging movement is possible is A. centric occlusion. B. retruded contact position. C. postural position of the mandible (rest vertical dimension). D. transverse horizontal axis (terminal hinge position).

D. transverse horizontal axis (terminal hinge position).

Two months after an extraction, a patient becomes ill with hepatitis. The dentist may be at fault if he A. does not wear a mask. B. does not use an antiseptic before injection. C. does not wear gloves when performing surgery. D. uses cold sterilization for his surgical instruments.

D. uses cold sterilization for his surgical instruments.

Digitalis should be given to patients with atrial fibrillation who require quinidine to avoid A. hypokalemia. B. thromboembolism. C. sino-atrial bradycardia. D. ventricular tachyarrhythmias. E. prolongation of P-R interval of the ECG.

D. ventricular tachyarrhythmias.

What percent of the blood alcohol level is the most likely to produce a lethal effect in 50 per cent of the population? A. 0.05% B. 0.10% C. 0.20% D. 0.03% E. 0.05%

E. 0.05%

Endodontic therapy is completed on a tooth with a periapical radiolucency. A marked reduction in the size of the radiolucency can be expected in approximately A. 1 week. B. 3 weeks. C. 1 month. D. 2 months. E. 1 year.

E. 1 year.

How many hours per day should a cervical pull headgear be worn to achieve the most effective results? A. 6 B. 8 C. 10 D. 12 E. 14

E. 14

Which of the following has the best prognosis for treatment? A. A tooth with external apical root resorption B. A tooth with external lateral root resorption C. A tooth with a vertical fracture that extends through the floor of the pulp chamber D. An avulsed tooth that was replanted within one hour E. A tooth with a small area of internal resorption

E. A tooth with a small area of internal resorption

A higher than average mandibular plane angle is often associated with which of the following? A. A deep overbite and a vertical growth pattern B. A deep overbite and a Class I division 2 malocclusion C. A deep overbite and a long lower anterior face height D. A vertical growth pattern and a Class II division 2 malocclusion E. A vertical growth pattern and a long lower anterior face height

E. A vertical growth pattern and a long lower anterior face height

Which of the following statements is true regarding the directly acting cholinomimetics? A. Methacholine has more prominent cardiovascular action than carbachol B. Carbachol and bethanechol are almost completely resistant to action of ACHE C. Bethanechol has some degree of selectively for gastrointestinal and urinary bladder D. smooth muscle E. All of these

E. All of these

Each of the following is a pharmacologic effect of phenothiazines EXCEPT one. Which one is this EXCEPTION? A. Sedation B. An antiemetic effect C. Alpha-adrenergic blockade D. Potentiation of the action of narcotics E. An anticonvulsant

E. An anticonvulsant

Which of the following characterizes replacement resorption? A. Chronic pain B. Apical pathosis C. Rapid progression D. Acute inflammation E. Ankylosis

E. Ankylosis

A vital permanent tooth has an irreversible pulpitis and an open apex. Which of the following represents the treatment-of-choice for this tooth? A. Formocresol pulpotomy B. Indirect pulp therapy C. Calcium hydroxide pulpotomy D. Conventional root canal therapy E. Apexification

E. Apexification

Which of the following is the primary substance or material removed during root planing? A. Stain B. Dentin C. Plaque D. Calculus E. Cementum

E. Cementum

Which of the following MOST commonly causes sinus tracts in the gingival tissues of children? A. Pericementitis B. Periapical cyst C. Periodontal abscess D. Acute periapical abscess E. Chronic periapical abscess

E. Chronic periapical abscess

A prescription includes each of the following parts EXCEPT one. Which one is the EXCEPTION? A. Superscription B. Inscription C. Subscription D. Transcription E. Conscription

E. Conscription

Each of the following methods can be used to control pain EXCEPT one. Which one is this EXCEPTION? A. Cortical depression B. Psychosomatic (hypnosis) methods C. Raising the pain threshold D. Blocking the sensory pathway E. Depression of the autonomic nervous system

E. Depression of the autonomic nervous system

Symptoms of pain and tenderness upon palpation of the temporomandibular joint are usually associated with which of the following? A. Impacted mandibular third molars B. Flaccid paralysis of the painful side of the face C. Flaccid paralysis of the non painful side of the face D. Excitability of the second division of the fifth nerve E. Deviation of the jaw to the painful side upon opening the mouth

E. Deviation of the jaw to the painful side upon opening the mouth

Which of the following is an irreversible cholinesterase inhibitor? A. Edrophonium B. Physostigmine C. Neostigmine D. Pilocarpine E. Diisopropylfluorophosphate

E. Diisopropylfluorophosphate

Which of the following most accurately explains how biofeedback works? A. Reduces cognitive dissonance B. Stimulates sympathetic nervous system C. Relaxes and, to some extent, hypnotizes patient D. Distracts and engages patient in active coping task E. Enables patient to gain control of certain physiological functions

E. Enables patient to gain control of certain physiological functions

In which of the following locations do premalignant lesions of squamous epithelium MOST often appear (in the U.S. population)? A. Palate B. Gingival tissue C. Buccal mucosa D. Dorsum of the tongue E. Floor of the mouth

E. Floor of the mouth

Most recent studies on transport media show that the greatest longevity of PDL cells in avulsed permanent incisors is with A. milk. B. saliva. C. warm water. D. buffered saline. E. Hank's solution.

E. Hank's solution.

The clinical activity of a single intravenous dose (10 mg) of diazepam is most dependent on which of the following? A. Alpha half-life B. Beta half-life C. Renal excretion D. Enzymatic degradation E. Hepatic biotransformation

E. Hepatic biotransformation

Which of the following antihypertensive agents acts directly on arterial smooth muscle to cause vasodilation? A. Methyldopa (Aldomet®) B. Clonidine (Catapres®) C. Guanethidine (Ismelin®) D. Metoprolol (Lopressor®) E. Hydralazine (Apresoline®)

E. Hydralazine (Apresoline®)

Which of the following is the most frequent cause of porosity in a porcelain restoration? A. Moisture contamination. B. Improper metal degassing. C. Use of excessive firing temperature. D. Use of inadequate firing temperature. E. Inadequate condensation of porcelain.

E. Inadequate condensation of porcelain.

Inhalation of amyl nitrite can result in each of the following EXCEPT one. Which is the EXCEPTION? A. Tachycardia B. Coronary artery dilation C. Peripheral arteriolar dilation D. A decrease in arterial blood pressure E. Increased motility of the small bowel

E. Increased motility of the small bowel

During which of the following stages of tooth development does fused or geminated teeth occur? A. Apposition B. Calcification D. Eruption and exfoliation E. Initiation and proliferation

E. Initiation and proliferation

Which of the following statements best describes clindamycin (Cleocin®)? A. It inhibits cell wall synthesis. B. It does not penetrate well into bony tissue. C. It is usually given in combination with erythromycin. D. It is effective against gram-negative bacteria only. E. It is effective against most anaerobes.

E. It is effective against most anaerobes.

Each of the following describes nitrous oxide EXCEPT one. Which one is this EXCEPTION? A. It is widely used by hospital anesthesiologists. B. It is essentially not metabolized. C. It minimally alters cardiac output. D. It is heavier than air. E. It is explosive.

E. It is explosive.

Each of the following describes the experimental gingivitis model EXCEPT one. Which one is this EXCEPTION? A. It demonstrates the relationship between plaque formation and gingivitis. B. It demonstrates that the bacterial ecology changes as plaque accumulates. C. It demonstrates that gingivitis is a reversible disease. D. It proves that gingivitis progresses to periodontitis. E. It supports the non-specific plaque hypothesis.

E. It proves that gingivitis progresses to periodontitis.

The Therapeutic Index (T.I.) of a drug is defined as A. ED50/LD50. B. ED1 /LD99. C. LD1 /ED99. D. LD99/ED1. E. LD50/ED50.

E. LD50/ED50.

A fracture through the neck of the condyle can result in the forward displacement of the condylar head because of the pull of which of the following muscles? A. Temporal B. Masseter C. Buccinator D. Medial pterygoid E. Lateral pterygoid

E. Lateral pterygoid

Metabolism of a drug will usually result in conversion to each of the following EXCEPT one. Which one is the EXCEPTION? A. Inactive form B. More active compound C. Less active compound D. More water-soluble compound E. Less ionized compound

E. Less ionized compound

The Occupational Safety and Health Administration's (OSHA) "Bloodborne Pathogens Standard" deals with each of the following EXCEPT one. Which one is the EXCEPTION? A. Disposal of medical waste B. Exposure Control Plan C. Hepatitis B vaccination D. Instrument sterilization and storage E. Material Safety Data Sheets (MSDS)

E. Material Safety Data Sheets (MSDS)

Which of the following classes of drugs, when combined with a narcotic analgesic, is the MOST likely to produce a fatal drug interaction? A. Cardiac glycosides B. Oral anticoagulants C. Tricyclic antidepressants D. Oral antidiabetic agents E. Monoamine oxidase inhibitors

E. Monoamine oxidase inhibitors

Which of the following cells are the MOST radioresistant to cell death? A. Lymphocytes B. Granulocytes C. Erythroblasts D. Epithelial cells E. Muscle cells

E. Muscle cells

Root planing should be avoided during the modified Widman flap procedure BECAUSE the modified Widman flap margin will be placed apically to the level of altered root surfaces. A. Both the statement and the reason are correct and related. B. Both the statement and the reason are correct but NOT related. C. The statement is correct but the reason is NOT. D. The statement is NOT correct but the reason is an accurate statement. E. NEITHER the statement nor the reason is correct.

E. NEITHER the statement nor the reason is correct.

Which of the following side effects is seen most frequently with administration of nitrous oxide and oxygen? A. Hallucinations/dreams B. Respiratory depression C. Tachycardia D. Hand tremors E. Nausea

E. Nausea

The mucosa of the hard palate is the usual intraoral site for which of the following conditions? A. Mucocele B. Sialolithiasis C. Minor aphthous ulcer D. Major aphthous ulcer E. Necrotizing sialometaplasia

E. Necrotizing sialometaplasia

Which of the following statements best describes why L-dopa eventually becomes ineffective in the treatment of Parkinson's disease? A. L-dopa absorption slows with aging. B. Dopa decarboxylase activity increases with age. C. Dietary intake of pyridoxine speeds L-dopa metabolism. D. Patients gradually develop tolerance because L-dopa induces liver enzymeactivity. E. Neuronal cell loss in the substantia nigra is progressive and continuous over the course of the disease.

E. Neuronal cell loss in the substantia nigra is progressive and continuous over the course of the disease.

Which of the following is the most common postoperative problem associated with mandibular sagittal-split osteotomies? A. Infection B. TMJ pain C. Periodontal defects D. Devitalization of teeth E. Neurosensory disturbances

E. Neurosensory disturbances

Which of the following cells in the inflammatory infiltrate of an acute periodontal abscess are the MOST numerous? A. B-lymphocytes B. T-lymphocytes C. Macrophages D. Plasma cells E. Neutrophils

E. Neutrophils

A 34-year-old black woman presents for a routine oral examination. She is asymptomatic and there is no evidence of decay or tooth destruction, although slight periodontal disease is present. Radiographically, there are periapical radiolucencies present on two mandibular central incisors. Electric pulp testing indicates all teeth are responsive in a similar fashion. Which of the following is the treatment of choice? A. Initiate root canal treatment on the two teeth. B. Test cavities on mandibular central incisors. C. Identify the dark, potentially necrotic pulp chambers by transillumination. D. Refer for evaluation to rule out possible malignancy. E. No treatment is necessary at this time.

E. No treatment is necessary at this time.

Classifications of an impacted mandibular third molar are usually based on the A. extent of soft tissue and/or bone coverage. B. relation of the tooth to the opposing third molar. C. position of the tooth in relation to the long axis of the second molar. D. All of these E. Only (a) and (c) above

E. Only (a) and (c) above

Which of the following is effective for treatment of a penicillinase-producing systemic staphylococcal infection when taken orally? A. Methicillin B. Penicillin V C. Streptomycin D. Moxalactam E. Oxacillin

E. Oxacillin

A patient has the asymptomatic condition seen in the radiograph. The MOST probable diagnosis is A. Hypercementosis. B. Complex odontomas. C. Sickle cell anemia. D. Sclerosing osteitis. E. Periapical cemental dysplasia.

E. Periapical cemental dysplasia.

Which of the following teeth have a natural tendency to drift mesially? A. Primary maxillary second molars B. Permanent mandibular central incisors C. Primary mandibular canines D. Mandibular second premolars E. Permanent maxillary second molars

E. Permanent maxillary second molars

Which of the following is indicated to correct a condition of atropine poisoning evidenced by rapid heart rate, dry mouth and gastrointestinal inactivity? A. Nicotine B. Homatropine C. Epinephrine D. Pilocarpine E. Physostigmine

E. Physostigmine

A dentist would like to obtain block anesthesia of the entire second division of a patient's trigeminal nerve. For this purpose, the dentist should administer an intraoral injection in which of the following? A. Foramen ovale B. Pterygoid plexus C. Foramen spinosum D. Infraorbital foramen E. Pterygopalatine fossa

E. Pterygopalatine fossa

Which of the following is most likely to cause a yellowish discoloration in a primary maxillary incisor following trauma? A. Blood pigments B. Pulp hyperemia C. Internal resorption D. Pulp necrosis E. Pulp chamber calcification

E. Pulp chamber calcification

Which of the following is the least likely use of arthroscopy of the TMJ? A. Diagnosis of disk perforations B. Lysis of adhesions C. Disk manipulation D. Therapeutic lavage E. Repair of disk perforations

E. Repair of disk perforations

A panoramic radiograph of an asymptomatic adult discloses a well-defined 1.5 cm radiolucency below the mandibular canal. The lesion is not palpable. Which of the following is the most likely diagnosis? A. Ameloblastoma B. Traumatic bone cyst C. Odontogenic keratocyst D. Focal osteoporotic bone marrow defect E. Salivary gland inclusion defect (Stafne)

E. Salivary gland inclusion defect (Stafne)

Excretion of an acidic drug will be enhanced if the patient is given which of the following? A. Mannitol B. Aldosterone C. Sodium chloride D. Ammonium chloride E. Sodium bicarbonate

E. Sodium bicarbonate

Stress and illness are often related. The best description of their relationship is which of the following? A. Stress is a primary cause of illness. B. Illness is an adaptation to stress. C. Stress is a psychological reaction. D. They often occur together but are causally unrelated. E. Stress is contributory to illness and illness is usually stressful.

E. Stress is contributory to illness and illness is usually stressful.

Which of the following drugs may cause renal lithiasis (kidney stones) resulting from its limited solubility in water? A. Streptomycin B. Erythromycin C. Acetaminophen D. Deteriorated tetracyclines E. Sulfamethoxazole (a constituent of Bactrim DS)

E. Sulfamethoxazole (a constituent of Bactrim DS)

A dentist will use a buccal coil spring to regain space for a mandibular second premolar. Which of the following is an undesirable side effect that is most commonly associated with this procedure? A. Pain B. Gingival irritation C. Severe mobility of the tooth D. Tendency for the first molar to intrude E. Tendency for the first premolar to rotate

E. Tendency for the first premolar to rotate

Which of the following clinical findings is generally NOT found in advanced osteoarthritis of the temporomandibular joint? A. Crepitus B. Antegonial notching C. Flattening of articular surface D. Tendency toward apertognathia E. Tendency toward prognathism

E. Tendency toward prognathism

A newly condensed amalgam restoration seems to chip away when being carved. What is the likely cause of this problem? A. A low-copper alloy was used. B. Moisture contamination occurred. C. The alloy was incompletely wetted with mercury. D. The amalgam was not condensed with the recommended pressure. E. The amalgam was condensed after its working time elapsed.

E. The amalgam was condensed after its working time elapsed.

Which of the following characterize(s) the histopathologic changes in chronic gingivitis? A. The loss of rete pegs and the dissolution of the basement membrane B. An inflammatory infiltrate in which macrophages predominate C. An inflammatory infiltrate in which neutrophilic leukocytes and mast cells predominate D. The destruction of the principal fibers of the periodontal ligament E. The disruption of the gingival fibers and an inflammatory infiltrate of plasma cells, lymphocytes, and neutrophilic leukocytes

E. The disruption of the gingival fibers and an inflammatory infiltrate of plasma cells, lymphocytes, and neutrophilic leukocytes

A cleft lip occurs following the failure of permanent union between which of the following? A. The palatine processes B. The maxillary processes C. The palatine process with the frontonasal process D. The maxillary process with the palatine process E. The maxillary process with the frontonasal process

E. The maxillary process with the frontonasal process

Which of the following explains why mandibular molars should NOT be placed over the ascending area of the mandible? A. The denture base ends where the ramus ascends. B. The molars would interfere with the retromolar pad. C. The teeth in this area would encroach on the tongue space. D. The teeth in this area would interfere with the action of the masseter muscle. E. The occlusal forces over the inclined ramus would dislodge the mandibular denture.

E. The occlusal forces over the inclined ramus would dislodge the mandibular denture.

Which of the following penicillins would be used to treat a Pseudomonas infection? A. Nafcillin (Unipen) B. Amoxicillin (Amoxil) C. Benzathine penicillin (Bicillin) D. Phenoxymethyl penicillin (Pen-Vee K) E. Ticarcillin (Thar)

E. Ticarcillin (Thar)

What is the purpose of incorporating loops and/or helices into an archwire? A. To increase anchorage B. To lower the modulus of elasticity C. To provide a center of rotation D. To facilitate bodily tooth movement E. To increase the activation range

E. To increase the activation range

Why is it advisable to dispense the liquid component of cement immediately before mixing? A. To avoid absorption of moisture from the air B. To avoid spreading over a large area of the slab C. To allow tempering of the powder by the mixing slab D. To reduce the temperature influence of the mixing slab E. To prevent evaporation of the volatile components

E. To prevent evaporation of the volatile components

Which of the following is a major reason for adding vasoconstrictors to local anesthetic injections? A. To decrease bleeding B. To reduce systemic toxicity C. To enhance the onset of action D. To decrease allergic reactions E. To prolong the duration of anesthesia

E. To prolong the duration of anesthesia

Soft tissue can be damaged by each of the following EXCEPT one. Which one is this EXCEPTION? A. Placement of gingival retraction cord B. Home bleaching treatment C. Use of an interproximal carver D. Placement of a matrix band in a tofflemire retainer E. Topical fluoride treatment

E. Topical fluoride treatment

Excessive vertical dimension of occlusion may result in which of the following? A. Poor denture retention B. Increased interocclusal distance C. Drooping of the corners of the mouth D. Creases and wrinkles around the lips E. Trauma to underlying supporting tissues

E. Trauma to underlying supporting tissues

Which of the following drug groups is currently the mainstay of treatment in depressive psychoneurotic disease? A. Amphetamines B. Phenothiazines C. Benzodiazepines D. Monoamine oxidase inhibitors E. Tricyclic (imipramine-like) antidepressants

E. Tricyclic (imipramine-like) antidepressants

Each of the following measures can minimize fractures of the maxillary alveolar process EXCEPT one. Which one is the EXCEPTION? A. Use of controlled force when using forceps and elevators B. Removal of buccal bone and/or sectioning of teeth C. A thorough presurgical analysis and planning alterations in the surgical approach D. Use of the maxillary pinch grasp to detect expansion of the alveolar bone E. Use of forceps with beaks that grasp the roots more firmly

E. Use of forceps with beaks that grasp the roots more firmly

Spontaneous gingival hemorrhage or acute stomatitis, observed in a pediatric patient under treatment for absence seizures, indicates withdrawal of which of the following anticonvulsant drugs? A. Phenytoin B. Ethosuximide C. Carbamazepine D. Phenobarbital E. Valproic acid

E. Valproic acid

Radiographic signs of trauma from occlusion include each of the following EXCEPT one. Which one is this EXCEPTION? A. Hypercementosis B. Root resorption C. Alteration of the lamina dura D. Alteration of the periodontal space E. Vascular infiltration of the periodontal ligament

E. Vascular infiltration of the periodontal ligament

Which of the following is the most serious result of digoxin intoxication? A. Bradycardia B. Renal failure C. Atrial tachycardia D. Atrial fibrillation E. Ventricular fibrillation

E. Ventricular fibrillation

Three carpules (2 ml carpules, 40 mg/ml) of local anesthetic X are required to obtain adequate local anesthesia. To obtain the same degree of anesthesia with local anesthetic Y, five carpules (2 ml carpules, 40 mg/ml) are required. If no other information about the two drugs is available, then it is accurate to say that drug X A. 0 B. is less potent than drug Y. C. is more efficacious than Y. D. is less efficacious than drug Y. E. X&Y are = in potency & efficacy.

E. X&Y are = in potency & efficacy.

Subconjunctival hemorrhage is most commonly found in which of the following fractures? A. Nasal B. LeFort I C. Frontal sinus D. Zygomatic arch E. Zygomaticomaxillary complex

E. Zygomaticomaxillary complex

A 22-year-old male patient complains of dull pain in the posterior left mandibular region. A radiograph made of this area reveals not only a radiolucency around the first molar roots, but also a radiopacity of bone peripheral to this radiolucency. Which of the following best explains this condition? It is A. normal for this patient. B. a developmental abnormality. C. an error in radiographic technique. D. a manifestation of systemic bone disease. E. a reaction to an apical inflammatory disease.

E. a reaction to an apical inflammatory disease.

Nalidixic acid is best described as A. an antiviral agent. B. an antifungal agent. C. an antityphoid agent. D. an antituberculous agent. E. a urinary tract antiseptic.

E. a urinary tract antiseptic.

A 22-year-old female patient has a diffuse erythema of the gingiva with scattered, isolated ulcers on the tongue and in the oropharynx. The marginal gingiva is edematous and inflamed. Her condition is of three days' duration, is painful; and she has elevated temperature and malaise. The MOST likely diagnosis is A. aphthous stomatitis. B. erosive lichen planus. C. benign mucous membrane pemphigoid. D. necrotizing ulcerative gingivitis. E. acute herpetic gingivostomatitis.

E. acute herpetic gingivostomatitis.

Ethyl alcohol is a good antidote for methanol poisoning because A. it inhibits methanol metabolism. B. it competes successfully with methanol for alcohol dehydrogenase. C. it prevents formation of formaldehyde. D. it prevents damage to the optic nerve. E. all of the above.

E. all of the above.

The main function of the liner used in a casting ring is to A. provide venting of the mold. B. help prevent shrinkage porosity. C. provides easy divesting of the casting. D. provides additional water to the investment mix. E. allows uniform and uninhibited setting expansion of the investment.

E. allows uniform and uninhibited setting expansion of the investment.

Vestibuloplasty is performed by A. using a bone file to smooth projections. B. reducing buccal bone from the alveolar ridge. C. reflecting a flap and using rongeurs to remove bone. D. grafting iliac or rib bone onto the alveolar process. E. apically repositioning the muscle and mucosal attachments to the jaw.

E. apically repositioning the muscle and mucosal attachments to the jaw.

Propranolol (Inderal®) exerts its major antianginal effect by A. dilating coronary arteries. B. dilating systemic blood vessels. C. increasing cardiac contractility. D. stimulating vagal slowing of the heart. E. blocking beta-adrenergic receptors of the heart.

E. blocking beta-adrenergic receptors of the heart.

Inhalation of 100% oxygen is contraindicated for a person who has A. a flail chest. B. pneumothorax. C. acute bronchitis. D. acute viral pneumonia. E. chronic obstructive lung disease.

E. chronic obstructive lung disease.

A patient has two or more of her teeth joined only by cementum. This phenomenon is known as A. fusion. B. hypercementosis. C. gemination. D. dilaceration. E. concrescence.

E. concrescence.

If periapical radiolucency is present at the apex of a tooth with a middle third root fracture and the apical canal space is non-negotiable, the required treatment would be A. extraction because of the poor prognosis. B. calcium hydroxide treatment of the coronal segment only. C. apical surgery with a Super EBA reverse filling. D. apical surgery with removal of the apical segment root canal treatment on the coronal segment. E. extraction with removal of the apical segment and replantation of the coronal segment.

E. extraction with removal of the apical segment and replantation of the coronal segment.

Cycloplegia can be produced by A. ephedrine. B. pilocarpine. C. amphetamine. D. acetylcholine. E. homatropine bromide.

E. homatropine bromide.

The highest incidence of fibrous gingival enlargement (hyperplasia) is related to A. puberty. B. diabetes. C. leukemia. D. pregnancy. E. medication.

E. medication.

The phenomenon whereby various light sources produce different perceptions of color is called A. fluorescence. B. incandescence. C. opalescence. D. translucency. E. metamerism.

E. metamerism.

A negative response to the electric pulp tester immediately after a severe luxation of a tooth indicates that the A. pulp is inflamed. B. electric pulp tester is giving a false reading. C. pulp is necrotic and should be removed. D. blood supply is interrupted, and the negative response may be temporary. E. neural transmission is interrupted, and the negative response may be temporary.

E. neural transmission is interrupted, and the negative response may be temporary.

In a complete denture patient, when the teeth, occlusion rims, and central bearing point are in contact and the mandible is in centric relation, the length of the face is known as the A. interocclusal rest space. B. vertical dimension. C. physiologic rest position. D. rest vertical dimension. E. occlusal vertical dimension.

E. occlusal vertical dimension.

To prevent exposure of a dehiscence or fenestration on a prominent root, the dentist's best approach when elevating a flap is to use a (an) A. double flap. B. stripping procedure. C. full thickness flap. D. apically positioned flap. E. partial or split thickness flap.

E. partial or split thickness flap.

Alcoholic euphoria results from A. increased activity of the cerebrum. B. increased activity of thalamic areas. C. increased activity of limbic synapses. D. decreased activity of medullary centers. E. removal of the inhibitory effect of the cortex.

E. removal of the inhibitory effect of the cortex.

A patient experienced a blow to the mouth. Radiographs show a horizontal mid-root fracture of a maxillary central incisor. The tooth is NOT mobile and NOT symptomatic; however, it does not respond to pulp testing. No radiographic lesion is present. The best treatment is to A. institute root canal treatment to include both segments of the tooth. B. extract the coronal segment and surgically remove the apical segment. C. surgically remove the apical segment and reverse fill the coronal segment. D. reform root canal treatment and place an endodontic endosseous implant pin to hold the two segments together. E. render no treatment at this time and periodically recheck clinically and radiographically.

E. render no treatment at this time and periodically recheck clinically and radiographically.

A tooth with an incomplete crown fracture A. should be treated immediately with a direct pulp cap. B. requires placement of a stainless steel crown to provide good pulpal protection. C. does not need to be evaluated with thermal and electric pulp tests. D. requires recontouring of chipped enamel. E. requires vitality tests to determine the extent of pulpal damage.

E. requires vitality tests to determine the extent of pulpal damage.

The chronic use (2 or more weeks) of clindamycin (Cleocin) is not recommended because A. many people develop allergies while on the drug. B. severe hepatotoxicity frequently occurs. C. severe renal toxicity frequently occurs. D. there is a high incidence of blood dyscrasias. E. there is a high incidence of pseudomembranous colitis.

E. there is a high incidence of pseudomembranous colitis.

Which of the following is the PRIMARY DETERMINANT of the outline form of a Class V preparation? Tooth anatomy Height of gingival crest Extension of the carious lesion Restorative material to be placed

Extension of the carious lesion

Which pair of anesthetics is most likely to show cross-allergy? A. Lidocaine - mepivacaine B. Prilocaine - tetracaine C. Procaine - mepivacaine D. Procaine - lidocaine E. Lidocaine - benzocaine

Lidocaine - mepivacaine


Kaugnay na mga set ng pag-aaral

Business Communication final exam

View Set

Anatomy: Movements of Synovial Joints (Ch. 9)

View Set

AP Italiano- Governo-READING FACTS and FACTS PAGINE 5 & 6 del packet(da SC3-p 250-252 ed altro)-la costituzione e il governo italiano-2024

View Set

Quiz 5 Information Security Fundamentals

View Set

Chapter 16: Drug Therapy to Decrease Pain, Fever, and Inflammation Supplement

View Set